Você está na página 1de 187

co

www.BankExamsToday.com,Continuous Job Updates For


Every hour CRT

w
w

.B

an
k

Ex

am
sT

od
ay
.

Arithmetic
Aptitude

Get banking study material on BankExamsToday.com Page 1

www.BankExamsToday.com,continuous job updates for every hour


CRT

od
ay
.
am
sT

w
w

.B

an
k

Ex

1. Basic Calculations
2. Number Systems
3. L.C.M & H.C.F
4. Percentages
5. Profit & Loss
6. Averages
7. Ratio & Proportions
8. Time & Distance
9. Time & Work
10.Mensurations 2D & 3D
11. Interest
12. Clocks
13. Calendars
14. Probability
15. Partnership
16. Mixture or Allegation
17. Problem on Ages
18. Problem on Trains
19. Boats and Streams
20. Pipes & Cisterns

co
m

Index

Get all banking study material on BankExamsToday.com

Page 2

www.BankExamsToday.com,continuous job updates for every hour


CRT
BASIC CALCULATIONS
VBODMAS

:
:
:
:
:
:
:

- (bar)
[{( )}]
of

x
+
-

Note: There are three brackets. 1. ( )


2. { }
They are removed strictly in the order ( ), { } and [ ].

1.

Simplify: 4

1
2

1
5

1
1
of5
2
3

11

9
16
of
2
3

11

3 1

Sol: Given expression


9
2

16
5

5
4

9
2

16
5

9
2

16
5

9
2

16
5

9 16
of
2
3

69
8

9
2

16
5

9 16
2 3

69
8

9
2

16
5

1
24

9
2

16

1035
120

9
2

.B
w
w

5
8

5
8

9
16
of
2
3

11

9
16
of
2
3

11

19
8

an
k

1
4

Ex

3. [ ]

am
sT

Solved Example:

od
ay
.

V for Vinculum
B for Bracket
O for Of
D for Division
M for Multiplication
A for Addition
S for Subtraction

co
m

The order of various operations in exercises involving brackets and functions must be performed strictly
according to the order of the letters of the word VBODMAS. Each letter of the word VBODMAS stands as
follows:

5
8

69
8

1051
540 1051
=
120
120
511
120

Get all banking study material on BankExamsToday.com

Page 3

www.BankExamsToday.com,continuous job updates for every hour


CRT
Square Root And Cube Root
Square: A number multiplied by itself is known as the square of the given number.
E.g. square of 3 is 3 x 3 = 9
Square Root: Square root of a given number is that number which when multiplied by itself is equal to the
given number. It is denoted by the symbol

.
2

Thus,

co
m

E.g. square root of 16 is 4 because 4 = 4 x 4 = 16

16 = 4.

Methods of finding the Square Root:

od
ay
.

1.
2.
3.

Prime Factorization Method: This method is used when the given number is a perfect square or when
every prime factor of that number is repeated twice. Follow the steps as mentioned below.
First find the prime factors of the given number.
Group the factors in pairs.
Take one number from each pair of factors and then multiply them together. This product is the square root
of the given number.

E.g. Find the square root of 225.


Sol:
225 = 5 x 5 x 3 x 3
So, 225 = 5 x 3 = 15.
II.

Method of Division: This method is used when the number is large and the factors cannot be easily
determined.
Find the square root of 180625.

w
w

.B

an
k

Ex

E.g.

am
sT

I.

So, the square root of 180625 i.e. 180625 is 425.

Explanation:

First separate the digits of the number into periods of two beginning from the right. The last period may be
either single digit or a pair.
Find a number (here it is 4) whose square may be equal or less then the first period (here it is 18).
Find the remainder (here it is 2) and bring down the next period (here it is 06).
Double the quotient (here 4) and write to the left (here 8).
The divisor of this stage will be equal to the above sum (here 8) with the quotient of this stage (here 2)
suffixed to it (here 82).
Repeat this process till all the periods get exhausted.
The final quotient is equal to the square root of the given number (here it is 425).

1.

2.
3.
4.
5.
6.
7.

Get all banking study material on BankExamsToday.com

Page 4

www.BankExamsToday.com,continuous job updates for every hour


CRT
Square root of a Decimal: If the given number is having decimal, separate the digits of it into periods of t wo to
the right and left starting from the decimal point and then proceed as followed in the example.

So, 1.498176 = 1.224

od
ay
.

co
m

E.g. 1. Find the square root of 1.498176.

Note: The square root of a decimal cannot found exactly, if it has an odd number of decimal places.

am
sT

Try with finding the square root of 0.1790136


Square Root of a Fraction:

Case 1: If the denominator is a perfect square, the square root is found by ta king the square root of the
numerator and denominator separately.

2601
=
49

2601
49

51 51

7 7

2
51
=7
7
7

an
k

Sol:

2601
.
49

Ex

E.g. Find the square root of

.B

Case 2: If the denominator is not a perfect square, the fraction is converted into decimal and then square root is
obtained or the denominator is made perfect square by multiplying and dividing a suitable number and then its
square root can be determined.
E.g. Find the square root of
461
=
8

461 2
=
8 2

w
w

Sol:

461
.
8

922
16

30 .3644
= 7.5911 (nearly)
4

Cube: Cube of a number is obtained by multiplying the number itself thrice.

E.g. 64 is the cube of 4 as 64 = 4 x 4 x 4.


Cube Root: The cube root of a number is that number which when raised to the third power produces the given
number, that is the cube root of a number a is the number whose cube is a.
The cube root of a is written as

a.

Get all banking study material on BankExamsToday.com

Page 5

www.BankExamsToday.com,continuous job updates for every hour


CRT
Methods to find Cube Root:
1. Method of Factorization:
a.
First write the given number as product of prime factors.
b.
Take the product of prime numbers, choosing one out of three of each type. This product gives
the cube root of the given number.
E.g. Find the cube root of 9261.

so,
2.

9261

co
m

Sol:9261 = 3 x 3 x 3 x 7 x 7 x 7
3 7 = 21

Method to find Cube Roots of Exact Cubes consisting the numbers up to 6 Digits:

then Cube root ends in


1
8
7
4
5
6
3
2
9
0

am
sT

If the cube ends in


1
2
3
4
5
6
7
8
9
10

Example
1
8
27
64
125
216
343
512
729
1000

Ex

Sl. No
1
2
3
4
5
6
7
8
9
10

od
ay
.

Before we discuss the actual method it is better to have an overview of the following table.

an
k

The method of finding the cube root of a number up to 6 digits which is actually a cube of some number
consisting of 2 digits can be well explained with the help of the following examples.
E.g. 1. Find the cube root of 19683.

Sol: First make groups of 3 digits from the right side.

w
w

.B

19,683 : 19 lies between 2 3 and 3 3 , so left digit is 2.


687 ends in 3, so right digit is 7. [See the table.]
Thus, the cube root of 19683 is 27.
E.g. 2. Find the cube root of 614125.

614 125 : 614 lies between 8 3 and 9 3 , so left digit is 8.


125 ends in 5, so right digit is 5. [See the table.]
Thus, the cube root of 614125 is 85.

Exercise
1.

Let a and b be two integers such that a + b = 10. Then the greatest value of a x b is ___
1. 20
2. 100
3. 21
4. 24

2.

If a factory A makes x cars in an hour and another factory B, makes y cars every half an hour, how many cars will
both factories make in 4 hours?
1. 4x+4y
2. 4x+8y
3. 8x+4y
4. 4x_2y

Get all banking study material on BankExamsToday.com

Page 6

www.BankExamsToday.com,continuous job updates for every hour


CRT
3.

Which of the following is the same as 50+12?


1. 10(5+3)
2. (60 5)+(100 2)

4.

If x*y = xy-

3. 25 12x2

4. 50 4 3

3. -16

4. -17

3. 4

4. 5

6.

If the 23x5x = 5x103, then the value of x is ________


1. 4
2. 3
3. 2

4. 1

7.

If 5x =

If 4 x
1. 2

8.

2. 3

1
, then the value of x is ________
25

1
2

If 7

2. -2

5
1
xy = 17, then the value of (x, y) is ________
4
x

1. (6, 2)
9.

3
4

1.
10.

2. (7, 2)

4 2 3
=?
3 2 3
9
4

2.

1
of 111
37

an
k
2.

.B

19
39

23
47

3. 9

1
3
but less than ?
2
2
29
3.
57

w
w

2. 0.1

15.

4. (8, 2)

4.

7
36

4. 12

4.

33
49

3. 100

4. 10

The number 1027-1 is not divisible by ____


1. 9
2. 90

3. 11

4. 10

1. 0

3. 2.3

4. None of these

w
14.

11
18

4. -4

(0.01)2 (0.11)2 (0.014)2


=?
(0.004)2 (0.011)2 (0.0014)2

1. 0.01

13.

2. 3

3. (9, 2)

3.

Which of the following fraction is greater than


1.

12.

3
2

1
of ? = 0
3

1. 1
11.

3. 2

am
sT

1.

32 , then the value of x is _______

Ex

5.

2. 17

od
ay
.

1. 16

co
m

1
x
then the value of 6* is _______
3
y

2.33 0.027
=
(2.3)2 0.69 0.090

2. 2.6

If 102y = 25 then what is the value of 10y ?


1. -5

2. 5

Get all banking study material on BankExamsToday.com

3.

1
25

4.

1
25

Page 7

www.BankExamsToday.com,continuous job updates for every hour


CRT

17.

b
2a
= 0.25 then what should the value of
a
2a
4
1. 1
2.
9
0.1
0.01
Which number is equal to
0.01
0.1

If

1. 1.01

b
b

2
?
9

2. 1.1

3.

5
9

4. 2

3. 10.1

4. 10.01

co
m

16.

What is the value of [0.3+0.3-0.3-0.3 x (0.3 x 0.30)]


1. 0.09
2. 0.27
3. 0.60

4. None of these

19.

Find the number which is equal to (50)3 + (-30)3 + (-20)3


1. 3x50x30x(-20)
2. 30x50x3x20
3. 3x50x(-30)

4. 3x(-30)x(-20

od
ay
.

18.

Find the value of the following:


20.

111111x11 = _______
1. 122221

21.

5776800x11 = ___________
1. 65344800
2. 63544800

22.

12369x11 = ________
1. 135069

2. 136059

23.

15.60x0.30 = ?
1. 4.68

2. 0.458

26.

3. 62544800

4. 63545800

3. 136069

4. 135059

3. 0.468

4. 0.0468

am
sT

4. 12222221

Ex

2.

3. 222221

63
5

3.

18
7

If 2276 155 = 79.2, the value of 122.76 15.5 is equal to


1. 7.092
2. 7.92
3. 79.02

4. None of these

4. 79.2

17.28 ?
= 200
3.6 0.2

w
w

1. 120

12 0.09 of 0.3x2 = ?
1. 0.80

2. 1.20

3. 12

4. 0.12

2. 8.0

3. 80

4. None of these

2. 18

3. 2

4. None of these

27.

?
x7 =?
0.01

an
k

25.

3420
19
35
1.
9

.B

24.

2. 1222221

28.

20

8 0.5
12
20 ?

1. 8
29.

If 5 = 2.24, the value of


1. 0.168

3 5
2 5

0.48

will be __________

2. 1.68

Get all banking study material on BankExamsToday.com

3. 16.8

4. 168

Page 8

www.BankExamsToday.com,continuous job updates for every hour


CRT

2.

17
110

31.

5555+6666-9999-1111 = ?
1. 1001
2. 1011

32.

Which is greater among


1.

5
9

3.

0.17
11

3. 1111

5 15 7
8
,
and ,
,
9 19 8
9
15
2.
19

3.

2. 818

3. 988

34.

132-123 = ?
1. 369

2. 396

3. 496

35.

36.

1. 5

11
28

1
7

13

2
7

1
=?
4

2. 5

13
28

4.

am
sT

(17)2+(23)2 = ?
1. 718

3. 6

17
11

4. 1221

7
8

33.

1
2

4.

co
m

0.289
=?
0.00121
170
1.
11

8
9

od
ay
.

30.

11
28

4. 8283

4. 469

4. 6

15
28

What approximate value should come in place of the question mark (?) in the following equation?
66

2
% of ? = 32.78x18.44
6

2. 880

3. 920

Ex

1. 900

4. 940

What should come in place of the question mark (?) in the following equation?
85.147+34.912x6.2+? = 802.293
1. 8230
2. 8500
3. 8410
4. 8600

38.

What should come in place of question mark (?) in the following equation?
5679+1438-2015 = ?
1. 5192
2. 5012
3. 5102

.B

4. 5002

Four of the five parts numbered (1), (2), (3), (4) and (5) are exactly equal. Which of the parts is not equal to the other
four? The number of that part is the answer.

w
w

39.

an
k

37.

1. 40% of 160+

1
of 240 2. 120% of 1200
3

3. 38x12-39x8

4. 6

1
of 140-2.5x306.4
2

40.

In the following equation what value would come in place of question mark(?)?
5798-? = 7385-4632
1. 3225
2. 2595
3. 2775
4. 3045

41.

What should come in place of question mark (?) in the following equation?
197x?+162 = 2620
1. 22
2. 12
3. 14

Get all banking study material on BankExamsToday.com

4. 16

Page 9

www.BankExamsToday.com,continuous job updates for every hour


CRT
42.

Which of the following numbers are completely divisible by seven?


A. 195195
B. 181181
C. 120120
1. only A & B
2. only B & C
3. only D & B

43.

what should come in the place of the question mark (?) in the following equation
9
20
77
1.
125

10
=?
17

2. 11

9
10

3.

119
450

4. 1

What should come in the place of the question mark (?) in the following equation?
28
?

?
112

1. 70

2. 56

3. 48

29
90

4. 64

od
ay
.

44.

5
12

co
m

21
25

D. 891891
4. All are divisible

What should come in the place of the question mark (?) in the following equation?
48 ? +32 ? = 320
1. 16
2. 2
3. 4
4. 32

46.

What should come in the place of the question mark (?) in the following equation?
(7 ?)2
49

81

1. 9

2. 2

1352

1. 81

50.

2. 1

3. 243

4. 9

4. 7

Solve: 4- 6
1. 5

4. 8

12

an
k

Simplify: 18 10 4 +32 (4+10 2-1) = ___________


1. 5
2. 9
3. 8
5

4 3 .

2. 4

.B

49.

4. 4

What should come in the place of the question mark (?) in the following equation?
452 272

48.

3. 3

Ex

47.

am
sT

45.

If x=4, y=3, then the value of x


1
2. 1
2

2 is _________
4
3.
5

4.

5
4

w
w

7
1.
4

3. 6

Get all banking study material on BankExamsToday.com

Page 10

www.BankExamsToday.com,continuous job updates for every hour


CRT
Answers
46) 3
47) 1
48) 4
49) 3
50) 3

co
m

31) 3
32) 4
33) 2
34) 4
35) 3
36) 1
37) 2
38) 3
39) 2
40) 4
41) 2
42) 4
43) 4
44) 2
45) 1

od
ay
.

16) 1
17) 3
18) 2
19) 2
20) 2
21) 2
22) 2
23) 1
24) 4
25) 2
26) 4
27) 4
28) 2
29) 2
30) 1

w
w

.B

an
k

Ex

am
sT

1) 4
2) 2
3) 2
4) 3
5) 2
6) 1
7) 2
8) 3
9) 2
10) 2
11) 4
12) 1
13) 3
14) 4
15) 2

Get all banking study material on BankExamsToday.com

Page 11

www.BankExamsToday.com,continuous job updates for every hour


CRT
NUMBER SYSTEM

Rational Numbers (Q): Numbers which are in the form of

co
m

Natural Numbers (N): Counting numbers 1, 2, 3, . . . . . are called Natural numbers. They are also called Positive
Integers.
N = {1, 2, 3, . . . . . . . .}
Whole Numbers (W): All the natural numbers including 0 together constitute the set of Whole numbers.
W = {0, 1, 2, 3, . . . . . . . .}
Integers (I or Z): All the whole numbers including negative counting numbers together constitute the set of
Integers.
I or Z = {. . . . . . ., -3,-2,-1, 0, 1, 2, 3 . . . . . . . .}
p
, where p, q are integers and q 0, are called
q

Q={

p
, (q
q

0)/ p, q

od
ay
.

Rational numbers.
Z}

E.g. -3, 1, 3.2,

1 22
,
, etc.
3 7

am
sT

Note:
1. Rational numbers are divided into two groups, namely integers and non -integers.
2. Non-integer belonging to the set of rational numbers is called fraction.
Fraction: A number expressed in the form of

p
is also called fraction, where p is the numerator and q is the
q

denominator. Fraction is a part of an integer.


E.g.

6 2
,
,
5 7

1
, etc.
6

E.g.

1 3 7
,
,
, etc.
5 7 9

Ex

Proper Fraction: Fractions in which Numerator < Denominator are called Proper Fractions.

.B

an
k

Improper Fraction: Fractions in which Numerator > Denominator are called Improper Fractions.
E.g. 8/3, 7/5, 9/4, etc.
Mixed Fraction: It has two parts. One is integer and the other is a fraction.
E.g. 1 1/3, 2 3/5, 5 4/3, etc.
Note:
1. All the mixed fractions can be converted into improper fractions.
2. A rational number can be expressed in the decimal form.
3. The decimal form of a rational number is either recurring or a terminating decimal.

w
w

E.g. 10/3 = 3.3333 (recurring)


3/4 = 0.75 (terminating)

Irrational Numbers (Q): A number which cannot be expressed in the form of rational number is called an
Irrational number.
For an irrational number, the decimal part is non -recurring and non-terminating.
E.g. 2 = 1.414. It is non-recurring and non-terminating.
Even number: An integer divisible by 2 is called an Even number.
E.g. 2, 4, 6, 8,..
Odd Number: An integer not divisible by 2 is called an Odd number.
E.g. 1, 3, 5, 7,.
Prime Numbers: Numbers which are not divisible by any number other then 1 and itself are called Prime numbers.
E.g. 2, 3, 5, 7,.
Composite Numbers: Except 1, the numbers which are not prime are called Composite numbers
E.g. 4, 6, 9, 12,
Co-prime Numbers: Numbers which do not have any common factor other than 1 are called Co -prime numbers.
Get all banking study material on BankExamsToday.com

Page 12

www.BankExamsToday.com,continuous job updates for every hour


CRT
E.g. (4, 15), (9, 22), (12, 29),

Value
Value
Value
Value
Value

of
of
of
of
of

4
5
6
2
1

=
=
=
=
=

4
5
6
2
1

ones = 4, Face Value of 4 = 4


tens = 50, Face Value of 5 = 5
hundreds = 600, Face Value of 6 = 6
thousands = 2000, Face Value of 2 = 2
ten thousands = 10,000, Face Value of 1 = 1

od
ay
.

Place
Place
Place
Place
Place

co
m

Note:
1. 1 is neither prime nor composite.
2. 2 is an even prime number.
3. Co-prime numbers can be prime or composite numbers.
4. Any two prime numbers are always Co-prime numbers.
5. Any two consecutive positive integers are always co -primes.
Place Value of a Digit in a Numeral: The value of where the digit is in the number, such as units, tens, hundreds,
etc.
Face Value: Face Value of a number is the number itself.
Consider the number 12654:

am
sT

Perfect Numbers: If the sum of the factors of a given number is twice the number, the number is said to be a
Perfect number.
E.g. Factors of 6 = 1, 2, 3, 6 and 1 + 2 + 3 + 6 = 12
28, 496, etc.are the other examples of perfect numbers.
MULTIPLICATION TIPS:

Ex

1. For multiplication of a given number by 9, 99, 999, etc., that is by 10 n 1, the easy way is: Put as many zeros to
the right of the multiplicant as there are nines in the mul tiplier and from the result subtract the multiplicant and
get the answer.
E.g. Multiply 2893 by 99.

an
k

Sol:2893 x 99 = 2893 (100 1) = 289300 2893 = 286407.


2. For multiplication of a given number by 11, 101, 1001, etc., that is by 10 n + 1, the easy way is: Place n zeros to
the right of the multiplicant and then add the multiplicant to the number so obtained.

.B

E.g. Multiply 3782 by 11.

w
w

Sol:3782 x 11 = 3782 (10 + 1) = 37820 + 3782 = 41602.


3. For multiplication of a given number by 15, 25, 35, etc. Double the multiplier and then multiply the multiplicant
by this new number and finally divide the product by 2.

E.g. Multiply 5054 x 15 = (5054 x 30) = (151620) = 75810

4. For multiplication of a given number by 5, 25, 125, 625, etc., that is, by a number which is some power of 5.
Place as many zeros to the right of the multiplicant as is the power of 5 in the multiplier, then divide the number
so obtained by 2 raised to the same power as is the power of 5.
E.g. 2982 x 5 = 29820/2 = 14910
5739 x 25 = 573900/2 2 = 143475

Get all banking study material on BankExamsToday.com

Page 13

www.BankExamsToday.com,continuous job updates for every hour


CRT
a) No. of factors of a given number: If N a p b q c r ..... then the number of factors of N = (p + 1) (q + 1) (r
+ 1)., where a, b, c are prime factors of N and p, q, r,. are positive integers.
E.g. Find the number of factors of 24.
3

Sol:24 = 2 3
The number of factors of 24 = (3 + 1) (1 + 1) = 8.

integers.

bq 1 1
b 1

cr 1 1
..........
.........where a, b, c are prime factors of N and p, q, r,. are positive
c 1

E.g. Find the sum of the factors of 24.


Sol:24 = 23 31
23 1 1
2 1

Sum of the factors of 24 =

31 1 1
3 1

60.

od
ay
.

ap 1 1
a 1

a p b q c r ..... then the sum of the factors of N =

co
m

b) Sum of the factors of a given number: If

c) No. of ways of expressing a given number as a product of two factors:

a p b q c r ..... where a, b, c are prime factors of N and p, q, r,. are positive integers then the number
1
(p
2

am
sT

If

of ways in which N can be expressed as product of two factors =

1)(q

1)(r

1)..... .

E.g. Find the number of ways of expressing 48 as a product of two factors.


Sol:48 =

2 4 31

No. of ways =

1
(p
2

1)(q

1)

1
(4
2

1)(1

1)

5.

bq

an
k

cr ..... where a, b, c are prime factors of N and p, q, r,. are positive integers then the number
1
(p 1)(q 1)(r 1)........ 1 .
of ways in which N can be expressed as product of two factors =
2

If N

ap

Ex

d) No. of ways of expressing a given number which is a perfect square as a product of two factors:

E.g. Find the no. of ways of expressing 36 as a product of two factors.

2 2 32

No. of ways =

1
(p
2

1)(q 1) 1

.B

Sol:36 =

1
(2 1)(2 1) 1
2

5.

w
w

TIPS ON SQUARES:

Condition
To square any
number ending
with 5.
To square a
number in which
every digit is one.

To square a
number which is
nearer to 10 x.

Method

Example
(35) 2 = {3(3+1)}25 =
1225

(a5) 2 = {a(a+1)}25

Count the number of digits in the given number and start writing
numbers in ascending order from one to this number and then in
descending order up to one.
Use the formula:
x2

(x 2

y2 )

y2

Get all banking study material on BankExamsToday.com

(x

y)(x

y)

y2

(11) 2 = 121,
(111) 2 =12321
(1004) 2 =
(1004 4)
(1004 + 4) + (4) 2 =
1000(1008) + 16 =
1008016

Page 14

www.BankExamsToday.com,continuous job updates for every hour


CRT
DIVISION:
Dividend = (Divisor x Quotient) + Remainder

co
m

E.g.

Divisibility
by

Rule

Units digit of the number should be zero or


divisible by 2.

Sum of the digits in the number should be


divisible by 3.

Number formed by the last two digits should


be divisible by 4 or are both zero.

Example

Should satisfy divisibility rules of 2 and 3.

.B

Number formed by the last three digits should


be divisible by 8. or zeros

w
w

an
k

The unit digit of the given number is doubled


and then it is subtracted from the number
obtained after omitting the unit digit. If the
result is divisible by 7, then the given number
is also divisible by 7.
7

4, 2, 102,
etc.
1782

am
sT

Units digit of the number should be 0 or 5.

Ex

od
ay
.

TESTS OF DIVISIBILITY

4784,
300, etc.

448

1576
1395

10

Number should end in zero.

1000

11

Sum of digits at odd places Sum of digits at


even places should be 0 or divisible by 11.

12

Should satisfy divisibility rules of 3 and 4.

156

14

Should satisfy divisibility rules of 2 and 7.

322

4784 Since 84 is divisible by 4,


4784 is also divisible by 4.

4518

Get all banking study material on BankExamsToday.com

1+7+8+2 = 18 which is divisible by


3 hence 1782 also divisible by 3.

120, 625,
etc.

Sum of the digits in the number should be


divisible by 9.

Last two digits in the number should be 0 or


divisible by 25.

Explanation

38797

448 44 8(2) = 44 16 = 28
which is divisible by 7 and hence
448 is also divisible by 7.
1576 576 is divisible by 8 and
hence 1576 is also divisible by 8.
1395 1+3+9+5 = 18 is divisible
by 9 and hence 1395 is also
divisible by 9.
38797 Sum of digits at odd
places = 3+7+7 = 17 Sum of digits
at odd places = 8+9 =17 and 17
17 =0, hence 38797 is divisible by
11.
156 is divisible by 2 and 3 hence
156 is also divisible by 12.
322 is divisible by 2 and 7 hence
322 is also divisible by 14.
175 75 is divisible by 25 and
hence 175 is also divisible by 25.
Page 15

www.BankExamsToday.com,continuous job updates for every hour


CRT
25

175
Last three digits in the number should be 0 or
divisible by 125.

125

2250

2250 250 is divisible by 125 and


hence 2250 is also divisible by 125.

Steps to find whether a given number is prime number or not:

co
m

1. Find the least positive integer, a such that a 2 > given number.
2. Test the divisibility of given number by every prime number that is less than a.
3. The given number is a prime number only if it is not divisible by any of these primes.

1.
2.
3.
a)

od
ay
.

E.g. Check whether 923 is a prime number or not?

923 lies between 900 and 961 which are perfect squares having square roots 30 and 31 respectively.
Prime numbers less than 31 are 2,3,5,7,11,13,17,19,23,29.
923 are not divisible by any of these numbers and hence it is a pri me number.
To find the number in the unit place for odd numbers: When there is an odd digit in the unit place except 5,
multiply the number by itself until you gets 1 in the unit place.

(... 1) n = ( 1)

am
sT

(... 3) 4 n = ( 1)
(... 7) 4 n = ( 1)

(... 9) 2 n = ( 1) where n = 1, 2, 3, . . .

(... 2) 4 n = ( 6)
(... 4) 2 n = ( 6)

an
k

(... 6) n = ( 6)

Ex

b) To find the number in the units place for even numbers: When there is an even digit in the unit place, multiply
the number by itself until you gets 6 in the unit place.

(... 8) 4 n = ( 6) where n = 1, 2, 3, . . .

.B

c) If there is 1, 5 or 6 in the units place of the given number: If there is 1, 5 or 6 in the unit place of the given
number, then after any times of its multiplication, it will have the same digit in the unit place.

w
w

(... 1) n = ( 1)
(... 5) n = ( 5)

(... 6) n = ( 6) where n = 1, 2, 3, . . .

Solved Examples
1.

On dividing 64652 by a certain number, the quotient is 101 and the remainder is 12. Find the divisor.
Sol: Here, the required number is divisor.
Divisor =
=

Dividend- Rem ainder


Q uotient

64652 12
101

64640
101

640

Get all banking study material on BankExamsToday.com

Page 16

www.BankExamsToday.com,continuous job updates for every hour


CRT
2.

A number when divided by 160 leaves a remainder 52 and the quotient is 15. Find the number.
Sol: Here, the required number is dividend.
Dividend = (Divisor x Quotient) + Remainder
= (160 x 15) + 52
= 2452
Find the least number of 5 digits which is exactly divisible by 642.
Sol: The least number of 5 digits is 10,000.
Dividing this number by 642, the remainder is 370.
So, the required number is 10,000 + (642 - 370) = 10272.
Find the greatest number of 5 digits which is exactly divisible by 642.
Sol: The greatest number of 5 digits is 99,999.
Dividing this number by 642, the remainder is 489.
So, the required number is 99,999 - 489 = 99510.

5.

od
ay
.

4.

co
m

3.

Find the number nearest to 14800 which is exactly divisible by 245.

6.

Find whether 577 is a prime number.

am
sT

Sol: The remainder on dividing 14800 by 245 is 100.


So, the number required number = 14800 100 = 14700 which is exactly divisible by 245.

Ex

Sol: (24) 2 = 576 < 577 and (25) 2 = 625 > 577
n = 25
Prime numbers less than 25 are 2, 3, 5, 7, 11, 13, 17, 19 and 23.
Since, 577 is not divisible by any of these numbers, it is a prime number.
How many numbers up to 531 are divisible by 15?

an
k

7.

.B

Sol: Divide 531 by 15.


531 = 35 x 15 + 6
The quotient is the required number and here it is 35.
So, there are 35 numbers up to 531 are divisible by 15.
How many numbers up to 200 are divisible by 5 and 7 together?

w
w

8.

Sol: L.C.M. of 5 and 7 = 35.


Divide 200 by 35.
200 = 5 x 35 + 20
The quotient is the required number and here it is 5.
So, there are 5 numbers up to 200 are divisible by 35.
Find the number in the unit place in (729)59 .

9.

Sol: (729)59
10.

(729)58 729

(...1) 729

9 in the unit place.

Find the number in the unit place in (98)42 .

Sol: (98)42

(98)4

10

(98)2

(...6) (...4)

4 in the unit place.

Get all banking study material on BankExamsToday.com

Page 17

www.BankExamsToday.com,continuous job updates for every hour


CRT
11.

Find the number in the unit place in (636)36 .

Sol: (636 )

36

(... 6) 36

6 in the unit place.

12. Convert 0.4444.. into a rational number.


Sol: Let x = 0.4444..(1)

9x

od
ay
.

10 x = 4.4444.
- x = 0.4444.
---------------------9 x = 4.0000.
---------------------4
.
9

101 .

13. Convert 5.626262.. into a rational number.


Sol: Let x = 5.626262.. (1)

102 .

Exercise

2. 1

The number divisible by 99 is_________


1. 3572404
2. 135792

3.

45
9

3. 913464

4.

101
9

4. 114345

The smallest number which when subtracted from 43079 makes it exactly divisible by 9 is
1. 4
2. 5
3. 6
4. 7

3.

(3x3)+6x7 14+7 3x3 = ________

.B

2.

O.1 + 8 9 x11
89
1.
9

w
w

1.

an
k

Ex

am
sT

Since 2 digits (62) is repeating multiply equation 1 on both sides by


100 x = 562.6262.(2)
Subtract Equation 1 from 2 on both sides
100 x = 562.6262.
- x = 5.6262..
--------------------------99 x = 557.0000.
--------------------------557
99x = 557
x=
99

co
m

Since 1 digit (4) is repeating multiply equation 1 on both si des by


10 x = 4.4444.(2)
Subtract Equation 1 from 2 on both sides

4.

The number 111, 111, 111, 111 is divisible by


1. 9, 11
2. 5, 11

5.

The largest natural number by which the product of three consecutive even natural numbers is always divisible is
1. 16
2. 24
3. 48
4. 96

6.

The number of divisors of 120 individuality is


1. 8
2. 12

Get all banking study material on BankExamsToday.com

3. 5, 9

3. 14

4. 3, 11

4. 16

Page 18

www.BankExamsToday.com,continuous job updates for every hour


CRT
7.

The greatest number which exactly divides 826 is


1. 2
2. 7

8.

The least number which is a perfect square as well as a factor of 936 is


1. 36
2. 9
3. 4

4. 16

9.

Which of the following is a perfect number?


1. 28
2. 30

3. 14

4. 16

10.

Which of the following numbers is divisible by 11?


1. 21434799
2, 74325566

3. 85437657

4. 93825677

11.

The value of 320.16x320.04 is __________


1. 2. 2
2

3. 2

4. 4. 2

12.

Find the sum of 1+2+6+12+..+90.


1. 550
2. 440

3. 330

13.

How many numbers are there between 200 and 300 in which 0 occurs only once?
1. 18
2. 885
3. 115
4. 1000
885 885 885 115 115 115
=
885 885 115 115 885 115

1. 770

2. 885

od
ay
.

co
m

4. 413

am
sT

14.

3. 59

3. 115

4. 388

4. 1000

The difference between the squares of two numbers is 256000 and the sum of the numbers is 1000. The numbers are
1. 628, 372
2. 600, 400
3. 640, 630
4. None

16.

A number is as much greater than 21 as is less than 71. The number is


1. 39
2. 46
3. 41

17.

The sum of three numbers is 102. If the ratio between first and second be 2:3 and that between second and third be
5:3, then the second number is _____
1. 30
2. 45
3. 27
4. 48

18.

The value of 1+

= _________

.B

w
w

1.

29
19

1
9

2.

10
19

The value of 9+9x9-9 9 is ___________


1. 17
2. 89

19.

4. 49

an
k

Ex

15.

3.

29
10

3. 9

4.

10
9

4. None

20.

On dividing a number by 999, the quotient is 377 and the remainder is 105. The number is
1. 476727
2. 376538
3. 376728
4. 359738

21.

0.77777 is equal to
1.

77
83

2.

99
77

Get all banking study material on BankExamsToday.com

3.

77
99

4.

77
109

Page 19

www.BankExamsToday.com,continuous job updates for every hour


CRT
22.

199+299+399+..9999 is divisible by
1. 99
2. 98
3. 100

23.

If the fraction
143
156

143 136 63
17
,
,
and
are arranged in ascending order, then which is the third number?
156 153 60
16
17
136
63
2.
3.
4.
16
153
60

co
m

1.

4. 101

24.

How many three-digit numbers, when divided by 15, leave the remainder 15?
1. 49
2. 50
3. 60

4. 62

25.

The value of (112+122+132++202) is


1. 3854
2. 2485

4. 2870

26.

A boy was asked to multiply a certain number by 25. He multiplied by 52 and got his answer more than the correct
one by 324. The number to be multiplied was
1. 12
2. 15
3. 25
4. 52

1.

3
2
5

5
................ 2
7

5
999

1001
999

2.

997
is equal to
999
1001
3

od
ay
.

1
2
3

am
sT

27.

3. 3485

3.

4.

3
1001

A number consists of two digits whose sum is 15. If 9 is added to the number, then the digits change their places. The
number is
1. 69
2. 78
3. 87
4. 96

29.

One of the two consecutive positive integers, the sum of whose squares is 761, is
1. 15
2. 24
3. 20
4. 25

30.

The denominator of a rational number is 3 more than its numerator. If the numerator is increased by 7 and the
denominator is decreased by 7, we obtain 2. The rational number is

31.

3n

3n

3n

3n

3.

7
10

4.

8
11

3.

2
3

4.

3
4

is

2.

w
w

1
2

15
18

2.

Find the value of


1.

4
3

A number N when divided by 120 gives a remainder 76. What is the remainder obtained when the same number is
divided by 8?
1. 2
2. 3
3. 4
4. 5

32.

1
4

.B

1.

an
k

Ex

28.

33.

What is the value of 1+

1
1

1
7
13
2.
19
1

1.
34.

19
13

3.

18
13

The sum of two consecutive prime numbers is 30. The two numbers are
1. 18, 12
2. 7, 23
3. 11, 19

Get all banking study material on BankExamsToday.com

4.

7
6

4. 13, 17
Page 20

www.BankExamsToday.com,continuous job updates for every hour


CRT
35.

By how much is 11% of 22.2 less than 10% of 24.4?


1. 0.2
2. 0.02
3. 0.002

36.

The sum of two numbers is 29 and the difference of their squares is 145. The difference between the number is
1. 13
2. 5
3. 8
4. 11

37.

The ratio between a two digit number and the sum of the digits of that number is 4:1. If the digit in the units place is
3 more than the digit in the tenth place, what is the number?
1. 24
2. 63
3. 36
4. Cant be determined

38.

The difference between a two digit number and the number obtained by interchanging the digits is 27. What is the
sum of the two digits of the number?
1. 3
2. 6
3. 9
4. Cant be determined

39.

0.02% of a number is 20. What will be 20% of the number?


1. 200
2. 4000
3. 20,000

40.

A number whose fifth part increased by 5 is equal to its fourth part diminished by 5 is
1. 160
2. 200
3. 180
4. 220

41.

What will be the remainder when 2975 is divided by 30?


1. 28
2. 27
3. 1

42.

If a number is divided by 527, the remainder 42. What will be the remainder if it is divided by 17?
1. 8
2. 7
3. 16
4. 15

43.

Two different numbers when divided by 47, leave remainders 13 and 23 respectively. If their sum is divided by the
same number 47, what will be the remainder?
1. 35
2. 36
3. 20
4. 25

44.

The units digit in the product (2767)135x(576)298 is


1. 2
2. 6

3. 8

4. 3

45.

Find the number of divisors of 240?


1. 20
2. 10

3. 24

4. 18

46.

Find the sum of the divisors of 360?


1. 1107
2. 1170

3. 1175

4. 1180

47.

What is the total number of possible values of p so that the number 3p8p4709 is divisible by 9?
1. 0
2. 1
3. 2
4. 3

co
m

od
ay
.

am
sT

Ex

an
k

.B

What fraction of a week is a second?


1.

50.

4. 29

1
1
1
of a distance by car,
of the remaining by bus and
of the still remaining by train and the
3
3
3

remaining 72 km by scooter. Find the distance he traveled?


1. 216
2. 243
3. 336

49.

4. 2000

w
w
A man travels

48.

4. 0.0002

1
60480

2.

1
86400

3.

1
604800

4. 576

4. 604800

4
of a number exceeds its 2/3 by 8. The number is
5

1. 30

2. 60

Get all banking study material on BankExamsToday.com

3. 75

4. 90

Page 21

www.BankExamsToday.com,continuous job updates for every hour


CRT
The expression 1/1.2 + 1/2.3 + 1/3.4 + . + 1/n(n+1) for any natural number n, is _____
1. Always greater than 1 2. Always less than 1
3. Always equal to 1
4. Not definite

52.

A two digit number becomes five-sixth of itself when its digits are reversed. The difference of the two digits is 1. The
number is ______
1. 45
2. 54
3. 56
4. 65

53.

The sum of the squares of two numbers is 3341 and the difference of their squares is 891. The numbers are _______
1. 25, 46
2. 35, 46
3. 25, 36
4. None of these

54.

Of the three numbers, the sum of the first two is 45, the sum of the second and third is 55 and the sum of the third and
thrice the first is 90. The third number is _______
1. 30
2. 27
3. 39
4. 52

55.

Three numbers are in the ratio 3 : 4 : 5. The sum of the largest and the smallest equals the sum of the third and 52.
The smallest number is _______
1. 20
2. 27
3. 39
4. 52

56.

Of the three numbers, the first is twice the second and is half of the third. If the average of these numbers be 56, the
numbers in order are _________
1. 48, 96, 24
2. 48, 24, 96
3. 96, 24, 48
4. 96, 48, 24

57.

The number of prime factors in the expression (6)10 x (7)17 x (11)27.


1. 54
2. 64
3. 71

4. 81

58.

The number of prime factors in 2222 x 3333 x 5555 is _______


1. 3
2. 1107
3. 1110

4. 1272

59.

The total number of prime factors of the product (8) 20 (15)24 (7)15 is _______
1. 59
2. 98
3. 123

60.

24 is divided into two parts such that 7 times the first part added to 5 times the second part makes 146. The first part
is _________
1. 11
2. 13
3. 16
4. 17

61.

The sum of squares of the numbers is 80 and the square of their difference is 36. The product of the two numbers is
______
1. 22
2. 44
3. 58
4. 116

od
ay
.

am
sT

Ex

an
k

.B

The product of two numbers is 120. The sum of their squares is 289. The sum of the two numbers is ________
1. 20
2. 23
3. 169
4. None of these
A fraction becomes 4 when 1 is added to both the numerator and denominator and it becomes 7 when 1 is subtracted
from both the numerator and denominator. The numerator of the given fraction is ________
1. 2
2. 3
3. 7
4. 15

63.

4. 138

w
w

62.

co
m

51.

64.

Three numbers are in the ratio 5 : 4 : 3. The sum of the largest and the smallest equals the sum of the third and 72.
The largest number is ________
1. 60
2. 27
3. 90
4. 65

65.

The sum of three numbers is 132. If the first number be twice the second and third number be one-third of the first,
then the second number is _______
1. 32
2. 36
3. 48
4. 60

Get all banking study material on BankExamsToday.com

Page 22

www.BankExamsToday.com,continuous job updates for every hour


CRT
66.

If the unit digit in the product 75 ? x 49 x 867 x 943 be 1, then the value of ? is ______
1. 1
2. 3
3. 7
4. 9

67.

What is the value of the digit in the place of $ in 3875 $ 622, if the number is divisible by 9?
1. 4
2. 3
3. 2
4. 1

68.

What is the least value of A + B, if 407A234B is exactly divisible by 9?


1. 3
2. 4
3. 5

69.

If a number is subtracted from the square of its one half, the result is 48. The square root of the number is
__________
1. 4
2. 5
3. 6
4. 8

70.

Two numbers are such that the ratio between them is 3 : 5, but if each is increased by 10, the ratio between them
becomes 5 : 7. The numbers are _________
1. 3, 5
2. 7, 9
3. 13, 22
4. 15, 25

Answers

46) 2
47) 2
48) 2
49) 3
50) 2
51) 2
52) 1
53) 2
54) 1
55) 3
56) 2
57) 2
58) 3
59) 3
60) 2

am
sT

31) 3
32) 3
33) 1
34) 4
35) 2
36) 2
37) 3
38) 4
39) 3
40) 2
41) 4
42) 1
43) 2
44) 3
45) 1

Ex

an
k

16) 2
17) 2
18) 1
19) 2
20) 3
21) 3
22) 3
23) 1
24) 3
25) 2
26) 1
27) 3
28) 2
29) 3
30) 2

61) 1
62) 2
63) 4
64) 3
65) 2
66) 4
67) 2
68) 4
69) 1
70) 4

w
w

.B

1) 4
2) 4
3) 2
4) 4
5) 3
6) 4
7) 3
8) 3
9) 1
10) 3
11) 3
12) 3
13) 1
14) 4
15) 1

od
ay
.

co
m

4. 7

Get all banking study material on BankExamsToday.com

Page 23

www.BankExamsToday.com,continuous job updates for every hour


CRT
L.C.M. AND H.C.F.
Common Multiple: A common multiple of two or more numbers is a number which is exactly divisible by each one of
them.
E.g. 32 is a common factor of 8 and 16

co
m

Least Common Multiple (L.C.M): The least multiple among all the common multiples of given numbers is called
Least Common Multiple.
Methods of finding L.C.M.
Method
a.
b.
c.

of Prime Factors
Resolve each given number into prime factors.
Take out all the factors with highest powers that occur in given numbers.
Find the product of these factors. This product will be L.C.M.

od
ay
.

1.

E.g. Find the L.C.M. of 12, 14 and 20.

am
sT

12 = 2 2 x 3
14 = 2 x 7
20 = 2 2 x 5

So, the L.C.M. = 2 2 x 3 x 5 x 7 = 420


2.

Method of Division

an
k

Ex

E.g. Find the L.C.M. of 12, 15, 18 and 20.

.B

So, the L.C.M. = 2 x 2 x 3 x 5 x 3 = 180

w
w

Common Factor: A common factor of two or more numbers is a number which divides each of
them exactly.
E.g. 4 is a common factor of 8 and 12

Highest Common Factor (H.C.F): Highest common factor of two or more numbers is the
greatest number that divides each one of them exactly. It is also called Greatest Common Divisor
or Greatest Common Measure.
Methods of finding H.C.F.
1.

Method of Prime Factors


E.g. Find the H.C.F. of 50 and 70
Sol: 50 = 2 x 5 x 5
70 = 2 x 5 x 7
Common factors are 2 and 5.
So, H.C.F. = 2 x 5 = 10

Get all banking study material on BankExamsToday.com

Page 24

www.BankExamsToday.com,continuous job updates for every hour


CRT
2.

Method of Division
E.g. 1. Find the H.C.F. of 3332, 3724.

So, the H.C.F. of 3332, 3724 is 196.


E.g. 2. Find the H.C.F. of 10, 15 and 23.

am
sT

Step 1: First find the H.C.F. of 10 and 15. It is 5


Step 2: Then find the H.C.F. of this 5 and 23. It is 1.
So, the H.C.F. of 10, 15 and 23 is 1.
Note:

od
ay
.

co
m

Sol:

L.C.M. and H.C.F. of fractions

2.

Product of two numbers = L.C.M. of two numbers x H.C.F. of two numbers.

3.

To find the greatest number that will exactly divide a, b and c, simply find the H.C.F. of a, b and c.

4.

To find the greatest number that will divide a, b and c leaving remainders x, y and z respectively, find the
H.C.F. of (a x), (b y) and (c z).

5.

To find the least number which is exactly divisible by a, b and c, simply find the L.C.M. of a, b and c.

an
k

.B

To find the least number when divided by a, b and c leaving remainders x, y and z respectively, find the
(L.C.M. of a, b and c) k, where k = (a x) = (b y) = (c z).
To find the least number which when divided by a, b and c leaves the same r emainder r in each case, find
(L.C.M. of a, b and c) + r.
Two numbers when divided by a certain divisor give remainders r1 and r2 . When their sum is divided by

7.

w
w

6.

Ex

1.

8.

the same divisor, the remainder is


9.

r3 . Then the divisor is given by

r1

r2

r3 .

A number on being divided by d1 and d2 successively leaves the remainders r1 and r2 , respectively. If the
number is divided by d1 x d2 , then the remainder = ( d1 x r2 + r1 ).

10.

To find the greatest number that will divide x, y and z leaving the same remainder r in each case:

Get all banking study material on BankExamsToday.com

Page 25

Case 1:

When the value of remainder r is given


Required number = H.C.F. of (x r), (y r) and (z r).

Case 2:

When the value of remainder is not given


Required number = H.C.F. of |(x y)|, |(y z)| and |(z x)|.

b.

Leaves remainder k in each case:


Required number = (n-digit greatest number R) + k.

od
ay
.

11. To find the n-digit greatest number which when divided by x, y and z:
a. Leaves no remainder i.e. exactly divisible
Step 1: Find the L.C.M. of x, y and z. Let it be L.
Step 2: Divide the n -digit greatest number by this L. Let the remainder be R.
Step 3: Required number = (n-digit greatest number R).

co
m

www.BankExamsToday.com,continuous job updates for every hour


CRT

12. To find the n-digit smallest number which when divided by x, y and z:

am
sT

a) Leaves no remainder i.e. exactly divisible


Step 1: Find the L.C.M. of x, y and z. Let it be L.
Step 2: Divide the n -digit smallest number by this L. Le the remainder be R.
Step 3: Required number = n-digit smallest number + (L R).
b) Leaves remainder k in each case:
Required number = n-digit smallest number + (L R) + k.

Solved Examples

Find the greatest number that will exactly divide 200 and 310.

Ex

1.

Sol: The required number = H.C.F. of 200 and 310 = 10.


Find the greatest number that will divide 148, 246 and 623 leaving remainders 4, 6 and 11 respectively.

an
k

2.

Sol: The required number = H.C.F. of (148 4), (246 6), and (623 11)
= H.C.F. of 144, 240, 612 = 12.
Find the smallest number that is exactly divisible by 45, 63 and 50.

.B

3.

4.

w
w

Sol: Required number = L.C.M. of 45, 63 and 50 = 3150.


Find the least number which when divided by 36, 48 and 64 leaves the remainders 25, 37 and 53 respectively.

Sol: (36 25) = (48 37) = (64 53. = 11


Required number = (L.C.M. of 36, 48 and 64) 11
= 576 11 = 565

5.

Find the least number which when divided by 12, 16 and 18, will leave the remainders 7 in each case.
Sol: Required number = (L.C.M. of 12, 16 and 18) + 7
= 144 + 7 = 151

6.

Find the greatest number which will divide 772 and 2778 so as to leave the remainder 5 i n each case.
Sol: Required number = H.C.F. of (772 5) and (2778 5)
= H.C.F. of 767 and 2773 = 59.
Get all banking study material on BankExamsToday.com

Page 26

www.BankExamsToday.com, best study material for bank exams


CRT
7.

Find the greatest number which on dividing 152, 277 and 427 leaves equal remainder.
Sol: Required number =
H.C.F. of |(152 277)|, |(277 - 427)|, |(427 152)|
= H.C.F. of 125, 275 and 150 = 25.
Find the greatest number of 4 digits which, when divided by 12, 18, 21 and 28 leaves 4 as a remainder in each
case.

co
m

8.

Sol: L.C.M. of 12, 18, 21 and 28 = 252.


Greatest 4-digit number = 9999.
The remainder when 9999 is divided by 252 = 171
So, the required number = (9999 171) + 4 = 9832.

Find the greatest number of 4 digits which, when divided by 12, 15, 20 and 35 leaves no remainder.
Sol: L.C.M. of 12, 15, 20 and 35 = 420.
The remainder when 9999 is divided by 420 = 339
So, the required number = (9999 339) = 9660.

od
ay
.

9.

am
sT

10. Find the least number of 4 digits which is divisible by 2, 4, 6 and 8.


Sol: L.C.M. of 2, 4, 6 and 8 is 24.
The least number of 4 digits = 1000
The remainder when 1000 divided by 24 = 16.
So, the required number = 1000 + (24 16) = 1008.

Ex

11. Find the smallest number of 4 digits when divided by 12, 18, 21 and 28 leaves remainder 5 in each case.

an
k

Sol: L.C.M. of 12, 18, 21 and 28 = 252


The least number of 4 digits = 1000
The remainder when 1000 divided by 252 = 244.
So, the required number = 1000 + (252 244) + 5 = 1013.

.B

12. Two numbers when divided by a certain divisor give remainders 16 and 12 respectively. When their sum is
divided by the same divisor, the remainder is 4. Find the divisor.
Sol: Required divisor = 16 +12 4 = 24.

w
w

13. A number on being divided by 10 and 11 successively leaves the remainders 5 and 7, respectively. Find the
remainder when the same number is divided by 110.

Sol: Required remainder = 10 x 7 + 5 = 75.


14. Find the least number which when divided by 8, 10 and 15 leaves the remainders 3, 5 and 10, respectively.
Sol: Here, 8 3 = 10 5 = 15 10 = 5
L.C.M. of (8, 10, 15) = 120
The required least number = 120 5 = 115.

Get all banking study material on BankExamsToday.com

Page 27

www.BankExamsToday.com, best study material for bank exams


CRT
Exercise
1.

Find the L.C.M. of 12, 25, 30 and 20.


1. 240
2. 300

2.

Find the L.C.M. of 2 x 3 x 52 x 7, 23 x 32 x 5 and 22 x 33 x 52.


1. 23 x 53 x 32
2. 23 x 32 x 5 x 7
3. 23 x 32 x 5 x 7

3.

Find the L.C.M of


3
5

4 3 2
8
,
,
and
.
5 10 15
15
4
2. 3
5

3. 4

4. 150

4
5

4. 3

Find the L.C.M. of 0.01, 0.1, 0.001 and 0.0001.


1. 0.1
2. 0.01

3. 0.001

5.

Find the H.C.F. of 300, 450 and 525.


1. 125
2. 50

3. 75

6.

Find the H.C.F. of 32 x 52 x 7 , 3 x 52 x 7, 33 x 72 x 52.


1. 3 x 5 x 72
2. 3 x 5 x 7
3. 3 x 52 x 7

7.

Find the G.C.D. of


3
10

am
sT

1.

3.

3
5

od
ay
.

4.

3 9
6
12
,
,
and
.
5 20 25
25
3
2.
100

4. 23 x 33 x 52 x 7

co
m

1. 4

3. 320

3
1000

4. 1

4. 25

4. 32 x 52 x 72

4. 3

Find the G.C.D. of 0.01, 0.1, 0.001, 0.0001.


1. 0.1
2. 0.01

3. 0.001

4. 0.0001

9.

Find the G.C.D. of 0.2, 2.0, 0.02, 20.0.


1. 0.2
2. 0.02

3. 0.002

4. 2

10.

L.C.M. of two numbers is 192 and their H.C.F. is 16. If one of them is 48 then find the other.
1. 32
2. 64
3. 48
4. 68

11.

The product of two numbers is 1575 and their L.C.M. is 315. Find the numbers.
1. 30 and 45
2. 35 and 45
3. 45 and 65
4. 45 and 75

12.

The L.C.M. of two numbers is 12 times their H.C.F. The sum of L.C.M. and H.C.F. is 65. If one of them is 20. Find
the other.
1. 15
2. 18
3. 25
4. 16

an
k

.B

w
w

The L.C.M. of two numbers is 4 times their H.C.F. The product of numbers is 900. Find their L.C.M.
1. 64
2. 48
3. 45
4. 60

13.

Ex

8.

14.

Two numbers are in the ratio 5 : 8 and their H.C.F. is 4. Find the numbers.
1. 25 and 40
2. 20 and 32
3. 30 and 48

15.

Three numbers are in the ratio 3 : 6 : 8 and their L.C.M. is 120. Find the smaller number.
1. 16
2. 18
3. 15
4. 12

16.

Find the greatest number which will divide 1026, 1215 and 2349 exactly.
1. 24
2. 27
3. 29

Get all banking study material on BankExamsToday.com

4. 15 and 24

4. 37

Page 28

www.BankExamsToday.com, best study material for bank exams


CRT
Find the greatest possible length which can be used to measure the length 6 m 76 cm, 4 m 81 cm and 7 m 67 cm
exactly.
1. 13 cm
2. 15 cm
3. 23 cm
4. 17 cm

18.

Find the greatest number that will divide 1657 and 2037 to leave remainders 6 and 5 respectively.
1. 121
2. 123
3. 129
4. 127

19.

Find the greatest number that will divide 520, 1140 and 1220 leaves the remainders 7, 6 and 5 respectively.
1. 29
2. 23
3. 27
4. 21

20.

Find the greatest number which divides 208, 1194 and 1449 leaving 4 as the remainder in each case.
1. 27
2. 19
3. 17
4. 13

21.

Find the greatest number which divides 284, 678 and 1618 leaves the same remainder in each case.
1. 2
2. 3
3. 4
4. 6

22.

Find the least number which is exactly divisible by 10, 12, 14, 16 and 18.
1. 5010
2. 5030
3. 5040

23.

Find the least number which when divided by 12, 16, 18 leaves 5 as remainder in each case.
1. 143
2. 149
3. 139
4. 144

24.

Find the smallest number which when decreased by 3 is exactly divisible by 9, 12, 15, 18 and 21.
1. 1253
2. 1263
3. 1267
4. 1243

25.

Which of the following when increased by 4 is exactly divisible by 12, 14, 16, 28 and 20?.
1. 5034
2. 5006
3. 5026
4. 5036

26.

Find the least number which when divided by 35, 45 and 55 leaves remainders 20, 30 and 40 respectively.
1. 3450
2. 3250
3. 3420
4. 3410

27.

Find the least number of five digits which is exactly divisible by 4, 6, 8, 12 and 16.
1. 10025
2. 10024
3. 10032
4. 10034

28.

Find the greatest number of five digits which is exactly divisible by 16, 24, 28 and 32.
1. 99456
2. 99556
3. 99446
4. 99566

29.

Find the greatest number less than 800 which is exactly divisible by 12, 15, 20 and 30.
1. 740
2. 720
3. 780
4. 760

od
ay
.

am
sT

Ex

an
k

.B

Find the greatest number between 600 and 700 which is exactly divisible by 20 and 30
1. 660
2. 640
3. 620
4. 480
Find the least number of four digits which when divided by 4, 5 and 6 leaving the remainder 2 in each case.
1. 1022
2. 1012
3. 1032
4. 1002

31.

4. 5020

w
w

30.

co
m

17.

32.

Find the greatest number of four digits which when divided by 10, 15 and 20 leaves remainder 3 in each case.
1. 9983
2. 9663
3. 9963
4. 9960

33.

Find the least number which when divided by 8, 12, 15 and 18 leaves a remainder 3 in each case and exactly divisible
by 11.
1. 360
2. 361
3. 363
4. 383

Get all banking study material on BankExamsToday.com

Page 29

www.BankExamsToday.com, best study material for bank exams


CRT
Find the least number which when divided by 15, 20, 24 and 30 leaves 1 as remainder but when divided by 19 leaves
no remainder.
1. 228
2. 361
3. 380
4. 399

35.

Five bells toll at an interval of 6, 9, 12, 15 and 18 seconds respectively beginning together. After what interval of
time will they toll again together?
1. 1 min
2. 4 min
3. 2 min
4. 3 min

36.

4 traffic signals at four different places change at an interval of 5 sec, 10 sec, 15 sec and 25 sec respectively. If they
change simultaneously at 10 : 20 A.M., at what time will they again in change simultaneously?
1. 10 : 22: 30 A.M.
2. 10 : 24: 30 A.M.
3. 10 : 21: 30 A.M.
4. 10 : 24: 40 A.M.

37.

4 wheels moving 24, 32, 48 and 52 revolutions in a minute starting at a certain point on the circumference
downwards. After what interval of time will they come together again in the same position?
1. 15 sec
2. 30 min
3. 30 sec
4. 15 min

38.

Four men start together to travel the same way around on a circular path of 36 km with speeds 2 km/h, 3 km/h, 4
km/h and 6 km/h respectively. When they meet at the same point again after?
1. 24 hours
2. 36 hours
3. 12 hours
4. 18 hours

39.

Find the least number of square tiles required to pave the floor of a room of 5 m 44 cm long and 3 m 74 cm broad.
1. 178
2. 164
3. 176
4. 172

40.

Find the least number of square tiles required to pave the floor of a room of 8 m 99 cm long and 6 m 67 cm broad.
1. 713
2. 723
3. 717
4. 719

41.

A trader has three kinds of liquids of first kind 117 lit, second kind 130 lit and the third kind 143 lit. Find the least
number of full casks of equal sizes which this can be stored without mixing them.
1. 28
2. 24
3. 36
4. 30

42.

The H.C.F. of two numbers is 11 and their product is 1452. How many pairs of such numbers possible?
1. 1
2. 2
3. 3
4. 4

43.

The H.C.F. of two numbers is 15 and their sum is 195. How many pairs of such numbers possible?
1. 1
2. 3
3. 4
4. 6

44.

The H.C.F. of two numbers is 23 and their sum is 184. How many such pairs of numbers are possible?
1. 0
2. 2
3. 3
4. 4

45.

Find the least perfect square which is exactly divisible by 20, 30, 40 and 60.
1. 2500
2. 3600
3. 900

od
ay
.

am
sT

Ex

an
k

.B

w
w

4. 1600

Find the least number by which 240 must be multiplied in order to produce a multiple of 300.
1. 2
2. 4
3. 5
4. 6

46.

co
m

34.

47.

64 mango trees, 48 apple trees and 80 orange trees have to plant in rows such that each row contains the same
number of trees of one variety only. Find the least number of rows in which the trees may be planted.
1. 18
2. 12
3. 16
4. 24

48.

Find the greatest number of five digits which can be divisible by to 15, 20, 24, 27, 32 and 36.
1. 90039
2. 90139
3. 86400
4. 90019

49.

Find the smallest number of five digits which when divided by 52, 56, 78 and 91 leaves remainders 28, 32, 54 and 67
respectively.
1. 10896
2. 10196
3. 10496
4. 10696

Get all banking study material on BankExamsToday.com

Page 30

www.BankExamsToday.com, best study material for bank exams


CRT
When finding the H.C.F. of two numbers by division method, the quotients obtained are 1, 2 and 5 respectively and
the last divisor is 34. Find the numbers.
1. 374 and 544
2. 324 and 646
3. 314 and 624
4. 616 and 348

51.

When finding the H.C.F. of two numbers by division method, the quotients obtained are 2, 2, 4 and 3 respectively
and their H.C.F. is 23. Find the smallest number which is less than 100.
1. 686
2. 667
3. 647
4. 657

52.

Four bells first begin to toll together with an interval of 5, 10, 15 and 20 seconds. How many times do they toll
together in an hour?
1. 60
2. 59
3. 61
4. 58

53.

3 men start together to walk along a circular track at the same rate. The length of their tracks is 56 cm, 64 cm and 80
cm. How far will they be in step again?
1. 21 m 20 cm
2. 22 m 40 cm
3. 20 m 20 cm
4. 21 m 40 cm

54.

Find the least number of soldiers in a regiment such that they stand in rows of 10, 15 and 20 and form a perfect
square.
1. 400
2. 1600
3. 900
4. 2500

55.

A trader has three kinds of sugar of 77 kg, 147 kg and 252 kg. Find the least number of bags of equal size required to
pack them without mixing.
1. 64
2. 68
3. 66
4. 60

56.

The sum of two numbers is 721 and their H.C.F. is 103. Find the numbers.
1. 319, 103
2. 309, 412
3. 103, 512

57.

The L.C.M. and H.C.F. of two numbers are 693 and 11 respectively. One of them is 77. Find the other.
1. 91
2. 98
3. 97
4. 99

58.

Find the least number of boys so that they can be arranged in the groups of 20 or 25 or 40.
1. 200
2. 250
3. 240
4. 216

59.

Find the least perfect square which is divisible by 12, 15 and 20.
1. 225
2. 900
3. 1600

60.

The product of two numbers is 768 and their H.C.F. is 8. What are the numbers?
1. 31 and 28
2. 32 and 24
3. 33 and 26
4. 35 and 24

4. 520, 201

w
w

Exercise

1) 2
2) 4
3) 3
4) 1
5) 3
6) 3
7) 2
8) 4
9) 2
10) 2

4. 2500

.B

an
k

Ex

am
sT

od
ay
.

co
m

50.

11) 2
12) 1
13) 4
14) 2
15) 3
16) 2
17) 1
18) 4
19) 3
20) 3

21) 1
22) 3
23) 2
24) 2
25) 4
26) 1
27) 3
28) 1
29) 3
30) 1

Get all banking study material on BankExamsToday.com

31) 1
32) 3
33) 3
34) 2
35) 4
36) 1
37) 1
38) 2
39) 3
40) 1

41) 4
42) 2
43) 4
44) 2
45) 2
46) 3
47) 3
48) 3
49) 1
50) 1

51) 2
52) 3
53) 2
54) 3
55) 2
56) 2
57) 4
58) 1
59) 2
60) 2

Page 31

www.BankExamsToday.com, best study material for bank exams


CRT
Percentages
Understanding Percentages:

E.g.

co
m

Percent: The term per cent means per hundred or for every hundred. The word is derived from the Latin word per
centum.
Percentage: A fraction whose denominator is 100 is called a percentage.
Rate per cent: The numerator of the fraction is called rate per cent.
5
and 5 percent means the same thing i.e. 5 parts out of every hundred parts.
100

Basic Formulae:

od
ay
.

2.

1
1
into a rate per cent, multiply it by100 and put % sign i.e. 100%
n
n
3
E.g. What percentage is equivalent to ?
4
3
Sol: x 100 = 25%
4

To convert any fraction

To convert a per cent into a fraction, drop the per cent sign and divid e the number by 100.

1
%?
3
1
1
25
25
1
Sol: 8 % =
=
x
=
3
100
12
3
3

E.g. What fraction is 8

x% of a given number (N) =


E.g. 75 % of 800 = ?

75
x 800 = 600
100

an
k

Sol: 75 % of 800 =

x
xN
100

Ex

3.

am
sT

1.

If A is x % more than that of B, then B is less than that of A by

x
100 %.
100 x

5.

If A is x % less than that of B, then B is more than that of A by

x
100 %.
100 x

6.

If A is x % of C and B is y % of C, then A =

7.

If two numbers are respectively x % and y % more than a third number, then the first number is

w
w

.B

4.

x
y

100 % of the second and

100
100

the second number is

8.

x
x 100% of B.
y

100
100

y
x

100 % of the first.

If two numbers are respectively x % and y % less than a third number, then the first number is

100
100

x
y

% of the second and


the second number is

100
100

y
x

100 % of the first.

Get all banking study material on BankExamsToday.com

Page 32

100

www.BankExamsToday.com, best study material for bank exams


CRT
If the price of a commodity increases by N%, then the reduction in consumption so as not to increase the

9.

expenditure is
10.

100 %.

If the price of a commodity decreases by N%, then the increase in consumption so as not to decrease the
expenditure is

N
100 N

100 %.

If a number is changed (increased/decreased) successively by x % and y % then net % change is given by


x

xy
% which represents increase or decrease in value according as the sign is +ve or ve.
100

co
m

11.

N
100 N

Note: If x and y indicates decrease in percentage, then put ve sign before x and y else +ve sign.
If the population of a town (or the length of a tree) is P and its annual increase is r%, then:
(i) Population (or length of tree) after n years = P 1

r
100

If the population (or value of a machine in rupees) is P an d annual decrease (or depreciation) is r%, then

am
sT

13.

r
.
100
P

(ii) Population (or length of tree) n years ago =

od
ay
.

12.

(i)

Population (or value of machine) after n years = P 1

(ii)

Population (or value of machine) n years ago =

r
.
100

r
100

x
100

y
100

an
k

K 1

Ex

14. If a number K is increased successively by x % followed by y % and z %, then the final value of K will be
z
100

15. In an examination, the minimum pass percentage is x%. If a student scores y marks and fails by z marks, then
the maximum marks in the examination is

100(y
x

z)

.B

16. In an examination a % and b % students respectively fail in two different subjects while c % students fail in
both the subjects, then the percentage of students who pass in both the subjects will be (100 ( a + b - c ))%.

w
w

Percentage Change:
A change can be of two types an increase or a decrease.
When a value is changed from initial value to a final value,

% change = (Difference between initial and final value/initial value) X 100


Eg: If 20 changes to 40, what is the % increase?
Soln: % increase = (40-20)/20 X 100 = 100%.
Note:
1. If a value is doubled the percentage increase is 100.
2. If a value is tripled, the percentage change is 200 and so on.
Percentage Difference:
% Difference = (Difference between values/value compared with) X 100.
Eg: By what percent is 40 more than 30?
Soln: % difference = (40-30)/30 X 100 = 33.33%
(Here 40 is compared with 30. So 30 is taken as denominator)

Get all banking study material on BankExamsToday.com

Page 33

www.BankExamsToday.com, best study material for bank exams


CRT
Eg: By what % is 60 more than 30?
Soln: % difference = (60-30)/30 X 100 = 100%.
(Here is 60 is compared with 30.)
Hint: To calculate percentage difference the value that occurs after the word than in the question can directly be used as
the denominator in the formula.

w
w

.B

an
k

Ex

am
sT

od
ay
.

co
m

Important Points to Note:


1. When any value increases by
a. 10%, it becomes 1.1 times of itself. (since 100+10 = 110% = 1.1)
b. 20%, it becomes 1.2 times of itself.
c. 36%, it becomes 1.36 times of itself.
d. 4%, it becomes 1.04 times of itself.
Thus we can see the effects on the values due to various percentage increases.
2. When any value decreases by
a. 10%, it becomes 0.9 times of itself. (Since 100-10 = 90% = 0.9)
b. 20%, it becomes 0.8 times of itself
c. 36%, it becomes 0.64 times of itself
d. 4%, it becomes 0.96 times of itself.
Thus we can see the effects on a value due to various percentage decreases.
Note:
1. When a value is multiplied by a decimal more than 1 it will be increased and when multiplied by less than 1 it will be
decreased.
2. The percentage increase or decrease depends on the decimal multiplied.
Eg: 0.7 => 30% decrease, 0.67 => 33% decrease, 0. 956 => 4.4% decrease and so on.
Eg: When the actual value is x, find the value when it is 30% decreased.
Soln: 30% decrease => 0.7 x.
Eg: A value after an increase of 20% became 600. What is the value?
Soln: 1.2x = 600 (since 20% increase)
x = 500.
Eg: If 600 is decrease by 20%, what is the new value?
Soln: new value = 0.8 X 600 = 480. (Since 20% decrease)
Thus depending on the decimal we can decide the % change and vice versa.
Eg: When a value is increased by 20%, by what percent should it be reduced to get the actual value?
Soln: (It is equivalent to 1.2 reduced to 1 and we can use % decrease formula)
% decrease = (1.2 1)/1.2 X 100 = 16.66%.
3.
When a value is subjected multiple changes, the overall effect of all the changes can be obtained by multiplying all
the individual factors of the changes.
Eg: The population of a town increased by 10%, 20% and then decreased by 30%. The new population is what % of the
original?
Soln: The overall effect = 1.1 X 1.2 X 0.7 (Since 10%, 20% increase and 30% decrease)
= 0.924 = 92.4%.
Eg: Two successive discounts of 10% and 20% are equal to a single discount of ___
Soln: Discount is same as decrease of price.
So, decrease = 0.9 X 0.8 = 0.72 => 28% decrease (Since only 72% is remaining).
SOLVED EXAMPLES

1.

If Srujana salary is 20% more than that of Deepa, then how much percent is Deepas salary less than that of
Srujana?
Sol: Here, x = 20.
Required answer

x
100 %
100 x

Get all banking study material on BankExamsToday.com

Page 34

www.BankExamsToday.com, best study material for bank exams


CRT
20
100 20

4
2
100
% = 16 % = 16 %.
3
6
6

100 %

If Anitas income is 30% less than that of Saritha, then how much percent is Sarithas income more than that of
Anitha?

co
m

2.

Sol: Here, x = 30.


=
=
=
3.

x
100 %
100 x
30
100 %
100 30
6
300
% = 42 %.
7
7

If A is 25% of C and B is 30% of C, then what percentage of A is B?


Sol: A = 25% of C and B = 30% of C

am
sT

B
30
=
A
25
30
B=
x 100% of A
25

= 120% of A

Two numbers are respectively 25% and 50% more than a third number. What percent is the first of the second?
Sol: Here, x = 25 and y = 50

100 x
100 % of the second
100 y

100 25
100 % of the second
100 50

5.

w
w

.B

an
k

So, First number

Ex

4.

od
ay
.

Required answer

500
% of the second
6
1
2
= 83 % = 83 % of the second.
3
6

Two numbers are respectively 20% and 32% less than a third number. What percent is the second of the first?
Sol: Here, x = 20 and y = 32

So, Second number

100
100

y
x

100 32
100 20

100 % of the first


100 % of the first

= 85% of the first.


6.

If the price of a commodity increases by 50%, find how much percent its consumption be reduced so as not
increase the expenditure.
Sol: Reduction in consumption

N
100 N

100 %

Get all banking study material on BankExamsToday.com

Page 35

50
100 50

100 %

100
%=
3

1
33 %.
3

If the price of a commodity decreases by 50%, find how much percent its consumption be increased so as not
decrease the expenditure.
Sol: Increase in consumption =

N
100 N

100
50
100 %
100 50

= 100%.

If the salary of Mr. Shashi is first increased by 18% and thereafter decreased by 15%, what is the net change in
his salary?
Sol:

Here, x = 18 and y = -15

am
sT

8.

od
ay
.

7.

co
m

www.BankExamsToday.com, best study material for bank exams


CRT

= x

xy
%
100

= 18 15

(18)(15)
%
100

= 18 15

(18)(15)
%
100

Ex

So, the net % change in the salary

9.

an
k

= 0.3%.
Since the sign is +ve, there is an increase in the salary of person by 0.3%.
The population of a town is decreased by 20% and 40% in two successive years. What percent population is
decreased after two years?
Sol: Here, x = - 20 and y = - 40

w
w

.B

So, the net % change in population

= x

xy
%
100
( 20)( 40)
%
100

20

40

60

800
%
100

= - 52%.

Since the sign is -ve, there is decrease in population after two years by 52%.
10. If the side of a square is increased by 10%, its area increased by k%. Find the value of k.
Sol: Area of square = side x side
So, net % change in area

= x

xy
%
100

Get all banking study material on BankExamsToday.com

Page 36

www.BankExamsToday.com, best study material for bank exams


CRT
(10)(10)
% [Take x, y = 10]
100

= 10 10
= 21%
Hence, the area is increased by 21%.
Here, k = 21.

Sol: Area of circle = x radius x radius


= x

xy
%
100

= 4

(4)(4)
% [Take x, y = 4]
100

= 8

4
16
% = 8
%.
25
100

od
ay
.

So, net % change in area

co
m

11. The radius of a circle is increased by 4%. Find the percentage increase in its area.

12. The tax on a commodity is diminished by 12% and its consumption increases by 10%. Find the effect on
revenue.
Sol: Revenue = tax x consumption

=
=

xy
%
100

am
sT

So, net % change in revenue = x

12

( 12)(10)
% [Take x = - 12, y = 10]
100

10

120
% = - 3.2%.
100

Ex

So, the revenue decreases by 3.2%.


13. The population of a town increases by 4% annually. If its present population is 12500, what will it be in 2 years
time?

an
k

Sol: Here, P = 12500, r = 4 and n = 2.

Population after n years = P 1

r
.
100

w
w

.B

Population after 2 years = 125001

4
100

1
25

= 12500 1
= 12500

= 12500

26
25
26
25

26
25

= 13520.
14. The population of a town increases by 10% annually. If its present population is 12100, what it was 2 years
ago?
Sol: Here, P = 12100, r = 10 and n = 2.

Get all banking study material on BankExamsToday.com

Page 37

www.BankExamsToday.com, best study material for bank exams


CRT
P

Population n years ago =

r
100
12100

10
100

12100
11
10

12100
11 11
10 10

12100
10 10 = 10000.
11 11

co
m

Population 2 years ago =

Sol: Here, P = 100000, x = 5, y = - 10 and z = 15


x
100

So, Population after 3 years = P 1

y
100

z
100

5
100

10
100

am
sT

= 100000 1

od
ay
.

15. The population of a town is 100000. It increases by 5% during the first year. During the second year, it
decreases by 10% and increases by 15% during the third year. What is the population after 3 years?

100000 105 90 115


100 100 100

15
100

108675

16. In an examination, a student must get 50% marks to pass. If a student, who gets 110 marks, fails by 50 marks,
find the maximum marks.

an
k

Ex

Sol: It is clear, Minimum Marks = 110 + 50 = 160


Let the maximum marks = x
So, 50% of x = 160
x = 320
17. In an examination, 40% students failed in Mathematics and 51% failed in Science. If 16% failed in both the
subjects, find the percentage of those who passed in both the subjects.

w
w

.B

Sol: Here, a = 40, b = 51 and c = 16


Percentage of students passing both the subjects
= (100 ( a + b - c ))%
= (100 (40+5116))%
= 25%.
Percentages Fractions Conversions:
The following is a table showing the conversions of percentages and decimals into fractions:
Percentage
Decimal
Fraction

10%
12.5%
16.66%
20%
25%
30%
33.33%
40%
50%
60%
62.5%
66.66%

0.1
0.125
0.1666
0.2
0.25
0.3
0.3333
0.4
0.5
0.6
0.625
0.6666

1/10
1/8
1/6
1/5
1/4
3/10
1/3
2/5
1/2
3/5
5/8
2/3

Get all banking study material on BankExamsToday.com

Page 38

www.BankExamsToday.com, best study material for bank exams


CRT

Part-1
Exercise

od
ay
.

co
m

70%
0.7
7/10
3/4
75%
0.75
80%
0.8
4/5
83.33%
0.8333
5/6
90%
0.9
9/10
100%
1.0
1
Similarly we can go for converting decimals more than 1 from the knowledge of the above cited conversions as follows:
We know that 12.5% = 0.125 = 1/8
Then, 1.125 = [8(1)+1]/8 = 9/8 (i.e., the denominator will add to numerator once, denominator remaining the same.
Also, 2.125 = [8(2)+1]/8 = 17/8 (here the denominator is added to numerator twice)
3.125 = [8(3)+1]/8 = 25/8 and so on.
Thus we can derive the fractions for decimals more than 1 by using those les than 1.
We will see how use of fractions will reduce the time for calculations:
Eg: What is 62.5% of 320?
Soln: Value = (5/8) X 320 (since 62.5% = 5/8)
= 200.

1.

What percentage is 150 ml of 3 litres?


1. 2%
2. 5%

2.

If 40% of a number added to 1800, it gives the number itself. Find the number?
1. 3000
2. 257
3. 1500
4. 4500

3.

The difference between 12 % and 37% of a number is 750. Find the number?
1. 1500
2. 4500
3. 3000
4. 257

4.

A number exceeds its 75% by 50. Then, find the number?


1. 175
2. 500
3. 750

5.

If 30% of a number added to 441, it gives the number itself. Find the number?
1. 350
2. 630
3. 580

6.

The price of an article X is 10% less than that of article Y. By how much percent of the price article Y is more than
that of the price of X?

am
sT

Ex

an
k
2. 11

1
%
9

3. 90%

4. 50%

4. 200
4. 250

4. 10%

w
w

In a library 50% of the books in Hindi, 40% of the remaining is English, and 30% of the remaining is Telugu. The
remaining 18,900 books are Marathi. Find the total number of books?
1. 3,15,000
2. 45,000
3. 63,000
4. 90,000
The cost of 1 kg of sweets increases every year by 10%, After two years the cost of 1 kg of sweets will increase by
1. 10%
2. 20%
3. 21%
4. 25%

8.

1
%
11

.B

1. 9
7.

3. 8%

9.

The marks obtained by two candidates A and B in arithmetic are 6.25 and 5.75 respectively. Marks percentage of A
exceeds that of B by
1. 7.8%
2. 8%
3. 8.7%
4. 8.8

10.

Gopal spends 30% of his monthly income on food articles, 40% of the remaining on conveyance and clothes and
saves 50% of the remaining. If his monthly salary is Rs.18,400, how much money does he save every month?
1. Rs.3864
2. Rs.4264
3. Rs.3624
4. Rs.5888

Get all banking study material on BankExamsToday.com

Page 39

www.BankExamsToday.com, best study material for bank exams


CRT
A man spends 25% of his income on house rent, 45% of this income on food and 40% of the balance on conveyance.
If he is left with Rs.540 his income is
1. Rs.4500
2. Rs.3000
3. Rs.1350
4. Rs.900

12.

In an examination 450 candidates were boys and 550 were girls. If 32% of the boys and 38% of the girls passed the
examination, the percentage of failed candidates is
1. 35.3%
2. 62%
3. 64.7%
4. 68%

13.

After having spent 25% of his money on machinery, 30% on raw material and 10% on staff, a person is left with
Rs.70,000. The amount spent on raw material is
1. Rs.64000
2. Rs.60000
3. Rs.48000
4. Rs.40000

14.

The price of kerosene increases by 25%. By how much percentage a family must reduce consumption of kerosene so
as not to increase the monthly expenditure on kerosene?
1. 15%
2. 20%
3. 25%
4. 30%

15.

Fifty percent students in a class are N.C.C cadets and 40 percent of the rest are scouts. Find the percentage of the
total who are neither N.C.C. nor Scouts.
1. 18%
2. 15%
3. 25%
4. 30%

16.

A monthly return railway ticket costs 25 percent more than a single ticket. A weeks extension can be had for the
former by paying 5 percent of the monthly tickets cost. If the money paid for the monthly ticket (with extension) is
Rs.84, the price of the single ticket is
1. Rs.48
2. Rs.80
3. Rs.64
4. Rs.72

17.

Mahesh gets a certain sum as pocket money every month. He spends 20 percent on purchasing books and 25 percent
of the remainder on stationary, 10 percent of what is left is given as charity. The rest he uses for purchasing sweets
etc, and this works out to be Rs.108. His monthly pocket money is
1. Rs.200
2. Rs.250
3. Rs.350
4. Rs.500

18.

The population of a city increases at the rate of 10% annually. Its present population is 90.51 lakhs. The population 3
years ago was nearly ________
1. 60 lakhs
2. 68 lakhs
3. 71 lakhs
4. 72.8 lakhs

19.

8% of the people eligible to vote are between 18 and 21. In an election, 85% of those eligible to vote, who were
between 18 and 21, actually voted. In that election, the number of persons between 18 and 21, who actually voted,
was what percent of those eligible to vote?
1. 4.2
2. 6.4
3. 6.8
4. 8

20.

When the price of wheat is increased by 20%, a family has to reduce the consumption by 4 kg keeping the
expenditure fixed at Rs.80. The increased price per kg is
1. Rs.20
3. Rs.5
3. Rs.4
4. Rs.10

od
ay
.

am
sT

Ex

an
k

.B

w
w

In an examination every candidate took History or Geography of both, 64.8% took History and 60.2% of candidates
took Geography. If the total number of candidates is 2000, the candidates who took both are
1. 500
2. 400
3. 375
4. None of these

21.

co
m

11.

22.

In a school of 400 students, 40% of girls, 20% of the boys and 30% of the students failed in the exam. The % of
students passed in the exam is:
1. 75%
2. 76%
3. 45%
4. None

23.

In an election, the winning candidate received 10400 votes, which represented 52% of the electorate. The only other
candidate secured votes from 23% of the electorate, calculate the number of people who did not cast their votes?
1. 3500
2. 4000
3. 4800
4. 5000

Get all banking study material on BankExamsToday.com

Page 40

www.BankExamsToday.com, best study material for bank exams


CRT
In a competitive examination, a candidate obtained 45% marks and failed to get selected by 120 marks. Another
candidate obtained 58% marks and failed to get selected by 16 marks. Find the maximum marks and the percentage
of marks for selection?
1. 600, 60%
2. 750, 55%
3. 800, 65%
4. 800, 60%

25.

Three persons A, B and C whose salaries together amount to Rs.2560, spend 90, 85 and 80 percent in their salaries
respectively. If their savings are 5:6:14, find their respective salary?
1. 800, 640, 1120
2. 840, 600, 1120
3. 780, 760, 1020
4. 800, 760, 1000

26.

In an examination a candidate attempts 90% of the total questions at of these 80% his answers are correct. Each
question carries 2 marks for correct answer and negative (-1) mark for wrong answer. If the marks secured by the
candidate is 189 what is the total number of questions?]
1. 250
2. 150
3. 100
4. 350

27.

The sales of a company, reduced to 20%. After how much percentage increase the sales of the company be in
original?
1. 20%
2. 25%
3. 10%
4. 30%

28.

In a town 60% of people read Hindu, 50% of read the Times of India, 30% read both. Find the percentage of the
people who read neither?
1. 80%
2. 20%
3. 10%
4. 40%

29.

The population of the town 20,000. If the number of males are increased by 40% then at the end of the year the
population will be 23,200. Find the number of females, males in the town at the beginning?
1. 12000, 8000
2. 18000, 12000
3. 10000, 10000
4. None

30.

In an election where there are two candidates only contested. One candidate who gets 70% of the votes is elected by a
majority of 2000 votes. What is total number of votes?
1. 5000
2. 6000
3. 3000
4. None

31.

A candidate has to obtain 33% of the total marks to pass the examination. Ram got 25% of the total marks and failed
by 40 marks. What are the maximum marks?
1. 400
2. 750
3. 500
4. 5000

32.

In an examination A gets 28% of the total marks and fails by 24 marks. While B gets 31% of the total marks and got
48 marks more than the pass mark. Find the maximum marks and pass %?
1. 3000, 29%
2. 2400, 29%
3. 2500, 29%
4. 3000, 30%

33.

In an examination there were 5000 candidates out of which 3000 were boys. If 60% of the boys and 20% of the girls
passed. Find the percentage of failed candidates?
1. 60%
2. 76%
3. 56%
4. 62%

od
ay
.

am
sT

Ex

an
k

.B

w
w

If the price of sugar decrease by 25%. Find by how much percent must a householder increase his consumption of
that article so as not to reduce his expenditure?

34.

co
m

24.

1. 33

35.

2. 11

1
%
9

3.

3
%
100

4. None

If the price of wheat decreased by 20%, find by how much percent must a householder increase his consumption of
that article so as not to reduce his expenditure?
1. 11

36.

1
%
3

2
%
3

2. 13

2
%
3

3. 15

2
%
3

4. 16

2
%
3

The population of a town is 1,00,000 now. If it increases at a rate of 10% p.a. find the population after 2 years?
1. 112000
2. 121000
3. 120100
4. None

Get all banking study material on BankExamsToday.com

Page 41

www.BankExamsToday.com, best study material for bank exams


CRT
If the population of a town is 5000 now and it increases at 10% first year 20% in the second year then its population
after 2 years?
1. 6600
2. 5500
3. 6000
4. 5000

38.

Two numbers are 20% and 40% of the third number. How much % of the first number in the second number?
1. 20%
2. 200%
3. 10%
4. 100%

39.

A reduction of 20% in the price of oranges would enable a person to get 2 dozen more for Rs.50. Find the reduced
price per dozen.
1. Rs.3
2. Rs.5
3. Rs.4
4. Rs.4.5

40.

The tax on a commodity is diminished by 15% and its consumption increased by 10%. What is the effect on the
revenue derived from it?
1. 5.5% increase
2. 5.5% decrease
3. 6.5% decrease
4. 6.5% increase

Answers

am
sT

21) 1
22) 4
23) 4
24) 4
25) 1
26) 2
27) 2
28) 2
29) 1
30) 1

Ex

Exercise:

11) 2
12) 3
13) 2
14) 2
15) 4
16) 3
17) 1
18) 2
19) 3
20) 3

an
k

1) 2
2) 1
3) 3
4) 4
5) 2
6) 2
7) 4
8) 3
9) 3
10) 1

od
ay
.

co
m

37.

d. None

2. By what % is 200 more than 50?


a. 100
b. 200

d. None

.B

1. If 20% of 40% of a = 25% of a% of b, then what is b?


a. 8/5
b. 16/25
c. 8/25
c. 300

w
w

3. A value changes from 30 to 80. What is the percentage change?


a. 125
b. 166.66
c. 156

31) 3
32) 2
33) 3
34) 1
35) 4
36) 2
37) 1
38) 2
39) 2
40) 3

d. None

4. The population of a city is increased by 30% and thus became 78000. What is the original population?
a. 76000
b. 64200
c. 60000
d. None
5. In a theatre, the number of seats is increased by 20% and the price per ticket is increased by 10% but the public response
decreased by 30%. What is the net effect on the economy of the theatre?
a.10% rise
b. 7% fall
c. 7% rise
d. None
6. A saves 20% of his income. His income is increased by 20% and so he increased his expenditure by 30%. What is the
percentage change in his savings?
a. 20% fall
b. 4% fall
c. 20% rise
d. 4% rise

Get all banking study material on BankExamsToday.com

Page 42

www.BankExamsToday.com, best study material for bank exams


CRT
7. The price of petrol is increased by 25%. By what percent the consumption be reduced to make the expenditure remain the
same?
a. 25%
b. 33.33%
c. 20%
d. None
8. The side of a square is increased by 20%. The percentage change in its area is ___
a. 20%
b. 44%
c. 36%
d. None

co
m

9. If the length of a rectangle is increased by 33.33%, by what percentage should the breadth be reduced to make the area
same?
a. 20%
b. 33.33%
c. 25%
d. None

od
ay
.

10. In an election between two candidates, A and B, A secured 56% of the votes and won by 48000 votes. Find the total
number of votes polled if 20% of the votes were declared invalid.
a. 500000
b. 400000
c. 600000
d. None
11. A reduction of 10% in price of sugar enables a housewife to buy 5 kg more for Rs. 300/-. Find the reduced price per kg
of sugar.
a. 5/b. 4.5/c. 6/d. None

am
sT

12. From a 20lt solution of alt and water with 20% salt, 2lt of water is evaporated. Find the new % concentration of salt.
a. 20%
b. 23%
c. 25%
d. None
13. In a list of weights of candidates appearing for police selections, the weight of A is marked as 58 kg instead of 46.4 kg.
Find the percentage of correction required.
a. 30
b. 20
c. 24
d. None

Ex

14. A person spends 20% of his income on rent, 20% of the rest on food, 10% of the remaining on clothes and 10% on
groceries. If he is left with Rs. 9520/- find his income.
a. 10000/b. 15000/c. 20000/d. None

an
k

15. A shopkeeper offers three successive discounts of 10%, 20% and 30% to a customer. If the actual price of the item is
Rs. 10000, find the price the custome has to pay to the shopkeeper.
a. 5040/b. 4000/c. 6000/d. None

.B

16. If 10lt solution of water and alcohol containing 10% alcohol is to be made 20% alcohol solution, find the volume of
alcohol to be added.
a. 1 lt
b. 1.25 lt
c. 1.5 lt
d. 2 lt

w
w

17. A is twice B and B is 200% more than C. By what percent is A more than C?
a. 400
b. 600
c. 500
d. 200

18. In an examination, a student secures 40% and fails by 10 marks. If he scored 50%, he would pass by 15 marks. Find the
minimum marks required to pass the exam.
a. 250
b. 100
c. 110
d. 125
19. If A is 20% taller than B, by what percent is B shorter than A?
a. 20%
b. 25%
c. 16.66%

d. None

20. The population of a town increases at a rate of 10% for every year. If the present population is 12100, find the
population two years ago.
a. 11000
b. 9800
c. 10000
d. 10120
21. A solution of salt and water contains 15% salt. If 30 lt water is evaporated from the solution the concentration becomes
20% salt. Find the original volume of the liquid before water evaporated.
a. 100 lt
b. 120 lt
c. 200 lt
d. None
Get all banking study material on BankExamsToday.com

Page 43

www.BankExamsToday.com, best study material for bank exams


CRT
22. If 240 lt of oil is poured into a tank, it is still 20% empty. How much more oil is to be poured to fill the tank?
a. 300 lt
b. 60 lt
c. 120 lt
d. None
23. A and B were hired for the same salary. A got two 40% hikes whereas B got a 90% hike. What is the percentage
difference in the hikes they got?
a. 16%
b. 6%
c. 10%
d. 8%

co
m

24. The population of a town doubled every 5 years from 1960 to 1975. What is the percentage increase in population in
this period?
a. 800
b. 400
c. 700
d. 600

od
ay
.

25. In a test of 80 questions, Jyothsna answered 75% of the first 60 questions correctly. What % of the remaining questions
she has to answer correctly so that she can secure an overall percentage of 80 in the test?
a. 80%
b. 90%
c. 85%
D. 95%

Answers

1/5 X 2/5 X a = X a X b => b = 8/25


% difference = (200-50)/50 X 100 = 300 %
% increase = (80-30)/30 X 100 = 166.66 %
1.3 x = 78000 => x = 60000.
Net effect = 1.2 X 1.1 X 0.7
= 0.924 => 7.6% decrease.
6. Let I be the income.
Expenditure = 0.8I
Savings = 0.2I => 20%
New income = 1.2I (since 20% rise)
New expenditure = (0.8I) X 1.3 (Since 30% rise)
= 1.04I
So, new savings = 1.2I 1.04I = 0.16I => 16%
(So income decreased form 20% to 16%)
% decrease = (20-16)/20 X 100 = 20%.
7. It is equivalent to 1.25 decreased to 1.
% decrease = (1.25-1)/1.25 X 100 = 20%
8. % change in area = 1.2 X 1.2 (since area = side X side)
= 1.44 => 44%.
9. It is equivalent to 1.25 decreased to 1. So 20% decrease.
10.
Valid Votes:
A got 56% => B got 44%
Difference = 12% = 48000
So, 100% = 400000. These are valid votes.
But valid votes are only 80% of total votes.
So, 80% of total votes = 400000 => total votes = 500000
11. Total money = Rs. 300.
Saving of the lady = 10% of 300 = 30/With 30/- she bought 5 kg sugar => each kg costs Rs. 6/12. In 20lt, salt = 20% => 4 lt.
New volume = 18 lt (2 lt evaporated)
So, new % = 4/18 X 100 = 22.22%
13. % correction = (58-46.4)/58 X 100 = 20%
14. Three successive decreases of 20%, 20% and 10% => 0.8 X 0.8 X 0.9 = 0.576
Again 10% decrease => 0.576 0.1 = 0.476.
So, 0.476 x = 9520 => x = 20000.
15. Total discount = 0.9 X 0.8 X 0.7 = 0.504 of actual price.
So, price = 0.504 X 10000 = 5040.

w
w

.B

an
k

Ex

am
sT

1.
2.
3.
4.
5.

Get all banking study material on BankExamsToday.com

Page 44

w
w

.B

an
k

Ex

am
sT

od
ay
.

16. In 10 lt, alcohol is 10% = 1 lt.


Let x lt alcohol is added.
So, (1+x)/(10+x) = 20% = 1/5 => x = 1.25 lt.
17. A = 2B and B = 3C (ince 200% more)
A = 6C => 500 % more.
18. 50% of max marks 40% of max marks = 25
max marks = 250
Pass marks = 40% of max + 10 => 100 + 10 = 110.
19. A = 1.2 B => B = A/1.2 => 0.8333A => 16.66%.
(OR) Decrease from 1.2 to 1 => 16.66%.
20. 1.1 X 1.1 X x = 12100 => x = 10000.
21. Salt = 15% of x = 0.15x (x = volume of solution)
Now, 0.15x/(x-30) = 20% = 1/5 (since 30 lt evaporated)
x = 120 lt
22. 20% empty => 80 % full = 240 lt => 20% = 60 lt
23. A => 1.4 X 1.4 = 1.96
B => 1.9
=> 6% difference.
24. From 1960 to 1975, in 15 years population doubled every 5 yrs => three times
So, 2 X 2 X 2 = 8 times => 700% more.
25. [(75% X 60) + (x% X 20)] / 80 = 80% => x = 95. (since required is 80%)
(OR) 60 out of 80 is 3/4. So, (3/4 X 75) + (1/4 X x) = 80 => x =95.

co
m

www.BankExamsToday.com, best study material for bank exams


CRT

Get all banking study material on BankExamsToday.com

Page 45

www.BankExamsToday.com, best study material for bank exams


CRT
Profit and Loss
Introduction:
In any business transaction, it is common to have either profit or loss.
But the aim of any business is to earn profit.
The most commonly used term involving sale and purchase of any business are Cost Price and Selling Price.

co
m

1.
2.
3.

od
ay
.

What is Profit?
When a person does a business transaction and gets more than what he had invested, then he is said to have profit.
The profit he gets will be equal to the additional money he gets other than his investment.
So profit can be understood as the extra money one gets other than what he had invested.
Eg: A person bought an article for Rs. 100 and sold it for Rs. 120. Then he got Rs. 20 extra and so his profit is Rs. 20.
What is Loss?
When a person gets an amount less than what he had invested, then he is said to have a loss. The loss will be equal
to the deficit he got than the investment.
Eg: A person bought an article at Rs. 100 and sold it for Rs. 90. Then he got a deficit of Rs. 10 and so his loss is Rs. 10.

am
sT

Cost Price (CP):


The money that the trader puts in his business is called Cost Price. The price at which the articles are bought is
called Cost Price.
In other words, Cost Price is nothing but the investment in the business.

Ex

Selling Price (SP):


The price at which the articles are sold is called the Selling Price. The money that the trader gets from the business
is called Selling Price.
In other words, Selling Price is nothing but the returns from a business.

an
k

Marked/Market/List Price (MP):


The price that a trader marks or lists his articles to is called the Marked Price.
This is the only price known to the customer.

Profit or loss percentage is to be applied always to the Cost Price only.


Discount percentage is to be applied always to the Marked Price only.

w
w

Note:
1.
2.

.B

Discount:
The waiver of cost from the Marked Price that the trader allows a customer is called Discount.

Relationship Among CP, SP and MP:


A trader adds his profit to the investment and sells it at that increased price.
Also he allows a discount on Marked Price and sells at the discounted price.
So, we can say that,
o SP = CP + Profit. (CP applied with profit is SP)
o SP = MP Discount. (MP applied with discount is SP)
Understanding Profit and Loss:
So, by now we came to know that if CP is increased and sold it would result in profit and vice versa.
Also whatever increase is applied to CP, that increase itself is the profit.
For Rs. 10 profit, CP is to be increased by RS. 10 and the increased price becomes SP.
For 10% profit, CP is to be increased by 10% and it is the SP.
(From previous chapter we know that any value increased by 10% becomes 1.1 times.)
So, for 10% profit, CP increased by 10% => 1.1CP = SP.
Get all banking study material on BankExamsToday.com

Page 46

www.BankExamsToday.com, best study material for bank exams


CRT
SP = 1.1CP => SP/CP = 1.1 => 10% profit
SP = 1.07CP => SP/CP = 1.07 => 7% profit
SP = 1.545CP => SP/CP = 1.545 => 54.5% profit and so on.

o
o
o
Similarly,

If SP/CP > 1, it indicates profit.


If
SP/CP

<

1,

it

co
m

Note:
1.
2.

o SP = 0.9CP => SP/CP = 0.9 => 10% loss (Since 10% decrease)
o SP = 0.76CP => SP/CP = 0.76 => 24% loss and so on.
So, to calculate profit % or loss %, it is enough for us to find the ratio of SP to CP.
indicates

loss.

od
ay
.

Multiple Profits or losses:


A trader may sometimes have multiple profits or losses simultaneously. This is equivalent to having multiple
changes and so all individual changes are to be multiplied to get the overall effect.
Formulae:

3.

When S.P. and gain% are given, then


C.P.

4.

100
100

100
100 Loss%

C.P.

an
k

S.P.

When C.P. and loss% are given, then


100 Loss%
100

S.P.

C.P.

If a man buys a items for Rs.b and sells c items for Rs.d, then the

.B

7.

Gain%
100

When S.P. and loss% are given, then


C.P.

6.

S.P.

When C.P. and gain% are given, then


S.P.

5.

100
Gain%

am
sT

2.

Gain 100
C.P.
Loss 100
Loss % =
C.P.

Gain % =

Ex

1.

ad bc
bc

100

w
w

Gain or loss % =

Note: a) In case of gain percent, the result will be positive.


b) In case of loss percent, the result will be negative.
If the C.P. of x articles = S.P. of y articles, then

8.

Gain or loss % =

Note:
9.

y
y

100%

a) If x > y, it is % gain.
b) If x < y, it is % loss.

The Cost Price of an article is C.P. If it is sold at S.P 1., then gain% or loss% is x and if it is sold at a price S.P2.,
gain% or loss% is y then
S.P1
100 x

S.P2
C.P
(Or)
100
100 y

S.P1
x

S.P2
y

Get all banking study material on BankExamsToday.com

Page 47

www.BankExamsToday.com, best study material for bank exams


CRT
Note: a) If x or y is negative it indicates a loss.
b) If x or y is positive it indicates a gain.
A sells an article to B at a gain or loss of x%, and B sells it to C at a gain or loss of y%. If C pays Rs.z for it to B,
then
C.P. for A =

11.

y)

a) If x or y is negative it indicates a loss.


b) If x or y is positive it indicates a gain.

If A sells an article to B at a gain or loss of x%, and B sells it to C at a gain or loss of y% then the
Resultant Profit% or Loss % = x
Note:

xy
100

If x or y is negative it indicates a loss.


If x or y is positive it indicates a gain.
This expression represents resultant profit% or loss% according as it is positive or negative.

1.
2.
3.

If two different articles are sold at the same S.P., getting gain or loss of x% on the first transaction and gain or loss of
y% on the second transaction, then the
Overall % gain or % loss =

am
sT

12.

co
m

Note:

1002 z
(100 x)(100

od
ay
.

10.

100(x y) 2xy
%
(100 x) (100 y)

Note: This expression represents overall gain or loss according as its sign is positive or negative.]
If two different articles are sold at the same S.P., getting gain of x% on the first transaction and gain or loss of x% on
the second transaction, then the
Overall % loss =

x
10

Ex

13.

14.

an
k

Note: In this type of questions there is always a loss.


If a shopkeeper uses faulty measure and sells his goods at a gain or loss of x% then the
Overall % gain or % loss is

y x
100 y

w
w

Overall % gain or loss =

a.
b.

17.

A(100
(100 - loss%)
A(100
The C.P. of the item sold at gain =
(100 - loss%)

The C.P. of the item sold at loss =

gain%)
(100 gain%)
loss%)
(100 gain%)

If two successive discounts on an article at x% and y% respectively, then the


Overall Discount = x

18.

100 %

If a person buys two items for Rs.A and sells one at a loss of l% and other at a gain of g% and if each item was sold
at the same price, then

16.

True m easure
Faulty m easure

Note: If the merchant sells his goods at C.P., then x = 0.


If a merchant uses y% less weight or length and sells his goods at a gain or loss of x% then the

.B

15.

100 g
100 x

xy
%
100

If three successive discounts on an article at x%, y% and z% respectively, then the

Get all banking study material on BankExamsToday.com

Page 48

www.BankExamsToday.com, best study material for bank exams


CRT
Overall Discount = x

(xy

yz zx)
100

xyz
1002

A shopkeeper sells an item at Rs.x after giving a discount of d% on labeled price. Had he not given the discount, he
would have earned a profit of p% on the C.P. then the C.P. of each item is given by
C.P. =

1002 x
(100 d)(100 p)

co
m

19.

Solved Examples part -1


If C.P. = Rs.440, S.P. = Rs.480 then find the profit.
Sol: C.P. = Rs.440, S.P. = Rs.480
Profit = S.P. C.P. = 480 440 = Rs.40.
2.

If C.P. = Rs.135, S.P. = Rs.120 then find the loss.

3.

The cost price of a pen is Rs.400 and the selling price is Rs.460. Find the gain %.
Sol: C.P. = Rs.400, S.P. = Rs.460
Gain = S.P. C.P. = 460 400 = Rs.60
Gain 100
C.P.

60
100
400

15% .

Ex

Gain % =
4.

am
sT

Sol: C.P. = Rs.135, S.P. = Rs.120


Loss = C.P. S.P. = 135 120 = Rs.15.

Rajani bought a jewel for Rs.825 and sold it for Rs.750. Find the loss%.

Loss % =

an
k

Sol: C.P. = Rs.825, S.P. = Rs.750


Loss = C.P. S.P. = 825 750 = Rs.75
Loss 100
C.P.

75 100
825

100
11

1
%.
11

Shashi buys a T.V. set for Rs.9500. For how much should he sell in order to gain 4%?

.B

5.

od
ay
.

1.

Sol: C.P. = Rs.9500, gain% = 4%


100

Gain%
100

w
w
S.P.

S.P.

6.

100 4
100

C.P., if C.P. and gain% are given

9500 104 95

Rs. 9880.

Hari loses 8% by selling a Cooler for Rs.5520. Find the C.P. of the cooler.
Sol: S.P. = Rs.5520, loss% = 8%

C.P.

C.P.

7.

100
100 Loss%

100
100 8

S.P., if S.P. and loss% are given

5520

100
5520 Rs.6000.
92

By selling a toy for Rs.422, Kiran gains 5

1
% . Find the C.P. of the toy.
2

Get all banking study material on BankExamsToday.com

Page 49

www.BankExamsToday.com,continuous job updates for every hour


CRT
Sol: S.P. = Rs.422, gain% = 5

C.P.

8.

100

100
Gain%

100
11
100
2

S.P., if S.P. and gain% are given

422

200
422
211

Rs.400

co
m

C.P.

11
1
%.
%=
2
2

Venu buys oranges at the rate of Rs.10 per dozen and sells them at rate of 16 for Rs.11. Find his % gain or loss.
Sol: a = 12, b = Rs.10, c = 16, d = Rs.11
132
1
160

100%

28
100%
160

100%

od
ay
.

ad
1
bc

Gain or loss% =

17.5%

Since, the result is negative; there is a loss of 17.5%


9.

11 apples are bought at Rs.10 and sold at 10 for Rs.11. What is gain or loss%?

100%

121
1
100

100%

Ex

ad
1
bc

Gain or loss% =

am
sT

Sol: a = 11, b = Rs.10, c = 10, d = Rs.11

21
100%
100

21%.

an
k

10. The selling price of 18 articles is equal to the cost price of 10 articles. What is the profit%?
Sol: x = 18, y = 10

.B

Gain% =

100%

18 10
100%
10

80%.

w
w

11. By selling a mobile for Rs.2400, Phani lost 10%. What percent shall he gain or lose by selling it for Rs.2800.
Sol:S.P1.= Rs.2400, x = - 10%, S.P2.= Rs.2800, y =?
Here negative sign for x indicates loss.

We have,

S.P1
100 x

S.P2
100 y

2400
100 10

2800
100 y

6y

30

6
90

7
100 y

600 6y

630

Since the result is positive, Phani can have a gain of 5% by selling it for Rs.2800.
12. Avinash sells a bike to Shireesh at a gain of 15% and Shireesh again sells to Teja at a profit of 10%. If Teja pays
Rs.37950 to Shireesh then what is the cost price of the bike for Avinash?
Sol: Here, x = 15, y = 10 and z = Rs.37,950

Get all banking study material on BankExamsToday.com

Page 50

www.BankExamsToday.com,continuous job updates for every hour


CRT
C.P. for Avinash =

1002 z
(100 x)(100

y)

100 100 37950


(100 + 15)(100 + 10)

100 100 37950


115 110

= Rs.30,000.

co
m

13. Mounika sells an i-pod to Bindu at a gain of 12% and Bindu again sells it to swapna at a loss of 20%. If swapna
pays Rs.2240 to Bindu then what is the cost price of i-pod to Mounika?
Sol: Here, x = 12, y = - 20 and z = Rs.2240
1002 z
(100 x)(100

y)

od
ay
.

C.P. for Mounika =

100 100 2240


(100 12)(100 20)
100 100
4

100 100 2240


112 80

Rs.2500.

Sol: Here, x = 3, y = 5
Resultant Profit% = x

xy
100

am
sT

14. Sachin sells a machine to Krishna at a profit of 3% and Krishna sells it to Dhruva at a profit of 5%. Find the
resultant profit%.

3(5)
100

= 3 5

15
100

3
%.
20

an
k

Sol: Here, x = 10, y = - 8

Ex

15. Pratap sells a trouser to Rajesh at a profit of 10% and Rajesh sells it to Chandu at a loss of 8%. Find the resultant
profit or loss%.

Resultant Profit or loss% = x

.B

10

120
100

xy
100

( 8)

10( 8)
100

80
100

1
1 %.
5

w
w

This represents profit as the sign is positive.

16. Manoj sold two laptops, each for Rs.30000. If he makes 15% profit on the first and 10% loss on the second, what
is his gain or loss% on the whole transaction?
Sol: Here, x = 15, y = - 10
Overall % gain or % loss

100(x y) 2xy
%
(100 x) (100 y)

100(15 10) 2(15)( 10)


%
(100 15) (100 10)

500 300
%
205

200
%
205

40
%.
41

This represents profit as the sign is positive.

Get all banking study material on BankExamsToday.com

Page 51

www.BankExamsToday.com,continuous job updates for every hour


CRT
17. Chary sold two bicycles for Rs.1000 each, gaining 20% on one and losing 20% on the other. Find his total gain or
loss%.
Sol: Here, x = 20
x
10

20
10

% = 4%.

co
m

Overall % loss =

18. A dishonest shopkeeper professes to sell his goods at C.P. but uses a 950 g for kg weight. Find his gain%.
Sol: Here, True measure = 1000 g, False measure = 950 g, x = 0
Overall gain% is given by
True m easure
Faultym easure

od
ay
.

100 g
100 x

am
sT

100 g 1000
100 g 20
100
950
100
19
1900 19g 2000 19g 100
100
5
g
5
%.
19
19

19. A shopkeeper sells the goods at 22% loss on the C.P. but uses 10% less weight. What is his gain or loss%?
Sol: Here, x = - 22, y = 10
10 22
100 10

100 %

12
100 %
90

13

Ex

Overall gain or loss% =

120
9

40
3

an
k

1
%.
3

This represents loss as the sign is negative.

.B

20. Suresh buys two books for Rs.615 and sells one at a loss of 10% and other at a gain of 15%. If both the books are
sold at the same price, then find the cost price of two books.
Sol: The C.P. of the item sold at loss of 10%

A(100 gain%)
(100 - loss%) (100 gain%)
615 (100 15)
615 115
=
(100 - 10) (100 15)
205

w
w

= 3 x 115 = Rs.345.

The C.P. of the item sold at gain of 15%

A(100 loss%)
(100 - loss%) (100 gain%)
615 (100 10)
615 90
=
(100 - 10) (100 15)
205

= 3 x 90 = Rs.270.

Get all banking study material on BankExamsToday.com

Page 52

www.BankExamsToday.com, best study material for bank exams


CRT
21. Find a single discount equivalent to two successive discounts of 20% and 30%.
Sol: Overall Discount = x

xy
%
100

= 20 30

20(30)
%
100

44%.

Sol: Overall Discount


y

= 20 30

(xy

40

yz zx)
100
(20)(30)

600 1200 800


100

= 90

xyz
1002

(30)(40)
100

(40)(20)

24000
%
100 100

(20)(30)(40)
1002

90

= (90 26 2.4)% 66.4%.

od
ay
.

= x

co
m

22. Find a single discount equivalent to two successive discounts of 20% and 30% and 40%.

2600
100

24000
%
10000

am
sT

23. A shopkeeper sold the shirts at Rs.306 each after giving 15% discount on labelled price. If he had not given the
discount, he would have earned a profit of 20% on the cost price. Find the C.P. of each shirt.
Sol: Here, labeled price x = Rs.306, discount d = 15%, profit p = 20%
1002 x
(100 d)(100 p)

100 100 306


(100 15)(100 20)

100 100 306


85 120

Ex

Required C.P. =

= Rs.300.

1.

an
k

Solved Examples part 2:

A trader uses a 800gm weight instead of 1 kg. Find his profit %.


Soln: (He is buying 800 gm but selling 1000 gm.

.B

So, CP is for 800 gm and SP is for 1000 gm.)


SP/CP = 1000/800 = 1.25 => 25% profit.

A trader uses 1 kg weight for 800 gm and increases the price by 20%. Find his profit/loss %.

w
w

2.

Soln: 1 kg weight for 800 gm => loss (decrease) => 800/1000 = 0.8

20% increase in price => profit (increase) => 1.2

3.

So, net effect = (0.8) X (1.2) = 0.96 => 4% loss.

A milk vendor mixes water to milk such that he gains 25%. Find the percentage of water in the mixture.
Soln: To gain 25%, the volume has to be increased by 25%.
So, for 1 lt of milk, 0.25 lt of water is added => total volume = 1.25 lt
% of water = 0.25 / 1.25 X 100 = 20%.

4.

A trader bought an item for Rs. 200. If he wants a profit of 22%, at what price must he sell it?
Soln: CP=200, Profit = 22%.
So, SP = 1.22CP = 1.22 X 200 = 244/-.

Get all banking study material on BankExamsToday.com

Page 53

www.BankExamsToday.com, best study material for bank exams


CRT
5.

A person buys an item at Rs. 120 and sells to another at a profit of 25%. If the second person sells the item to
another at Rs. 180, what is the profit % of the second person?
Soln: SP of 1st person = CP of 2nd person = 1.25 X 120 = 150.
SP of 2nd person = 180.
Profit % = SP/CP = 180/150 = 1.2 => 20%.
A milk vendor mixes water to 20 lt of milk such that the ratio of milk and water is 4:3. He sold the mixture at Rs.

co
m

6.

12 per liter but bought the milk at Rs. 10 per liter. Find the profit % of the vendor.
Soln: milk : water = 4:3 => he bought 4 parts (milk) but sold 7 parts (mixture)
CP = 10 and SP = 12.
7.

od
ay
.

So, profit % = (SP/CP) X (SP/CP) = (7/4) X (12/10) = 2.1 => 110% gain.

A trader buys some apples at a price of 10 apples for Rs. 8 and sold them at a price of 8 apples for Rs. 10. Find his
profit or loss %.

Soln: He bought 10 apples for Rs. 8 and sold 8 apples for Rs. 10 => clearly got profit

SP > CP => (SP/CP) X (SP/CP) = (10/8) X (10/8) = 100/64 = 1.5625 => 56.25 % gain.

am
sT

8. A trader allows a discount of 25% on his articles but wants to gain 50% gain. How many times the CP should be
marked on the items?

Soln: CP applied with profit = MP applied with discount = SP

1.5CP = 0.75MP (since 50% gain and 25% discount) => MP = 2CP.
9.

By selling an item at a price a trader gains 40%. What is the profit / loss % if the item is sold at half the price?

Ex

Soln: SP =1.4CP => (SP/2) = 0.7CP => 30% loss.

10. A trader gets a profit of 25% on an article. If he buys the article at 10% lesser price and sells it for Rs. 2 less, he

an
k

still gets 25% profit. Find the actual CP of the article.


Soln: 25% gain => SP = 1.25CP..1.
Now, CP is 10% less => 0.9CP and SP is Rs. 2 less => (SP-2).

.B

Still, profit is 25% => (SP-2)=1.25(0.9CP) , where SP = 1.25CP (From 1)


CP = Rs. 16.

w
w

11. A trader gets a discount of 20% from the dealer and marks it at 20% more price then the actual MP to the
customer. Find his overall gain %.

Soln: Let MP be the price on the item.


Then, CP=0.8MP (20% discount) and SP = 1.2MP.

So, gain => SP/CP = 1.2/0.8 = 1.5 => 50%.


12. A trader allows a discount of 20% to the customer after marking the item up by 25%. Find his gain/loss% if he is
given a commission of 20% of the MP by the dealer.
Soln: Traders SP = 0.8 X (1.25MP) = MP (since 20% discount on 25% raised price)
Traders CP = 0.8 MP (20% commission)
So, gain = SP/CP = MP/0.8MP = 1.25 => 25%.

Get all banking study material on BankExamsToday.com

Page 54

www.BankExamsToday.com, best study material for bank exams


CRT
Exercise
1. The profit obtained by selling an article for Rs.56 is the same as the loss obtained by selling it for Rs.42. What is the cost
price of the article?
1) Rs.40

2) Rs.50

3) Rs.49

4) None of these

1) 13.13%

2) 13.33%

co
m

2. A dealer professes to sell his goods at cost price and uses an 880gm weight instead of a kg. What is his percentage of
gain?
3) 13.36%

4) 13.63%

od
ay
.

3. P sold an article for Rs.1,080 thereby losing 10% Q sold another article for Rs.1,800 at a loss of 10%. Who incurred a
greater loss?
1) P
2) Q
3) Cannot say
4) Both have equal
4. Swapna bought 15 apples for Rs.10 and sold them at the rate of 12 apples for Rs.12. What is the percentage of profit
made by her?
2) 150%

3) 125%

am
sT

1) 100%

4) None of these

5. By selling some cloth at the cost price a merchant still gained 19 1/21%. How much less cloth does he measure for a
meter?
1) 15cm

2) 16cm

3) 20cm

4) None of these

6. 30% loss on cost price in what percent loss on selling price?


2) 20%

3) 15%

Ex

1) 30%

4) None of these

1) 3%

an
k

7. Arun purchased a house for Rs.75,000 and a site for Rs.15,000 respectively, if he sold the house for Rs.83,000 and the
site for Rs.10,000, then find the resultant percentage of gain?
2) 31/3%

3) 30%

4) 331/3%

w
w

1) Rs.222

.B

8. The manufacturing cost of a watch is Rs.180 and the transportation lost is Rs.500 for 100 watches. What will be the
selling price if it is sold at 20% gains
2) Rs.216

3) Rs.221

4) Rs.220

9. A person, by selling an article at three-fourths of the list price incurs a loss of 20%. Find the profit percentage if he sells
at the list price?

1) 25%

2) 6.66%

3) 111/9%

4) None of these

10. A sells an article to B at a gain of 20%. B sells is to C at a gain of 25% and C in turn sells is to D at a loss of 331/3%. If
D paid Rs.1,000 for it, then what is the cost price of A.
1) Rs.1,000

2) Rs.2,000

3) Rs.3,000

4) Rs.4,000

11. Ajay had purchased a second hand scooter for 18,000 and spent Rs.1,800 for repairs. After one year he wanted to sell
the scooter. At what price should he sell it to gain 11 1/9%, if 91/11% is to be deducted at the end of every year on account
of deprecation?
1) Rs.18,000

2) Rs.19,800

3) Rs.20,000

Get all banking study material on BankExamsToday.com

4) Rs.22,500
Page 55

www.BankExamsToday.com, best study material for bank exams


CRT
12. After getting three equal successive discount percentages over a marked price of Rs.1,000 a customer has to pay 729 for
an article. What is the rate of each of the successive discounts?
1) 10%

2) 20%

3) 30%

4) 40%

2) 162/3% loss

1) 20%gain

3) 142/7%gain

co
m

13. One-fifth of the cost price, one-seventh of the marked price and one-sixth of the selling price are all equal. What is the
gain or loss to the trader?
4) 10%loss

1) 777/9%

2) 75%

3) 50%

od
ay
.

14. Due to a slump in the market, A, while selling 12 apples to B, allows him to count them as 9. But due to an overnight
demand A is forced to buy them back at the same rate as he sold and allows B to count 9 apples as 12. What is overall
gain percentage of B
4) 662/3%

15. A trader offers to give two articles free for every 10 articles I purchase. I get a total of 10 articles free for my purchase
and I sell them all at a rate such that I get back my investment from the sale of just 10 of the articles. What is my overall
percentage of profit
2) 150%

3) 500%

am
sT

1) 100%

4) 250%

16. A mechanic purchases a cooler for Rs.32,000 and incurs Rs.13,000 on installation and repairs. After one year he sold it
for Rs.40,000. What is the profit or loss percentage, if the deprecation rate of the machine is 20% p.a?
1) 81/3%

2) 121/12%

3) 161/4%

4) 111/9%

2) 31

an
k

1) 30

Ex

17. Ramya bought a certain number of apples at 6 apples for Rs.10 and sold them at 4 apples for Rs.10. Find the number of
apples she bought if total gain is Rs.60
3) 62

4) None of these

18. 5kg of ghee was bought by Venu for Rs.300. One kg becomes spoilt. He sells the remaining in such a way that on the
whole he incurs a loss of 10%. At what price per kg does he sell the ghee?
2) Rs.45.70

.B

1) Rs.46.25

3) Rs.46.60

4) Rs.67.50

w
w

19. A trader professes to lose 10% in selling 2kgs of rice. He uses 2 weighing stones, each of which is marked 1kg but
weighs less. If the percentage of profit is 26/7% and one of the two stones weighs only 800 gm, how much does the
second stone weigh
2) 850gm

3) 900gm

4) 950gm

1) 800gm

20. A girl sold her pen for Rs.39 and got a percentage of profit numerically equal to the cost price. The cost price of that
pen is..
1) Rs.25

2) Rs.20

3) Rs.30

4) None of these

21. A person loses 10% on one investment but gains 20% on another. If the ratio of the investments is 3:4, what is the
percentage of gain or loss on the two investments taken together?
1) 61/8%

2) 71/7%

3) 111/9%

Get all banking study material on BankExamsToday.com

4) None of these

Page 56

www.BankExamsToday.com, best study material for bank exams


CRT
22. A trader professes to sell all articles at a loss of 25%, but sells three-fifth of them at again of 25% and the remaining at a
loss of 25%. What is his overall percentage of gain or loss
1) 5% loss

2) 10% gain

3) 5% gain

4) No loss, No gain

1) Rs.500

2) Rs.300

co
m

23. A man sells an article at a profit of 20%. If he had bought it at 10% less and sold it for Rs.18 more, he would have
gained 40%. Find the cost price of the article.
3) Rs.400

4) Rs.550

24. An article was sold at a profit of 20%. If both cost price and selling price are Rs.100 less each, then magnitude of the
percentage of profit would have been 4 percentage points more than that in the first case. Then the cost price is
2) Rs.600

3) Rs.800

4) None of these

od
ay
.

1) Rs.500

25. A man bought 2 articles at the same price and sells them together at 30% gain. Had he bought the first article at 20%
less and the second article at 10% more and then sold them together for Rs.48 less, he would have gained 20% on the
whole. What is the total cost of 2 articles?
2) Rs.300

3) Rs.400

am
sT

1) Rs.200

4) Rs.500

26. A trader marks up the price of the product by 40%. If the discount is increased from 15% to 25%, his profit comes
down by Rs.42. What is the cost price?
1) Rs.150

2) Rs.200

3) Rs.250

4) Rs.300

Ex

27. The catalogue price of an article is Rs.15,000. If the discount is increased from 15% to 20%, then profit falls from
27.5% to 20%. Find the cost price of the article?
1) Rs.12,000

2) Rs.10,000

3) Rs.12,250

4) Rs.12,750

2) Rs.150

3) Rs.200

4) Rs.175

.B

1) Rs.180

an
k

28. The marked price of an article is Rs.300. If the selling price is 50% more than the amount of discount allowed, find the
selling price

w
w

29. The cost of an apple is 331/3% less than the cost of 1 mango. If a man sells four apples at the cost price of 5 mangoes,
what is his percentage of profit?
1) 75%

2) 81%

3) 87.5%

4) 90%

30. A merchant professed to sell 20 articles at a loss which is equals to the cost price of 2 articles but sold 18 articles at the
cost price of 20 articles. What is the gain percent?
1) 191/11%

2) 10%

3) 111/9%

4) 0%

31. The percentage by which the marked price exceeds the cost price of an article and the percentage of discount allowed
on the article are in the ratio of 3:2. If it is sold at the cost price, what is the percentage of discount allowed?
1) 20%

2) 25%

3) 331/3%

4) 50%

32. The purchase prices of three articles are in the ratio 3:4:5 the first one is sold at a profit of 10% and the second at a loss
of 7.5%. If the overall percentage of profit or loss of the first two articles is the same as the percentage profit or loss of
all the articles taken together, what is the percentage of profit or loss in the case of the third article?
Get all banking study material on BankExamsToday.com

Page 57

www.BankExamsToday.com, best study material for bank exams


CRT
1) 8.75

2) 1.25

3) 0

4) Cant be determined.

33. A dishonest oil merchant claims that he gets a profit of only 5% but he gives only one litre of oil instead of 1kg. If 1.25
liter of oil weighs 1kg what is his overall percentage of profit?
2) 25%

3) 26%

4) None of these

co
m

1) 31.25%

34. A fruit vendor sells mangoes and bananas and gets equal revenue from each. He gets a profit of 20% on each mango
and a profit of 25% on each banana. If the ratio of the number of bananas sold to the number of mangoes sold is 4:1,
what is the ratio of the cost price of a banana to that of a mango?
2) 6:25

3) 2:9

4) Cant be determined.

od
ay
.

1) 1:5

35. A trader buys 150 pens for Rs.1,000 and he marks each of them at Rs.10. He gives a discount of 20% on each pen and
he gives 1 pen free on bulk purchases of 9 pens. What is his minimum possible overall percentage of profit?
1) 8%

2) 10%

3) 20%

4) 5%

1) 4:1

am
sT

36. A trader gives a discount on an article such that the profit as a percent of marked price is the same as the discount as a
percent of cost price. What is the ratio of the actual profit percentage to the actual discount percentage an the article?
2) 2:1

3) 1:2

4) Cant be determined.

1) 20%

2) 25%

Ex

37. The cost price of a computer is Rs.1,000 less then the selling price of a television and the selling price of the computer
is 30% more than the cost price of the television. If the selling price of the computer is 4% more than the selling price of
the television, what is the percentage of profit on selling the television?
3) 162/3%

4) Cant be determined.

an
k

38. The marked prices of two articles are in the ratio of 1:2, their discount percentages are also in the ratio of 1:2 and the
profit they get is also in the ratio of 1:2. What is the ratio of their cost price?
2) 5:8

3) 2:5

4) Cant be determined.

.B

1) 1:2

w
w

39. A trader purchases two watches. He marks the first one up by Rs.200 over the cost price and gives a discount of 20% on
it. The second one he marks up by 50% and gives a discount of Rs.160. If he gains 15% on both the watches put together
and 8% on the first alone, what is the percentage of profit on the second watch?
1) 21%

2) 22%

3) 18.5%

4) Cant be determined.

40. Javed sells 2,000 mangoes in a week. He recovers his total cost by selling first 1,200 Mangoes. He sells the next 300
Mangoes for a loss of 20% and he sells the last 500 Mangoes for a loss of 40%. What is his overall percentage of profit?
1) 45%

2) 35%

3) 27%

4) 12.5%

Answers
1.

Profit at a price = loss at other price => CP must be numerically between those prices
CP = (56+42)/2 = Rs. 49.

Get all banking study material on BankExamsToday.com

Page 58

www.BankExamsToday.com, best study material for bank exams


CRT
2.

Gain % = 1000/880 => 1. 1363 => 13.63%

3.

For P, SP=1080 and loss=10% => CP = 1080/0.9 =1200 => loss = 1200-1080 = 120.

4.

She got profit => profit % = 15/10 X 12/12 = 1.5 => 50%.

5.

Profit % = 19 1/21 => 1.19047.

co
m

For Q, SP=1800 and loss=10% => CP = 1800/0.9 = 2000 => loss = 2000-1800 = 200.

Let he measure x cm for 100 cm. Then, 100/x = 1.19047 => x=84 cm

6.

Loss = 30% on CP i.e., 0.3CP => SP = 0.7CP

od
ay
.

So he measures 16 cm less for every meter.

Loss % on SP = loss/SP X 100 = 0.3CP/0.7CP X 100 = 42.85%.

Total CP = 90000 & total SP = 93000 => gain = SP/CP = 93000/90000 =1.0333 = 3.33%

8.

Total cost of a watch = 180 + (500/100) = 185.

am
sT

7.

Gain = 20% => SP = 1.2CP = 1.2 X 185 = 222


9.

0.75 MP = 0.8 CP (since 20% loss)


So, MP = 1.0666CP => 6.66% gain

Ex

10. 1.2 X 1.25 X 0.6666 X CP = 1000 => CP = 1000 (profits of 20%, 25% & loss of 33.33%)

an
k

11. Total CP=18000+1800 = 19800.

Depreciation = 9.09% and gain = 11.11% => SP = (0.9091)X(1.1111)X19800 = 20000.


12. Let f be the factor of discount => 1000 X f X f X f = 729 => f = 0.9 => 10% decrease.

.B

13. CP/5 = SP/6 => SP/CP=1.2 => 20% gain.

w
w

14. In two transactions B is gaining => SP > CP for B in two transactions.


So, gain% = 12/9 X 12/9 = 1.7777 => 77.77%.

15. 2 articles free for every 10 articles bought. So 10 free articles => 50 articles bought.
Money of 60 articles (10 articles free) is obtained by selling only 10 articles.
So SP of 10 articles = CP of 60 articles => SP/CP = 6 => 500% gain.
16. Total CP = 45000. Depreciation = 20% =>new CP = 0.8 X 45000 = 36000.
SP = 40000 => SP/CP = 40/36 = 1.1111 => 11.11% gain
17. CP => 6 apples for Rs. 10.

SP => 4 apples for Rs. 10 => 6 apples for Rs. 15

So, for 6 apples, gain is Rs.5 => Rs. 60 gain requires 72 apples.
Get all banking study material on BankExamsToday.com

Page 59

www.BankExamsToday.com, best study material for bank exams


CRT
18. CP of 5 kg ghee = 300. Loss = 10% => SP = 0.9 CP = 270. For Rs. 270, 4 kg are sold
SP for 1 kg = 270/4 =Rs. 67.5
19. Let w be the weight of the second stone.
Now, 0.9 X (1000/800) X (1000/w) = 1.0285 (since profit is 2.85%)

co
m

w = 900 gm (nearly)
20. SP = 39. Profit % = CP.

21. ratio = 3:4 => investments are 3/7 and 4/7.


Overall loss/gain % = (3/7)(-10) + (4/7)(20) = 50/7 = 7 1/7 %.

od
ay
.

CP + (CP% of CP) = SP => CP = 30/-.

22. 3/5th are sold at gain of 25% and 2/5th are sold at loss of 25%.

am
sT

First, (3/5 X 25 2/5 X 25) = 5% gain.


23. 20% gain => SP = 1.2 CP.

New CP = 0.9 CP and New SP = SP + 18 => 1.2CP+18.

24. 20% gain => SP = 1.2CP.

Ex

40% gain => new SP = 1.4 X new CP => (1.2CP+18) = 1.4(0.9CP) => CP = 300.

an
k

New CP = CP 100 and new SP = SP -100 & 24% gain => new SP=1.4 X new CP
CP 100 = 1.4 ( 1.2CP 100) => CP = 600.
25. Let each article costs x => Total CP = 2x and SP = 1.3 X 2x = 2.6x.

.B

New total CP = 0.8x + 1.1x = 1.9x, New SP = SP 48 = 2.6x 48 and gain = 20%

w
w

So, 2.6x 48 = 1.2 X 1.9x => 2x = 300.


26. MP = 1.4CP. Also 10% change is discount => Rs. 42 gain => 10% of 1.4CP = 42

CP = 300.

27. MP = 15000. 5% change in discount i.e., 5% of MP = 7.5 % of CP (profit change)


So, CP = 5/7.5 X MP = 10000.
28. MP = 300. SP = 1.5 X discount. Now, SP = MP discount => SP = 180.
29. CP of apple = 0.6666 X CP of mango.1
Man sold 4 apples for CP of 5 mangoes => his CP = 4 X CP of apple
And his SP = 5 X CP of mango.

Get all banking study material on BankExamsToday.com

Page 60

www.BankExamsToday.com, best study material for bank exams


CRT
So, SP/CP = (5XCP of mango)/(4XCP of apple) = 1.875 => 87.5%.
30. SP of 18 articles = CP of 20 articles => SP/CP = 20/18 = 1.1111 => 11.11% gain
31. If CP is raised by 3x %, the discount should be 2x %.

From inspection, 33.33% discount => 50% increase (since 3:2) and 1.5 X 0.6666 = 1.
32. CP of first two articles are in ration of 3:4.

od
ay
.

So for 2 articles, gain/loss % = (3/7)X10 (4/7)X7.5 = 0.

co
m

Also, after discount SP=CP => increase of 3x% X decrease of 2x%.

So, overall profit/loss% = 0 => (3/12)X10 (4/12)X7.5 + (5/12)x = 0 => x=0%.


33. Overall profit = 1.05 X (1.25/1) = 1.3125 => 31.25% gain
34. For mango, SP = 1.2 CPm and for banana SP = 1.25 CPb.

So, CPb/CPm = 6:25.

am
sT

Revenue from mango = revenue from banana => 1.2 CPm = 4 X 1.25 CPb (since they are sold in ratio of 1:4)

35. 150 pens for Rs.1000 => total CP = 1000.

1 pen free for every 9 pens => he can sell 135 pens (for least possible profit)

Ex

SP of each pen = 10 and discount = 20% => SP = 8.

Total SP = 135 X 8 = 1080 => SP/CP = 1080/1000 = 1.08 => 8%.

an
k

36. Profit% of MP = discount% of CP => profit%/discount% cant be determined without the values of MP and CP.
37. CP computer = SP TV 1000 and SP computer = 1.3 X CP TV.

.B

SP Computer = 1.04 SP TV => 1.3 CP TV = 1.04 SP TV => SP/CP = 1.25 => 25% gain.

w
w

38. Without the knowledge of atleast on of the prices the ratio of CPs cant be determined.
39. MP1 = CP1 + 200 and discount = 20%.Also MP2 = 1.5CP2 and discount = Rs. 160.

Also SP1/CP1 = 8% gain. With this information it cant be said what is the profit % on 2 nd watch.

40. 300 sold at loss of 20% and 500 old at a loss of 40% => loss% = (3/8)X20 + (5/8)X40
= 32.5 => loss factor = 0.675
Already he got a gain by SP of 1200 = CP of 2000.
So overall profit % = (2000/1200) X 0.675 = 1.125 => 12.5% gain.

Get all banking study material on BankExamsToday.com

Page 61

www.BankExamsToday.com, best study material for bank exams


CRT
Exercise
1.

Find the gain % if the C.P. is Rs.750 and S.P. is Rs.1000.


1. 16

2
3

2. 25

3) 33

1
3

4. 27

1
2

2.

A watch is sold for Rs.1080 at a loss of 10%. The C.P. of the watch is ___
1. 1125
2. 180
3. 1188

3.

An article is sold at Rs.45 at a loss of 10%. If it is sold at Rs.65, the gain % is ___
1. 10
2. 15
3. 20
4. 30

4.

A bicycle is sold at 20% gain. If it had been sold at 20% loss, the S.P. would have been Rs.120 less. The C.P. of the
bicycle is ___
1. 500
2. 300
3. 250
4)200

5.

A dishonest shopkeeper uses false weight to give 20% less quantity of food. His profit % is ___
1. 80
2. 75
3. 25
4. 20

6.

A shopkeeper buys two types of tea one at Rs.700 for 10 kg and the other at Rs.770 for 10 kg. He mixed the two
types and the mixture is sold at Rs.840 per 10 kg. His profit % is ___
1. 14

2
7

2. 16

1
11

am
sT

od
ay
.

co
m

4)1200

3. 12

9
11

4) 14

1
7

A man lost 10% by selling oranges at the rate of 11 for a rupee. How many should he sell them to gain 10% at a
rupee?
1. 10
2. 9
3. 15
4. 12

8.

By selling 20 articles a man gains the S.P. of 5 articles. The gain % is ___
1. 20

2. 25

Ex

7.

3. 33

1
3

4. 16

2
3

A trader marks his goods at 20% above the C.P. and allows a discount of 10%. His gain % is __
1. 8
2. 10
3. 15
4. 12

10.

A shopkeeper marks his goods in such a way that after allowing a discount of 5%, he gains 14%. How much % above
the C.P. is marked price?
1. 18
2. 20
3. 15
4. 25

11.

By selling a radio for Rs.572, a shopkeeper earns a profit equivalent to 30% of the C.P. of the radio. The C.P. of the
radio is ___
1. 340
2. 400
3. 440
4. 404

.B

w
w

A man bought an article and sold it at a loss of 5%. If he had bought it for 10% less and sold it for Rs.15.30 more, he
would have made a profit of 15%. The C.P. of the article is ___
1. 320
2. 180
3. 260
4. 90

12.

an
k

9.

13.

A merchant makes a profit of 12% on his goods after giving 20% off on the printed price. He marked his goods by
what % above his C.P.?
1. 40
2. 30
3. 25
4. 35

14.

Suresh bought 18 kg of rice @ Rs.4.50 per kg and 12 kg @ Rs.3.50 per kg. He mixes the two and sold the mixture @
Rs.5 per kg. What was his total gain in this transaction?
1. 25
2. 26
3. 27
4. 28

Get all banking study material on BankExamsToday.com

Page 62

www.BankExamsToday.com, best study material for bank exams


CRT
15.

The profit obtained by selling an article for Rs.560 is same as the loss when the same article is sold for Rs.506. The
C.P. of the commodity is ___
1. 510
2. 530
3. 533
4. 515

16.

A merchant professes to lose 5% on a certain tea powder, but he uses a weight of 900 g instead of 1 kg. His real
gain% is ___
1. 5

2. 5

5
9

3. 6

4. 6

6
7

If the profit % is numerically equal to the C.P. in rupees and the S.P. is Rs.39, what is the C.P.?
1. 21
2. 23
3. 30
4. 25

18.

A bicycle when sold for Rs.954 gave a loss of 10%. To earn 10% profit, it should be sold for Rs.___
1. 1616
2. 1661
3. 1166
4. None

19.

The C.P. of four articles is same as the S.P. of three articles. What is the loss or gain %?
1. 20 % profit

2. 33

1
% profit
3

od
ay
.

co
m

17.

3. 25 % loss

4. 30 % profit

A man bought 11 mangoes at Rs.10 and sold 10 mangoes at Rs.11. Find the loss or gain %.
1. 21% gain
2. 20% gain
3. 25% gain
4. 31% loss

21.

A man sold 20 articles and got S.P. of 2 articles as the profit. What is his profit %?
1. 10

2. 12

1
3

am
sT

20.

3. 11

1
9

4. 10

1
9

Two successive discounts of 10% and 20% are equivalent to a single discount of ___
1. 28%
2. 30%
3. 25%
4. 32%

23.

Three successive discounts 10%, 10% and 5% are equivalent to a single discount of ___
1. 23.05%
2. 30%
3. 25%
4. 23.5%

24.

The marked price of an article is Rs.3600. What will be the S.P., if two successive discounts of 5% and 10% are
offered?
1. 3780
2. 3078
3. 3708
4. 3807

25.

The S.P. of an article is Rs.700 after offering two successive discounts of 12 % and 16 % . What is the marked

C.P.
1. 2200

3. 960

4. 980

1
9

2. 2400

3. 2020

4. 2600

At what price above the C.P. an article is marked so that it earns 12% after 8% discount?

27.

2
3

1
2

A man marks his goods 35% above the C.P. and allows 11 % discount. If he sells the goods for Rs.2640 find the

w
w

26.

2. 1000

.B

price?
1. 906

an
k

Ex

22.

1. 25

2. 12

3. 21.7

4. 16

2
3

28.

A trader offers 30% discount on the Marked Price of an article yet makes 10% profit. If he gains Rs.140 by selling
the article, then find the Marked Price.
1. 2200
2. 2300
3. 2400
4. 2000

29.

What profit % is made by selling an article at a certain price, if by selling at two-third of the price there would be a
loss of 20%.
1. 20
2. 30
3. 15
4. 25

Get all banking study material on BankExamsToday.com

Page 63

www.BankExamsToday.com, best study material for bank exams


CRT
A man sells two horses at Rs.990 each and gains 10% on one and loses 10% on the other. Find his gain or loss%.
1. 1 % loss
2. 1 % gain
3. No loss no gain
4. 2% loss

31.

A dealer sold two articles for Rs.396 each, gaining 20% on one and losing 20% on the other. What is his gain or
loss%?
1. 4 % loss
2. 4 % gain
3. No loss no gain
4. 40% loss

32.

The C.P. of 15 pens was recovered by the sale of 12 pens only. Gain % is ___
1. 25

2. 20

3. 16

1
3

co
m

30.

4. 9

2
3

A tradesman by means of a false balance defrauds to the extent of 10% in buying goods and also defrauds to 10% in
selling. Find his gain %.
1. 20
2. 21
3. 22
4. 18

34.

By selling an article for Rs.450, a man loses 10%. Find his gain or loss % if he sells it for Rs.540.
1. 8 % loss

od
ay
.

33.

2. 8 % gain

3. 3% gain

4. 9

1
% gain
3

A trader marks his goods at 30% above the C.P. but constrained to give a discount of 30% to clear his stock. What is
gain or loss%?
1. 9% loss
2. 9% gain
3. No loss no gain
4. 3% gain

36.

Oranges are bought at 2 for a Re.1 and sold at 5 for Rs.3. What will be the gain or loss% in the transaction?
1. 25% loss
2. 25% gain
3. 20% gain
4. 20% loss

37.

A dealer sold an article at a loss of 6.25%. If he had purchased it for 10% lesser price and sold it for Rs.42 more, he
would have gained 10%. His original purchase price is ___
1. 750
2. 800
3. 840
4. 880

38.

A dealer purchases 22 pencils for Rs.20 and sells them @ 10 pencils for Rs.11. His profit % is __________
1. 10
2. 20
3. 21
4. 22

39.

A reduction of 20% enables a person to buy 5 more oranges for Rs.10. Find the price of each orange before
reduction.
1. 0.5 p
2. 0.75 p
3. Rs.1
4)0.40 p

40.

A man bought a watch at

an
k

Ex

am
sT

35.

.B

w
w

1. 15

Answers

1) 2
2) 4
3) 4
4) 2
5) 3
6) 1
7) 2
8) 3
9) 1
10) 2

19
20

th

of its C.P. and sold at 14% above its C.P. What is his gain?

2. 16

11) 3
12) 2
13) 1
14) 3
15) 3
16) 2
17) 3
18) 3
19) 2
20) 1

Get all banking study material on BankExamsToday.com

3. 17.5

21) 3
22) 1
23) 1
24) 2
25) 3
26) 1
27) 3
28) 1
29) 1
30) 1

4. 20

31) 1
32) 1
33) 3
34) 2
35) 1
36) 3
37) 2
38) 3
39) 1
40) 4

Page 64

www.BankExamsToday.com, best study material for bank exams


CRT
Averages
Average: The average of a number of quantities of the same kind is equal to their sum divided by the number of
those quantities. It is also called mean or arithmetic mean.
For example: The average of 1, 3, 5, 7 is

1 3 5
4

16
=4
4

co
m

Defenation2:
The concept of average is equal distribution of the overall value among all the things or persons present there. So the
formula for finding the average is as follows:
Average, A = Total of all things, T / Number of things, N
Basic Formulae:
Average =

2.

Sum of quantities = Average

4.

5.

Number of quantities

sum of quantities
Number of quantities =
Average

If the number of quantities in two groups be a 1 and a 2 and their average is x and y, respectively, then the
a x a2 y
combined average (average of all of them put together) is given by 1
.
a1 a2
If the average of a 1 quantities is x and the average of a 2 quantities out of them is y, the average of

am
sT

3.

remaining group (rest of the quantities) is given by

x + a.

The average of x 1 + a, x 2 + a,., x n + a is

b.

The average of x 1 - a, x 2 - a,., x n - a is

c.

The average of ax 1 , ax 2 ,.., ax n is ax , where a 0.

d.

The average of

x - a.

an
k

Ex

a.

x1 x2
x
x
,
,.........., n is
.
a a
a
a

The average of n quantities is equal to x. If one of the given quantities whose value is p, is replaced by a
new quantity having value q, the average becomes y, then q = p + n (y x).
The average of n quantities is equal to x. If a quantity is removed, the average becomes y. The value of the
removed quantity is n (x y) + y.

8.

a2 y
.
a2

x is the average of x 1 , x2 ,.., x n , then

.B

7.

If

a1x
a1

w
w

6.

od
ay
.

Sum of quantities
Num ber of quantities

1.

9.

The average of n quantities is equal to x. If a quantity is added, the average becomes y. The value of the
new quantity is n (y x) + y.

10. The average of first n natural numbers is

11. The average of squares of natural numbers till n is


12. The average of cubes of natural numbers till n is

Get all banking study material on BankExamsToday.com

(n 1)(2n 1)
.
6

n(n 1)2
.
4

Page 65

www.BankExamsToday.com, best study material for bank exams


CRT
13. The average of odd numbers from 1 to n is

(last oddnumber+ 1)

14. The average of even numbers from 1 to n is

(last evennumber+ 2)

2
15. If n is odd: The average of n consecutive numbers, consecutive even numbers or consecutive odd numbers

is always the middle number.

16. If n is even: The average of n consecutive numbers, consecutive even numbers or consecutive odd numbe rs

co
m

is always the average of the middle two numbers.

17. The average of first n consecutive even numbers is (n + 1).


18. The average of first n consecutive odd numbers is n.

1)(2n 1)
.
3
(n 1)(n 2)
.
20. The average of squares of consecutive even numbers till n is
3
n(n 2)
21. The average of squares of consecutive odd numbers till n is
.
3
22. If the average of n consecutive numbers is m, then the difference between the smallest and t he largest

2(n

od
ay
.

19. The average of squares of first n consecutive even numbers is

number is 2 (n 1).

am
sT

23. Geometric Mean or Geometric Average: It is useful in calculating averages of ratios such as average

population growth rate, average percentage increase and so on.


Geometric mean of x 1 , x 2 ,.., x n is denoted by
G.M. = n x1.x2...........xn

24. Harmonic Mean or Harmonic Average: It is useful for finding average speed of a vehicle, average

1 1
n x1

1
1
..........
x2
xn

an
k

H.M. =

Ex

production per day and so on.

25. If a certain distance is covered at a speed of x km/h and the same distance is covered at a speed of y km/h,

the average speed during the whole journey is

2xy
x y

km/h.

.B

26. If a person or a motor car covers three equal distances at the speed of x km/h, y km/h, z km/h, respectively,

then for the entire journey average speed of the person or motor car is

xy

3xyz
yz zx

km/h.

w
w

27. If a person covers A km at a speed of x km/h, B km at a speed of y km/h and C km at a speed of z km/h,

then the average speed during the whole journey is

28. If a person covers A

th

A
A
x

B
B
y

C
C
z

km/h.

part of the distance at x km/h, B th part of the distance at y km/h and the C th part at z

km/h, then the average speed during the whole journey is

Get all banking study material on BankExamsToday.com

A
x

1
B
y

C
z

km/h.

Page 66

www.BankExamsToday.com, best study material for bank exams


CRT
Level 1 examples
Example:
Consider three people A, B and C with total of Rs. 30/-. Their average becomes Rs. 10/- for each. If another person D joins
them with Rs. 50/- then he has Rs. 40/- more than actual average of Rs. 10/-.

co
m

So this Rs. 40/- will get distributed among those four and each gets Rs. 10/-. Thus the average becomes Rs. 20/- each.
Example:

Soln:

od
ay
.

The average age of a class of 30 students is 12. If the teacher is also included the average becomes 13 years. Find the
teachers age.

When the teacher is included there are totally 31 members in the class and the average is increased by 1 year. This means
that everyone got 1 extra year after distributing the extra years of the teacher.

am
sT

So extra years of the teacher are as follow: 31x1=31 years.


Age of the teacher = actual avg + extra years = 12 + 31 = 43 years.

Level 2 Examples

Sunil purchased 4 toys at the rate of Rs.100 each, 6 toys at the rate of Rs.150 each and 8 toys at the rate of
200 each. What is the average cost of one toy?
= 100 x 4 = Rs.400
= 150 x 6 = Rs.900
= 200 x 8 = Rs.1600
= 4 + 6 + 8 = 18

an
k

Sol: Cost of 4 toys


Cost of 6 toys
Cost of 8 toys
Total number of toys

Ex

1.

Average price of 1 toy =

The average marks obtained by 100 students in a competitive examination is 50. Find the total marks.

w
w

2.

.B

= Rs.161.11

Sol: Total marks

= Average marks x Number of students


= 100 x 50 = 5000.

The average weight of 40 students of section A of I -CET class is 55 kg and that of 50 students of section B is
60 kg. Find the average weight of all the 90 students of th e class.

3.

400 + 900 + 1600


18

Sol: Here, a 1 = 40, a 2 = 50 and x = 55, y = 60


Average weight

a1 x
a1

a2 y
a2

(40)(55)
40

(50)(60)
50

= 57.78 kg.

Get all banking study material on BankExamsToday.com

Page 67

www.BankExamsToday.com, best study material for bank exams


CRT
Average salary of all the 30 employees including 5 officers of a company is Rs.7500. If t he average salary of
the officers is Rs.15000, find the average salary of the remaining staff of the company.
Sol: Here, a 1 = 30, a 2 = 5 and x = 7500, y = 15000
Average salary of the remaining staff

a1 x
a1

=
=

a2 y
a2

30(750) 5(1500)
30 5

= 6000.

The average value of five numbers 7, 10, 16, 24 and 28 is 17. If 6 is added to each number, what will be the
new average?
Sol: The new average =

6.

The average of x numbers is 4x. If x 3 is subtracted from each given number, what will be the new average?
Sol: The new average =

7.

x + a = 17 + 6 = 23.

od
ay
.

5.

co
m

4.

x - a = 4x (x 3) = 3x + 3.

The average of 8 numbers is 20. If each of the numbers is multiplied by 8, find the average of a new set of
numbers.

am
sT

Sol: The average of a new set of numbers = a x = 8 x 20 = 160.

The average weight of 20 persons is increased by 3 kg when one of them whose weight is 50 kg, is replaced
by a new person. What is the weight of the new person?
Sol: The weight of the new person, q = p + n (y x).
= 50 + 20(3)
= 50 + 60
= 110 kg.

9.

The average age of 24 students and the Maths teacher is 16 years. If the Maths teachers age is excluded, the
average age reduces by 1 year. What is the age of the Maths teacher?

an
k

Ex

8.

.B

Sol: The age of Maths teacher

= n ( x - y) + y
= 25 (16 15) + 15)
= 40 years.

10. Find the average of first 79 natural numbers.

w
w

Sol: The required average

1
2
79 1
=
= 40.
2

11. Find the average of squares of the natural numbers from 1 to 47.

Sol: The required average

(n 1)(2n 1)
6
(47 1) 2(47) 1
=
6
48 95
=
= 760.
6

Get all banking study material on BankExamsToday.com

Page 68

www.BankExamsToday.com, best study material for bank exams


CRT
12. Find the average of cubes of the natural numbers from 1 to 15.

Sol: The required average

n(n 1)2
4

15(15 1)2
4
15 16 16
=
= 960.
4

co
m

13. Find the average of odd numbers from 1 to 50.

la s to d dnum b e r 1
2
49 1
=
= 25.
2

14. Find the average of even numbers from 1 to 61.

la s te ve nnum b e r 2
2
60 2
=
= 31.
2

am
sT

Sol: The required average

od
ay
.

Sol: The required average

15. Find the average of 5 consecutive numbers 4, 5, 6, 7, 8.

Sol: The required average = middle number = 6.

Ex

16. Find the average of consecutive odd numbers 21, 23, 25, 27, 29, 31.

Sol: The required average

an
k

= average of middle two numbers


25 27
= 26.
=
2
17. Find the average of first 25 consecutive even numbers.
Sol: The required average = (n + 1) = 25 + 1 = 26.

.B

18. Find the average of first 30 consecutive odd numbers.

w
w

Sol: The required average = n = 30.


19. Find the average of squares of first 16 consecutive even numbers.

Sol: The required average

1)(2n 1)
3
2(16 1) 2(16) 1
=
3
2 17 33
=
= 374.
3

2(n

20. Find the average of squares of consecutive even numbers from 1 to 35.

Sol: The required average

1)(n 2)
3
(34 + 1)(34 + 2)
=
3

(n

Get all banking study material on BankExamsToday.com

Page 69

www.BankExamsToday.com, best study material for bank exams


CRT
35 36
= 420
3

co
m

21. Find the average of squares of consecutive odd numbers from 1 to 44.
n(n + 2)
Sol: The required average
=
3
43(43 + 2)
=
3
43 45
=
= 645
3
22. If the average of 6 consecutive numbers is 48, then find the difference between the smallest and the largest

number?

od
ay
.

Sol: The required difference = 2 (n 1) = 2 (6 1) = 10.

23. The production of a company for three successive years has increased by 10%, 20% and 40% respectively.

Find the average annual increase of production.

Sol: Average annual increase = G.M. of x, y and z = (x y z) 3 %


1

am
sT

= (10 20 40) 3 % = 20%.

24. The population of a city in two successive years increases at the rates 12% and 3% respectively. Find the

average increase of two years.

Sol: Average population increase = G.M. of x and y


1

= (x

y) 2 %

Ex

an
k

= (12 3) 2 %
= 6%.

25. A man runs 2 km at 20 km/ph and another 2 km at 10 km/ph. Find the average speed for the whole distance in

covering 4 km.

2xy
x+y

.B

Sol: Required formula =

w
w

So, Average speed

and here, x = 20 and y = 10


=
=

2(20) (10)
20 10

km/ph

1
40
= 13 km/ph.
3
3

26. A train covers the first 150 km at a speed of 100 km/ph, another 150 km at a speed of 120 km/ph and the last

150 km at 80 km/ph. Find the average speed of the train for the entire journey.
Sol: Required Average speed =

3xyz
xy yz zx

km/ph

3(100)(120)(80)
(100 120) (120 80) (100 80)

2880000
29600

km/ph

km/ph

= 97.3 km/ph (app)

Get all banking study material on BankExamsToday.com

Page 70

www.BankExamsToday.com, best study material for bank exams


CRT
27. A person covers 8 km at a speed of 4 km/ph, 15 km at a speed of 5 km/ph and 20 km at a speed of 10 km/ph.

Find the average speed of the whole journey.

=
=

=
28. A person covers the first

A B C
A B C
x y z

km/ph.

8 + 15 + 20
8 15 20
+
+
4
5
10

15
2 3

43
7

km/ph.

20
km/ph.
2

co
m

od
ay
.

Sol: The required average speed

1
km/ph.
7

1 th
3 th
of the distance at 5 km/ph, the next
at 8 km/ph and the remaining distance
4
5

at 10 km/ph. Find the average speed during the entire journey.


1
A B C
+ +
x y z

am
sT

Sol: The required average speed

1
1 4 3 5 3 20
+
+
5
8
10

Ex

km/ph

1
1
3
3
+
+
20 40 200

w
w

.B

an
k

1
10 + 15 + 3
200

km/ph

km/ph

km/ph

1
200
km/ph = 7 km/ph.
7
28

29. A train covers 50% of the journey at 25 km/ph, 25% of the journey at 15 km/ph and the remaining at 10

km/ph. Find the average speed during the entire journey.


Sol: The required average speed

A
x

50
25

Get all banking study material on BankExamsToday.com

100
B C
y
z

km/ph

100
25 25
15 10

km/ph

Page 71

www.BankExamsToday.com, best study material for bank exams


CRT
=

Exercise

12

100
10 15
6

km/ph

600
8
= 16
km/ph.
37
37

The average marks obtained by 144 candidates in a certain examination is 55. Find the total marks?
1. 7290
2. 7920
3. 7930
4. 7390

2.

Ram travels half of a journey at the speed of 24km/hr and the next half at a speed of 16km/hr. What is the average
speed of the Ram during the whole journey?

2
km/hr
23

2. 26

A person runs the first

1
5

th

1
km/hr
23

1
km/hr
5

4. 16

1
km/hr
5

1. 2.02 km/hr

3. 25

2
km/hr
23

4. 25

1
km/hr
23

of the distance at 2km/hr, the next one half at 3km/hr and the remaining distance at

1km/hr. Find the average speed.


2.

11
km/hr
5

3.

5
km/hr
11

4. None

A man covers first 20% of the distance at 10km/hr, next 50% at 5km/hr and the remaining at 20km/hr. Find the
average speed of the train during the whole journey?
1. 25

25
km/hr
47

2. 25

5
km/hr
47

3. 25

Ex

5.

3. 19

A person divides his total route of journey into three equal parts and decides to travel the three parts with speeds of
20, 25 and 40km/hr respectively. Find his average speed during the whole journey?
1. 26

4.

2. 20km/hr

am
sT

3.

2
km/hr
5

od
ay
.

1. 19

co
m

1.

23
km/hr
47

4. None of these

The average salary of the entire staff in a office is Rs.220 per day. The average salary of officers is Rs.650 and that of
non-officers is Rs.170. If the number of officers is 25, then find the number of non-officers in the office.
1. 215
2. 315
3. 250
4. 350

7.

The average age of all the students of a class is 22 years. The average age of boys of the class is 26 years and that of
the girls is 19 years. If the number of girls in the class is 16, then find the number of boys in the class.
1. 12
2. 10
3. 6
4. 8

8.

There were 45 students in a hostel. If the number of students increases by 9, the expense of the mess increase by
Rs.25 per day while the average expenditure per head diminishes by Rs.1. Find the original expenditure of the mess.
1. Rs.390
2. Rs.295
3. Rs.395
4. Rs.400

.B

w
w

There were 42 students in a hostel. If the number of students increases by 7, the expenses of the mess increase by
Rs.32.5 per day while the average expenditure per head diminishes by Rs.1.5. Find the original expenditure of the
mess.
1. Rs.636
2. Rs.536
3. Rs.630
4. Rs.656

9.

an
k

6.

10.

The average weight of 20 balls is 2gm. If the weight of the bag be included the average weight increases by 0.02gm.
What is the weight of the bag?
1. 2.04gm
2. 2.42gm
3. 3.42gm
4. 3.04gm

11.

Find the average of first 14 consecutive even numbers.


1. 15
2. 14
3. 210

Get all banking study material on BankExamsToday.com

4. 28

Page 72

www.BankExamsToday.com, best study material for bank exams


CRT
Find the average of first 17 consecutive odd numbers.
1. 18
2. 289
3. 17

4. 34

13.

Find the average of squares of first 23 consecutive even numbers.


1. 750
2. 754
3. 725

4. 752

14.

Find the average of squares of consecutive even numbers from 1 to 22.


1. 184
2. 174
3. 182

4. 186

15.

Find the average of squares of consecutive even numbers from 1 to 35.


1. 484
2. 445
3. 408

4. 420

16.

Find the average of squares of consecutive even numbers from 1 to 62.


1. 1280
2. 1281
3. 1821

4. 1218

17.

The average weight of a group of 20 boys was calculated to be 89.4kg and it was later discovered that one weight
was misread as 78kg instead of the correct one of 87kg. The correct average weight is ____________
1. 88.95kg
2. 89.25kg
3. 89.55kg
4. 89.85kg

18.

The average weight of 15 students was calculated to be 52kg and it was later discovered that one weight was misread
as 21kg instead of the correct one of 12kg. The correct average weight is __________
1. 51.4kg
2. 50.6kg
3. 52.4kg
4. 51.6kg

19.

The average of the first and the second of three numbers is 10 more than the average of the second and the third of
these numbers. What is the difference between the first and the third of these three numbers?
1. 40
2. 10
3. 20
4. None

20.

The average of the first and the second of three numbers is 13 more than the average of the second and the third of
these numbers. What is the difference between the first and the third of these three numbers?
1. 25
2. 24
3. 26
4. 19

21.

The average marks scored by in English, Science, Mathematics and History is less than 15 from that scored by him in
English, History, Geography and Mathematics. What is the difference of marks is Science and Geography score by
him?
1. 40
2. 50
3. 60
4. Data inadequate

22.

The average temperature for Monday, Tuesday and Wednesday was 40 oC. The average for Tuesday, Wednesday and
Thursday was 41oC and that of Thursday being 45oC. What was the temperature on Monday?
1. 48oC
2. 41oC
3. 46oC
4. 42oC

od
ay
.

am
sT

Ex

an
k

.B

w
w

If average of 6 consecutive even numbers is 48, what is the difference between the smallest and the largest numbers?
1. 10
2. 12
3. 9
4. Data inadequate

23.

co
m

12.

24.

The average attendance of a college for the first three days of a week is 325, and for first four days it is 320. How
many were present on the fourth day?
1. 305
2. 350
3. 530
4. 503

25.

A car runs for t1 hours at v1 km/hr, t2 hours at v2 km/hr. What is the average speed of the car for the entire journey?
t1 t 2
v t
v2t2
1.
km/hr
2. 1 1
km/hr
v1 t 1 v 2 t 2
t1 t 2
3.

v1 t 2
v1

v 2 t1
km/hr
v2

Get all banking study material on BankExamsToday.com

4.

v1
v1 t 2

v2
km/hr
v 2 t1

Page 73

www.BankExamsToday.com, best study material for bank exams


CRT
The average age of a family of 6 members is 22 years. If the age of the youngest member be 7 years, the average age
of the family at the birth of the youngest member, was _____________
1. 15 years
2. 17 years
3. 17.5 years
4. 18 years

27.

5 years ago, the average of A, B, C and D was 45. With E joining them now, the average age of all the five is 49
years. How old is E?
1. 25 years
2. 40 years
3. 45 years
4. 60 years

28.

The average height of 40 students is 163cm. On a particular day, three students A, B, C were absent and the average
of the remaining 37 students was found to be 162cm. If A, B have equal heights and the height of C be 2cm less than
that of A, find the height of A?
1. 176cm
2. 166cm
3. 180cm
4. 186cm

29.

Of the three numbers, second is twice the first and is also thrice the third. If the average of the three numbers is 44,
the largest number is ____________
1. 24
2. 36
3. 72
4. 108

30.

The average weight of 3 men A, B and C is 84kg. Another man D joins the group and the average now becomes
80kg. If another man E, whose weight is 3kg more than that of D, replaces A, then average weight of B, C, D and E
becomes 79kg. The weight of A is ____
1. 70kg
2. 72kg
3. 75kg
4. 80kg

31.

A cricketer has completed 14 innings and his average is 30 runs. How many runs must he make in his next innings so
as to raise his average to 32?
1. 60
2. 55
3. 65
4. 50

32.

The average of marks obtained by 102 candidates in a certain examination is 18. If the average marks of passed
candidates is 21 and that of the failed candidates is 15, what is the number of candidates who failed the examination?
1. 51
2. 52
3. 61
4. 50

33.

The average of marks obtained by 65 candidates in a certain examination is 25. If the average marks of passed
candidates is 27 and that of the failed candidates is 14, what is the number of candidates who passed the
examination?
1. 55
2. 65
3. 60
4. 75

34.

The average of marks obtained by 115 candidates in a certain examination is 36. If the average marks of passed
candidates is 40 and that of the failed candidates is 17, what is the number of candidates who failed the examination?
1. 30
2. 25
3. 20
4. 34

35.

On a journey across Mumbai, a taxi averages 30km/hr for 60% of the distance, 20km/hr for 20% of it and 10km/hr
for the remainder. The average speed for the whole journey (in km/hr) is _______________
1. 20km/hr
2. 22.5km/hr
3. 25km/hr
4. 24.635km/hr

od
ay
.

am
sT

Ex

an
k

.B

w
w

The average of marks obtained by 108 candidates in a certain examination is 20. If the average marks of passed
candidates is 28 and that of the failed candidates is 16, what is the number of candidates who failed the examination?
1. 70
2. 78
3. 81
4. 72

36.

co
m

26.

37.

In a class there are 16 students whose average age is decreased by 3 months, when 2 students aged 24 and 26 years
respectively are replaced by the same number of students. Find the average age of the new students.
1. 23 years
2. 21 years
3. 18 years
4. None of these

38.

The average age of 44 boys in a class is 26 years. If the teachers age is included the average age of the boys and the
teacher becomes 27 years. What is the teachers age?
1. 69 years
2. 71 years
3. 59 years
4. Data inadequate

Get all banking study material on BankExamsToday.com

Page 74

www.BankExamsToday.com, best study material for bank exams


CRT
39.

The average age of 30 students in a class is 12 years. The average age of a group of 5 students is 10 years and that of
another group of 5 students is 14 years. Find the average age of the remaining students.
1. 14 years
2. 10 years
3. 12 years
4. Data inadequate

40.

The average age of 15 students and the class teacher is 15 years. If the class teachers age is excluded, the average
reduces by 1 year. What is the age of the class teacher?
1. 30 years
2. 31 years
3. 29 years
4. 28 years

21) 3
22) 4
23) 1
24) 1
25) 2
26) 4
27) 3
28) 1
29) 3
30) 3

31) 1
32) 1
33) 1
34) 3
35) 1
36) 4
37) 1
38) 2
39) 3
40) 3

od
ay
.

11) 1
12) 3
13) 4
14) 1
15) 4
16) 2
17) 4
18) 1
19) 3
20) 3

w
w

.B

an
k

Ex

am
sT

1) 2
2) 3
3) 1
4) 4
5) 4
6) 1
7) 1
8) 3
9) 1
10) 2

co
m

Answers

Get all banking study material on BankExamsToday.com

Page 75

www.BankExamsToday.com, best study material for bank exams


CRT
Ratios and Proportions
What is a ratio?
A ratio is a representation of distribution of a value present among the persons present and is shown as follows:
Or
A ratio is a comparison of two quantities by division.
a
or a b. It is denoted by a : b.
b

co
m

If a and b are two numbers, then the ratio of a to b is

Note:
1.
The two quantities in a : b that are being compared are called terms.
2.
The first term is called antecedent and the second term is called consequent.
4
with antecedent 4 and consequent 5.
5

od
ay
.

Ex: The ratio 4:5 represents


3.

A ratio is a number, so to find the ratio of two quantities; they must be expressed in the same units.

4.

A ratio does not change if both of its terms are multiplied or divided by the same number. Thus,

am
sT

TYPES OF RATIOS:

8
12
4
=
=
etc.
5
10
15

1.
2.

Duplicate Ratio: If a : b is the given ratio then its duplicate ratio is a 2 : b2.
Triplicate Ratio: If a : b is the given ratio then its triplicate ratio is a 3 : b3.

3.

Sub-duplicate Ratio: If a : b is the given ratio then its sub-duplicate ratio is

4.
5.
6.

Sub-triplicate Ratio: If a : b is the given ratio then its sub-triplicate ratio is 3 a : 3 b .


Inverse Ratio or Reciprocal Ratio: If a : b is the given ratio, then its inverse or reciprocal ratio is b : a.
Compound Ratio: If a : b and c : d are two given ratios, then ac : bd is the compound ratio of the given ratios.

b.

Ex

a :

an
k

Proportion: The equality of two ratios is called proportion.


Note:

a
c
= , then a, b, c and d are said to be in proportion.
d
b

1.

If

2.

We write it as a : b :: c : d and read as a is to b as c is to d.

3.

Each term of

4.

In the a : b :: c : d proportion, a, d are known as extremes and b, c are known as means.

5.

If four quantities are in proportion, then

w
w

.B

a
c
and
is called a proportional.
d
b

Product of Means = Product of Extremes

Ex:

3
6
= , we write as 3:2 :: 6:4 and say 3, 2, 6 and 4 are in proportion.
2
4

Here, 3 and 4 are extremes and their product = 3 x 4 = 12


2 and 6 are means and their product = 2 x 6 = 12

6.

Fourth Proportional: If a : b :: c : x, then x is called the fourth proportional of a, b, c.


We have,

a
c
= , or x
b
x

Thus, fourth proportional of a, b, c is

c
a

Get all banking study material on BankExamsToday.com

Page 76

www.BankExamsToday.com, best study material for bank exams


CRT
Third Proportional: If a : b :: b : x, then x is called the third proportional of a, b.
8.
Mean Proportional: If a : b is the given ratio, then the mean proportional of a and b is
Formulae:
7.

a
c
= , then
d
b

a)

Componendo Rule:

b) Dividendo Rule:
c)
2.

b
b

a
b

d
d

c
d

Componendo and Dividendo Rule:

a b
c
=
c
a b

d
d

If two numbers are in the ratio of a : b and the sum of these numbers is x, then these numbers will be
bx
respectively.
a b

mixture will be
4.

ax
bx
litres and
litres respectively.
a b
a b

If three numbers are in the ratio a : b : c and the sum of these numbers is x, then these numbers will be
bx
cx
and
respectively.
a b c
a b c

5.

Ex

The ratio between two numbers is a : b. If x is added to each of these numbers, the ratio becomes c : d, then the

an
k

bx(c d)
ax(c d)
and
.
ad bc
ad bc

The ratio between two numbers is a : b. If x is subtracted from each of these numbers, the ratio becomes c : d, then
the two numbers are

bx(d c)
ax(d c)
and
.
ad bc
ad bc

If the ratio of two numbers is a : b, then the number that should be added to each of the numbers in order to make

.B

8.

this ratio c : d is given by

ad bc
.
c d

If the ratio of two numbers is a : b, then the number that should be subtracted from each of the numbers in order to

w
w

9.

ax
,
b c

ax
bx
and
respectively, (where a > b).
a b
a b
ax
bx
b)
and
respectively, (where b > a).
b a
b a

two numbers are


7.

If two numbers are in the ratio a : b and the difference of these numbers is x, then these numbers will be
a)

6.

ax
and
a b

If in a mixture of x litres two liquids A and B are in the ratio of a : b, then the quantities of liquids A and B in the

am
sT

3.

co
m

If

od
ay
.

1.

ab .

make this ratio c : d is given by

bc
c

ad
.
d

10. There are four numbers a, b, c and d. The number that should be subtracted from each of the numbers so that the

remaining numbers may be proportional is given by

(a

ad bc
.
d) (b c)

11. There are four numbers a, b, c and d. The number that should be added to each of the numbers so that the new
numbers may be proportional is given by

(a

bc
d)

ad
.
(b c)

12. The incomes of two persons are in the ratio of a : b and their expenditures are in the ratio of c : d. If the savings of
each person be Rs.X, then their
aX(d c)
bX(d c)
and Rs.
and their
ad bc
ad bc
cX(b a)
dX(b a)
Expenditures are Rs.
and Rs.
.
ad bc
ad bc

Incomes are Rs.

Get all banking study material on BankExamsToday.com

Page 77

www.BankExamsToday.com, best study material for bank exams


CRT
13. If in a mixture of x litres of two liquids A and B, the ratio of liquids A and B is a : b, then the quantity of liquid B
x(ad bc)
.
c(a b)

to be added in order to make this ratio c : d is

14. In a mixture of two liquids A and B, the ratio of liquids A and B is a : b. If on adding x litres of liquid B to the
mixture, the ratio of A to B becomes a : c, then in the beginning the quantity of liquid A in the mixture was
bx
litres.
c b

co
m

and that of liquid B was

ax
c b

15. When two ingredients A and B of quantities q1 and q2 and cost price per unit c 1 and c2 are mixed to get a
mixture c having cost price per unit cm , then
c
c
q1
= 2 m and
cm c1
q2

c1

b) c m
16. If a : b

q1
q1

c2
q2

q2

n1 : d1 and b : c

n2 : d2 , then

n1 : d1 and b : c
a:b : c : d

(n1

n2 ) : (d1

n2 : d 2 and c : d
(n1

n2 ) : (d1

am
sT

a:b : c

17. If a : b

od
ay
.

a)

n2

d2 )

n3 : d 3 , then

n3 ) : (d1

n2

n3 ) : (d1

d2

n3 ) :(d1

d2

d3 )

18. If a mixture contains two ingredients A and B in the ratio a : b, then


Percentage of A in the mixture =

100% .

Ex

a)

an
k

b) Percentage of B in the mixture =

100% .

19. If two mixtures M1 and M2 contain ingredients A and B in the ratios a : b and c : d respectively, then a third
mixture M3 obtained by mixing M1 and M2 in the ratio x : y will contain:
c
y

cy

bx
a b
x

dy
c d
y

.B

100% ingredient A, and

w
w

a)

ax
a b
x

b)

100

ax
a b
x

100% ingredient B or

cy
c
y

100% ingredient B

If a total is divided among A, B and C such that A got 4 parts, B got 5 parts and C got 6 parts then it is represented in ratio
as A:B:C = 4:5:6.
So, 4:5:6 means that the total value is divided into 4+5+6 = 15 equal parts and then distributed as per the ratio.
Get all banking study material on BankExamsToday.com

Page 78

www.BankExamsToday.com, best study material for bank exams


CRT
Level 1 Examples:

Ex

Let the numbers be a and b.


A:B = 3:4 => A / B = 3 / 4.
Also, (A+8) / (B+8) = 5 / 6.

am
sT

od
ay
.

co
m

Example 1:
Divide Rs. 580 between A and B in the ratio of 14:15.
Soln:
A:B = 14:15 => 580 is divided into 29 equal parts => each part = Rs. 20.
So As share = 14 parts = 14 x 20 = Rs. 280
Bs share = 15 parts = Rs. 300.
Example 2:
If A:B = 2:3 and B:C = 4:5 then find A:B:C.
Soln:
To combine two ratios the proportions common for them shall be in equal parts. Here the common proportion is B for the
given ratios.
Making B equal in both ratios they become 8:12 and 12:15 => A:B:C = 8:12:15. Example 3:
Three numbers are in the ratio of 3: 4 : 8 and the sum of these numbers is 975. Find the three numbers.
Soln:
Let the numbers be 3x, 4x and 8x. Then their sum = 3x+4x+8x = 15x = 975 => x = 65.
So the numbers are 3x = 195, 4x = 260 and 8x = 520.
Example 4:
Two numbers are in the ratio of 4 : 5. If the difference between these numbers is 24, then find the numbers.
Soln:
Let the numbers be 4x and 5x. Their difference = 5x 4x = x = 24 (given).
So the numbers are 4x = 96 and 5x = 120.
Example 5:
Given two numbers are in the ratio of 3 : 4. If 8 is added to each of them, their ratio is changed to 5 : 6. Find two numbers.
Soln:

w
w

.B

an
k

Solving we get, A=12 and B = 16


Example 6:
A garrison has provisions for 120 soldiers for 240 days. After 180 days 60 more soldiers will join the group. For how many
more days will the provisions last?
Soln:
Actually after 180 days,
If 120 members are there provisions come for 60 more days (since total 240 days)
But now 180 members are there.
So number of days = (120/180) X 60 = 40 days.
Example 7:
If 24 men working for 12 hrs a day can do a work in 16 days, in how many days can 8 men working 6 hrs a day do it? Soln:
24 men 12 hrs 16 days
8 men 6 hrs - ? days (n)
n =16 X (12 / 6) X (24 / 8) ( since no of hrs reduced no of days has to increase and no of men reduced also increases no of
days i.e., inverse proportional)
=> n = 96 days.

Get all banking study material on BankExamsToday.com

Page 79

www.BankExamsToday.com, best study material for bank exams


CRT
Level-2

3.
4.
5.
6.

7.

od
ay
.

2.

The duplicate ratio of 4 : 5 is ______.


Sol: Duplicate ratio of 4 : 5 is 42 : 52 = 16 : 25.
The triplicate ratio of 2 : 3 is _______.
Sol: Triplicate ratio of 2 : 3 is 23 : 33 = 8 : 27.
The sub-duplicate ratio of 9 : 16 is ________.
Sol: Sub-duplicate ratio of 9 : 16 = 9 : 16 = 3 : 4.
The sub-triplicate ratio of 8 : 27 is _________.
Sol: Sub-triplicate ratio of 8 : 27 is 3 8 : 3 27 = 2 : 3.
The inverse ratio of 4 : 5 is _________.
Sol: Inverse ratio of 4 : 5 is 5 : 4.
The compound ratio of 3 : 4, 4 : 5 and 5 : 6 is _________.
Sol: Compound ratio of
3 : 4, 4 : 5 and 5 : 6 = (3 x 4 x 5) : (4 x 5 x 6)
= 60 : 120 = 1 : 2.
Find a fourth proportional to 2, 5, 6.
Sol: Let x be the fourth proportional to 2, 5, 6.
Then, 2 : 5 :: 6 : x or

8.

2
6
=
5
x

So, x = 15.
Find a fourth proportional to 5, 4.

am
sT

1.

co
m

EXAMPLES

Then 5 : 4 :: 4 : x or

5
4

Ex

Sol: Let x be the third proportional to 5, 4.


4
x

9.

16
5

3.2 .

an
k

Find the mean proportional between 32 and 8.

.B

Sol: The mean proportion between 32 and 8


= 32 8

256

16 .

w
w

10. Two numbers are in the ratio 3 : 4 and the sum of these numbers is 21. Find the two
numbers.
Sol: Here, a = 3, b = 4 and x = 21

3 21
ax
=
=9
a b
3 4
4 21
bx
The second number =
=
= 12
a b
3 4

The first number =

(or)
= 21 9 = 12.
11. Two numbers are in the ratio 3 : 5 and the difference of these numbers is 36. Find the two
numbers.
Sol: Here, a = 3, b = 5 and x = 36

Get all banking study material on BankExamsToday.com

Page 80

www.BankExamsToday.com, best study material for bank exams


CRT
3 36
ax
=
= 54
b a
5 3
5 36
bx
=
= 90
The second number =
b a
5 3

The first number =

(or)
= 54 + 36 = 90.

3,n2

a:b : c

5, d1

(n1

4, d2

n2 ) : (d1

n2 ) : (d1

d2 )

(3 5) : (4 5) : (4 6)
= 15: 20 : 24 .

od
ay
.

Sol: Here, n1

co
m

12. If a : b = 3 : 4 and b : c = 5 : 6, find a : b : c.

13. Given two numbers which are in the ratio of 2 : 3. If 6 is added to each of them, their ratio
is changed to 4 : 5. Find the two numbers.
Sol: Here, a : b = 2 : 3 and c : d = 4 : 5 and x = 6

ax(c d)
2(6)(4 5)
=
ad bc
2(5) 3(4)
12
6
=
2
bx(c d)
3(6)(4 5)
The second number =
=
ad bc
2(5) 3(4)

18
2

9.

am
sT

The first number

Ex

14. The ratio of two numbers is 5: 9 If each number is decreased by 5, the ratio becomes 5 : 11.
Find the numbers.

an
k

Sol: Here, a : b = 5 : 9 and c : d = 5 : 11 and x = 5


The first number

ax(d c)
5(5)(11 5)
=
ad bc
5(11) 9(5)

25 6
15
10
bx(d c)
9(5)(11 5)
=
=
ad bc
5(11) 9(5)
45 6
27 .
=
10

.B

w
w

The second number

15. Find the number that must be subtracted from the terms of the ratio 5 : 6 to make it equal to
2 : 3.
Sol: Here, a : b = 5 : 6 and c : d = 2 : 3
Required number

bc ad
c d
6(2) 5(3)
=
2 3

3.

16. Find the number that must be added from the terms of the ratio 11 : 29 to make it equal to
11 : 20.
Sol: Here, a : b = 11 : 29 and c : d = 11 : 20
Required number =

ad bc 11(20) 29(11)
=
c d
11 20

Get all banking study material on BankExamsToday.com

11 .

Page 81

www.BankExamsToday.com, best study material for bank exams


CRT
17. Find the number subtracted from each of the numbers 54, 71, 75 and 99 leaves the
remainders which are proportional.
Sol: Here, a = 54, b = 71, c = 75 and d = 99
Required number =

(a

54(99)
(54 99)

ad bc
d) (b c)

71(75)
(71 75)

3.

co
m

18. Annual income of Shashi and Ravi is in the ratio of 4 : 3 and the ratio of their an nual
expenses is 3 : 2. If each of them saves Rs. 500 at the end of year, then find their annual
income.
Sol: Here, a : b = 4 : 3 and c : d = 3 : 2 and X = 500
=

= Rs.2000

od
ay
.

aX(d c)
ad bc
4(500)(2 3)
=
4(2) 3(3)

So, Annual income of Shashi

2 00 0
1

bX(d c)
ad bc
3(500)(2 3)
=
4(2) 3(3)

am
sT

Annual income of Ravi

1500
1

= Rs.1500.

Ex

19. The incomes of Madhav and Kiran are in the ratio 9 : 4 and their expenditures are in the
ratio 5 : 2. If each saves Rs.1000, find their expenditures.
Sol: Here, a : b = 9 : 4 and c : d = 5 : 2 and X = 1000
=

cX(b a)
ad bc
5(1000)(4 5)
=
9(2) 4(5)

an
k

So, the expenditure of Madhav

5000
2

= Rs.2500

w
w

.B

Expenditure of Kiran

dX(b a)
ad bc
2(1000)(4 5)
=
9(2) 4(5)

2000
2

= Rs.1000.

20. 630 ml of a mixture contains milk and water in the ratio 7 : 2. How much more water is to
be added to get a new mixture containing milk and water in t he ratio of 7 : 3.
Sol: Here, a : b = 7 : 2 and c : d = 7 : 3 and x = 630
Required quantity of water to be added =

x(ad bc)
c(a b)

630 7(3) 2(7)


7(7 2)
14)

630(21
=
63

Get all banking study material on BankExamsToday.com

= 70 ml

Page 82

www.BankExamsToday.com, best study material for bank exams


CRT
21. A mixture contains milk and water in the ratio 8 : 3. On adding 6 litres of water, the ratio
of milk to water becomes 8 : 5. Find the quantity of water in the mixture.
Sol: Here, a : b = 8 : 3 and a : c = 8 : 5 and x = 6
So, the quantity of water in the mixture =

bx
c b

3(6)
5 3

9 litres.

10 , c 2

15 , cm

q1
q2

Required ratio,

12.2

c2 cm
cm c1

15 12.2
12.2 10

od
ay
.

Sol: Here, c1

co
m

22. In what ratio the two kinds of tea powder (Ex: general tea powder with lams a) must be
mixed together into one at Rs.10 per kg and another at Rs.15 per kg, so that the mixture ma y
cost Rs.12.2 per kg?

2.8
2.2

1.4 : 1.1

Thus, the two kinds of tea powder are mixed in the ratio 1.4 : 1.1.
23. In a mixture of two types of liquids L1 and L2 , the ratio L1 :L2 is 4 : 3. If the cost of liquid L1 is Rs.5 per litre and that of L2

Sol: Here, q1

4, q2

3, c1

5, c2

am
sT

then find the cost per litre of the resulting mixture.


10

c1

q1
q1

an
k

Ex

So, the cost of the resulting mixture, c m

c2
q2

=
=

c1

q2

q1
q1

c2
q2

5 4 10 3
4 3

q2
50
7

= Rs.7.14 (app).

.B

24. If a mixture contains water and alcohol in the ratio 3 : 7, what is the percentage quantity of
alcohol in the mixture?
Sol: Water (a) : alcohol (b) = 3 : 7

w
w

Percentage of alcohol in the mixture =

b
a

100%

7
3 7

100%

= 70%.
Note: Similarly you can find the percentage of water.
25. Two alloys contain gold and copper in the ratio 3 : 2 and 5 : 2. In what ratio the two alloys
should be added together to get a new alloy having gold and copper in the ratio of 2 : 1.
Sol: Here, a : b = 3 : 2 and c : d = 5 : 2
Let the two alloys be mixed in the ratio x : y.
Quantities of Gold and Copper should be added in the ratio

Get all banking study material on BankExamsToday.com

Page 83

www.BankExamsToday.com, best study material for bank exams


CRT

3x 5y
+
5
7
2x 2y
+
5
7

21x + 25y
14x + 10y
21x + 25y
14x + 10y

2
1

21x+25y = 28x+20y
7x = 5y
x
y

5
7

Exercise

am
sT

Hence, the two alloys should be mixed in the ratio 5 : 7.

od
ay
.

Since, the ratio of gold and copper in the new alloys is 2 : 1

co
m

ax
cy
+
a+b c + d
bx
dy
+
a+b
c+d

1.

Find the ratio of 6 hours to a day.


1. 1 : 4
2. 4 : 12

2.

Find the ratio of 2.5 and 2.05.


1. 41 : 50
2. 50 : 41

3.

Two quantities are in the ratio 3 : 5. If first quantity is 150 kg then find the other quantity.
1. 240 kg
2. 150 kg
3. 250 kg
4. 200 kg

4.

The ratio between two quantities is x : 21. If the original quantities are 200 m and 840 m. Find the value of x.
1. 5
2. 4
3. 3
4. 6

5.

The antecedent of a ratio which is equal to 7 : 9 is 70. Find its consequent.


1. 80
2. 90
3. 75

Ex

4. 1 : 3

.B

an
k

4. 21 : 23

The compound ratio of 2 : 3 and the inverse ratio of 7 : 9 is 54 : x. Find x.


1. 63
2. 48
3. 54

4. 60
4. 60

Two horse riders travel 150 km and 200 km taking 5 hours each. Find the ratio of their speeds.
1. 4 : 3
2. 5 : 4
3. 3 : 4
4. 4 : 5

7.

3. 15 : 14

w
w

6.

3. 2 : 3

8.

If A : B = 2 : 3 and B : C = 4 : 5, then find A : B : C.


1. 12 : 8 : 15
2. 8 : 12 : 15
3. 15 : 8 : 12

4. 12 : 15 : 8

9.

If 2A = 4B = 5C then find A : B : C.
1. 2 : 4 : 5
2. 10 : 4 : 5

3. 10 : 5 : 4

4. 5 : 4 : 3

10.

If

3. 2 : 4 : 3

4. 2 : 5 : 4

a b
c
= =
then find a : b : c.
2 3 4

1. 3 : 4 : 2

2. 2 : 3 : 4

Get all banking study material on BankExamsToday.com

Page 84

www.BankExamsToday.com, best study material for bank exams


CRT
11.

The three numbers are in the ratio


Find the numbers.
1. 72, 84, 108

1 2 3
: : . The difference between the greatest and the smallest numbers is 36.
2 3 4

2. 60, 72, 96

3. 72, 84, 96

4. 72, 96, 108

3. 4 : 5

4. 5 : 4

If (2x + 5y) : (3x + 5y) = 9 : 10 then find x : y.


1. 5 : 7
2. 7 : 5

13.

What least number must be subtracted from each term of the ratio 6 : 11 to make it equal to 4:9.
1. 1
2. 2
3. 3
4. 5

14.

The monthly salaries of two persons A and B are in the ratio 4 : 5. If each salary is decreased by Rs.500 then the ratio
becomes 3 : 4. Find the salary of A.
1. Rs.3000
2. Rs.2500
3. Rs.2000
4. Rs.2400

15.

Find the first part if Rs.1024 is divided in the ratio of 7 : 9.


1. Rs.540
2. Rs.576
3. Rs.448

16.

In a mixture of 28 litres, the ratio of milk and water is 5 : 2, if 2 litres of water is added to the mixture, find the ratio
of milk and water in the new mixture.
1. 2 : 3
2. 1 : 2
3. 2 : 1
4. 3 : 2

17.

The incomes of A and B are in the ratio 4 : 5 and the incomes of B and C are in the ratio 2 : 3. If Cs income is
Rs.1500 then find As income.
1. Rs.750
2. Rs.650
3. Rs.400
4. Rs.800

18.

Two numbers are in the ratio 3 : 5. If 10 is subtracted from each of them then the ratio becomes 1 : 5. Find the greater
number.
1. 24
2. 20
3. 25
4. 16

19.

Find the third proportion to 24 and 36.


1. 68
2. 60

3. 54

4. 48

20.

Find x, if 1.68 : 2.52 = 1.46 : x.


1. 1.19
2. 3.19

3. 2.11

4. 2.19

21.

What least number must be subtracted from each of 13, 16, 17, 21 to make them in proportion.
1. 2
2. 1
3. 0
4. 3

od
ay
.

am
sT

Ex

an
k

.B

20 pumps can empty a reservoir in 12 hours. In how many hours can 45 such pumps do the same work?
1. 4 1/3 hours
2. 5 1/3 hours
3. 3 1/3 hours
4. 6 1/3 hours
75 men can dig a canal in 90 days. How many more men are required to dig the canal in 18 days?
1. 250
2. 375
3. 200
4. 300

23.

4. Rs.60

w
w

22.

co
m

12.

24.

In a 40 kg of mixture of sugar and sand, the ratio of sugar to sand is 5 to 3. How many kg of sugar are to be added
into the mixture so that the ratio becomes 3 : 1?
1. 14 kg
2. 18 kg
3. 20 kg
4. 25 kg

25.

A contractor under took to finish a certain work in 124 days and employed 180 men. After 64 days he found that 2/3
of the work completed. How many men can he discharge now so that the work may finish in time?
1. 88
2. 84
3. 60
4. 52

Get all banking study material on BankExamsToday.com

Page 85

www.BankExamsToday.com, best study material for bank exams


CRT
The sum of the cubes of three numbers is 584 and the ratio of first two numbers and last two numbers is 1 : 2. Find
the middle number.
1. 2
2. 3
3. 4
4. 5

27.

If 3 chairs cost as much as 5 benches, 8 benches cost as much as 4 tables, 6 tables cost as much as 4 cots. If one cot
cost is Rs.3600 then find the cost of one chair.
1. Rs.2000
2. Rs.1500
3. Rs.2400
4. Rs.2500

28.

A bag contains Re.1, 50 p, 25 p and 10 p coins in the ratio of 3 : 5 : 7 : 8 and together worth of Rs.80.50. Find the
value of 25 p coins.
1. Rs.16.50
2. Rs.14.50
3. Rs.17.25
4. Rs.18.50

29.

Rs.3630 has been divided among A, B and C in such a way that if their share be diminished by Rs.5, Rs.10 and Rs.15
respectively, the remainders are in the ratio 3 : 4 : 5. Find the share of A.
1. Rs.900
2. Rs.805
3. Rs.905
4. Rs.1005

30.

A sum of Rs.7000 is divided among A, B and C in such a way that shares of A and B are in the ratio 2 : 3 and those
of B and C are in the ratio 4 : 5. Find the amount received by C.
1. Rs.2600
2. Rs.2800
3. Rs.3000
4. Rs.3900

31.

Men, women and children are employed to do a work in the proportion of 1 : 2: 3 and their wages are in the ratio 6 :
3: 2. When 50 men are employed, total wages of all the hands amount to Rs.450, find the wages paid to a man, a
women and a child?
1. Rs.3, Rs.1.50, Rs.1
2. Rs.1.50, Rs.2, Rs.3
3. Rs.4, Rs.3, Rs.1
4. None

32.

Rs.4250 is divided among 4 men, 5 women and 6 boys so that the share of a man, a woman and a boy are in the ratio
9 : 8 : 4. What is the share of a boy?
1. Rs.150
2. Rs.170
3. Rs.200
4. Rs.270

33.

A man divides his property so that his sons share to his wifes and wifes share to his daughter are both in the ratio 3
: 1. If the daughter gets Rs.10,000 less than the son, then find the total worth of the property.
1. Rs.16,000
2. Rs.15,000
3. Rs.16,250
4. Rs.17,000

34.

Pavan is younger than Murali by 10 years. If 5 years back their ages were in the ratio 1 : 2, how old is Murali?
1. 20
2. 15
3. 25
4. Data Inadequate

35.

If 40% of a number is added to another number then it becomes 125% of itself. What will be the ratio of first and
second number?
1. 8 : 5
2. 5 : 7
3. 5 : 8
4. None

od
ay
.

am
sT

Ex

an
k

.B

w
w

The ratio of present ages of Sunil and Sunny is 7 : 8 respectively. Four years hence this ratio become 9 : 10
respectively. What is Sunils present age in years?
1. 14
2. 17
3. 18
4. Data Inadequate

36.

co
m

26.

37.

Three numbers are in the ratio 1 : 2 : 3 and their product is 20250. The numbers are _______
1. 15, 30, 45
2. 25, 50, 75
3. 35, 70, 105
4. 45, 90, 135

38.

A varies inversely with B. When B is 64 units, A is 36 units. Find A when B is 48 units.


1. 54 units
2. 94 units
3. 48 units
4. 72 units

39.

If

2pa 3qb
4
a
p
7
=
and
=
then find
.
q
pa+ qb
3
b
5

1. 116:31

2. 19:31

Get all banking study material on BankExamsToday.com

3. 11:43

4. 18:35

Page 86

www.BankExamsToday.com, best study material for bank exams


CRT
40.

If x and y have the same sign and 4 (8x2 + xy)= 15 y2 then find Y:X = ____
1. 5:8
2. 8:5
3. 11:43

4. 8:3

Answers
31) 4
32) 2
33) 3
34) 3
35) 4
36) 1
37) 1
38) 3
39) 3
40) 2

co
m

21) 2
22) 2
23) 2
24) 3
25) 2
26) 3
27) 1
28) 3
29) 3
30) 3

od
ay
.

11) 4
12) 1
13) 2
14) 3
15) 3
16) 3
17) 4
18) 2
19) 3
20) 4

w
w

.B

an
k

Ex

am
sT

1) 1
2) 2
3) 3
4) 1
5) 2
6) 1
7) 3
8) 2
9) 3
10) 2

Get all banking study material on BankExamsToday.com

Page 87

www.BankExamsToday.com, best study material for bank exams


CRT
Time and Distance

Speed:

We have the relation between speed, time and distance as follows:


Speed = distance / time.
So the distance covered in unit time is called speed.

co
m

This forms the basis for Time and Distance. It can be re-written as Distance = Speed X Time or
Time = Distance / Speed.

The units of speed are kmph (km per hour) or m / s.


1 kmph = 5 / 18 m / s
1 m / s = 18 / 5 kmph

am
sT

Average Speed:

od
ay
.

Units of Speed:

When the travel comprises of various speeds then the concept of average speed is to be applied.
Average Speed = Total distance covered / Total time of travel

Note1: In the total time above, the time of rest is not considered.

Ex

Note2:

The terms, time and distance are related to the speed of a moving object.

2.

If the time taken is constant, the distance travelled is proportional to the speed i.e. more the speed; more
the distance travelled in the same time.

3.

If the speed is constant, the distance travelled is proportional to the time taken i.e. more the distance
travelled; more the time taken at the same speed.

4.

If the distance travelled is constant, the speed is inversely proportional to the time taken i.e. more the
speed; less the time taken for the same distance travelled.

w
w

.B

an
k

1.

Formulae:

Speed =

1.

Distance
Tim e

2.

Distance = Speed x Time

3.

Time =

4.

x km/hr = x

15
8

m/sec

5.

x m/sec = x

8
15

km/hr

Distance
Speed

Get all banking study material on BankExamsToday.com

Page 88

www.BankExamsToday.com, best study material for bank exams


CRT
If A covers a distance d 1 km at a speed s 1 km/hr and then d 2 km at s 2 km/hr, then the average speed during
the whole journey is
Average speed =

If A goes from X to Y at s 1 km/hr and comes back from Y to X at s 2 km/hr, then the average speed during
the whole journey is

8.

A goes from X to Y at s 1 km/hr and returns back from Y to X at s 2 km/hr. If he takes T hours in all, the
distance between A and B is T

9.

s1s2
km .
s1 s2

A and B start at the same time from two points P and Q towards each other and after crossing they take T 1
and T 2 hours in reaching Q and P respectively, then speed
A' s speed
B' s speed

10.

2s1s2
km/hr
s1 s2

T2
T1

a
of the original speed, then the change in time taken to cover the same distance is
b

If the new speed is


given by

Change in time =
11.

co
m

Average speed =

od
ay
.

7.

s1s2 (d1 d2 )
km/hr
s1d2 s2 d1

am
sT

6.

b
a

originaltime

A body covers a distance d in time T 1 with speed s 1 , but when it travels with speed s 2 covers the same
distance in time T 2 then
s1
T2

s2
T1

Ex

Product of speed
d

Differenceof speed
Differenceof tim e

Note: By equating any two of the above, we can find the unknowns as per the given question.
A train travels a certain distance at a speed of s 1 km/hr without stopping and it covers the same distance at
a speed of s 2 km/hr with stopping then

an
k

12.

The stopping time per hour =

14.

s2
s1

hr.

.B

If a train overtakes a pole or a man or a milestone, then the distance covered in overtaking = Length of the
train.
If a train overtakes a bridge or tunnel or a platform or another train, then the distance covered = Sum of
the two lengths.

Relative Speed: If two trains of lengths L 1 km and L 2 km, respectively are traveling in the same direction
at s 1 km/hr and s 2 km/hr respectively such that s 1 > s 2 , then their
a. Relative speed = s 1 s 2 .

15.

s1

w
w

13.

Differenceof speed
speedwithout stoppings

b.
16.

Time taken by the faster train to cross the slower train =

L1
s1

L2
hr.
s2

Relative Speed: If two trains of lengths L 1 km and L 2 km, respectively are traveling in the opposite
direction at s 1 km/hr and s 2 km/hr respectively then their
a.

Relative speed = s 1 + s 2 .

b.

Time taken by the faster train to cross each other =

Get all banking study material on BankExamsToday.com

L1
s1

L2
hr.
s2

Page 89

www.BankExamsToday.com, best study material for bank exams


CRT

18.

Two trains of lengths L 1 m and L 2 m run on parallel tracks. When running in the same direction, the faster
train passes the slower one in T 1 seconds, but when they are running in opposite directions with the same
speeds as earlier, they pass each other in T 2 seconds then
a.

Speed of the faster train =

b.

Speed of the slower train =

L1

L2
2

L1

L2
2

1
T1

1
T2

1
T1

m/s.
1
T2

m/s.

A train starts from a place at s 1 km/hr and another fast train starts from the same place after T hours at s 2
km/hr in the same direction. Then,

co
m

17.

a) The distance from the starting place at which both the trains will meet is given by

19.

s1 T
s2 s1

hr.

The distance between two stations A and B is d km. A train starts from A to B at s 1 km/hr. T hours later
another train starts from B to A at s 2 km/hr. Then,

am
sT

a) The distance from the A at which both the trains will meet is given by s1
b) The time after which the two trains will meet is given by

d s 2T
s1 s 2

d s2 T
s1 s2

km.

hr.

Two trains start simultaneously from the stations A and B towards each othe r each other with speed s 1
km/hr and s 2 km/hr, respectively. When they meet it is found that the second train had travelled d km more
than the first. Then the

Ex

20.

km.

od
ay
.

b) The time after which the two trains will meet is given by

s1 s2 T
s2 s1

s1
s2

s2
km.
s1

an
k

Distance between two trains = d

Level 1 Examples

.B

Example 1:

If a car travels along four sides of a square at 100 kmph, 200 kmph, 300 kmph and 400 kmph find its average speed.

w
w

Soln:
Average Speed = Total distance / Total time.
Let each side of square be x km. Then the total distance = 4x km.
The total time is sum of individual times taken to cover each side.
To cover x km at 100 kmph, time = x / 100.
For the second side time = x / 200.
Using this we can write average speed = 4x / (x/100 + x/200 + x/300 + x/400) = 192 kmph.
Example 2:
A man if travels at 5/6 th of his actual speed takes 10 min more to travel a distance. Find his usual time.
Soln:
Let s be the actual speed and t be the actual time of the man.
Now the speed is (5/6)s and time is (t+10) min. But the distance remains the same.
So distance 1 = distance 2 => s X t = (5/6)s X (t+10) => t = 50 min.
Get all banking study material on BankExamsToday.com

Page 90

www.BankExamsToday.com, best study material for bank exams


CRT
Example 3:
If a person walks at 30 kmph he is 10 min late to his office. If he travels at 40 kmph then he reaches to his office 5 min
early. Find the distance to his office.
Soln:

Now if d km are covered at 30 kmph then time = d/30. Similarly second time = d/40.
So, d/30 d/40 = 1 / 4 => d = 30 km.

When two objects move with speeds s1 and s2


a.
b.

In opposite directions their combined speed = s1 + s2


In same direction their combined speed = s1 ~ s2.

Example 4:

od
ay
.

Note:

co
m

Let the distance to his office be d. The difference between the two timings is given as 15 min = 1 / 4 hr.

am
sT

Two people start moving from the same point at the same time at 30 kmph and 40 kmph in opposite directions. Find the
distance between them after 3 hrs.
Soln:

Ex

Speed = 30 + 40 = 70 kmph (since in opposite directions)


Time = 3 hrs

Example 5:

an
k

So distance = speed X time = 70 X 3 = 210 km.

Soln:

.B

A starts from X to Y at 6 am at 40 kmph and at the same time B starts from Y to X at 50 kmph. When will they meet if X
and Y are 360 km apart?

w
w

Distance = 360 km
Speed = 40 + 50 = 90 kmph.
Time = distance / speed = 360 / 90 = 4hrs from 6 am => 10 am.

Example 6:

A starts from X to Y at 6 am at a speed of 50 kmph. After two hours B starts from Y to X at 60 kmph. When will they meet
if X and Y are 430 km apart?
Soln:
By the time B started A traveled for 2 hrs => 2 X 50 = 100 km.
So at 8 am, distance = 430 100 = 330 km
Speed = 50 + 60 = 110 kmph.
Time = distance / speed = 330 / 110 = 3 hrs from 8 am => 11 am.
Get all banking study material on BankExamsToday.com

Page 91

www.BankExamsToday.com, best study material for bank exams


CRT
Note:
When a train crosses a negligible length object (man / pole / tree) the distance that it has to travel is its own length.
When a train has to cross a lengthy object (train / bridge / platform) the distance it has to travel is the sum of its length and
the length of the object.

co
m

Example 7:
If a train traveling at 40 kmph crosses another train of length 100m traveling at 14 kmph in opposite direction in 30 s find
the length of the train.

Let length of train be d.


Distance to be covered = d + 100.
Speed = 40 + 14 = 54 kmph = 54 X 5 / 18 = 15 m / s
Time = 30 s.
Distance = speed X time => d+100 = 15 X 30 => d = 350 m.

am
sT

Note:

od
ay
.

Soln:

If a man rows a boat along the stream flowing at speed S2 then it is termed downstream speed and is given by
S down = S1 + S2 , where S1 is speed of boat in still water.

S up = S1 S2.

1.

Find the speed of a train which covers a distance of 160 km in 4 hours.


Sol: Speed =

Distance
Time

160
4

40km/hr.

How long does a train 200 m long running at the rate of 80 km/hr take to cross a telegraphic pole?

.B

2.

an
k

Level 2 Examples

Ex

If a man rows a boat opposite to the stream flowing at S2 then it is termed upstream and is given by

w
w

Sol: We know, in crossing the telegraphic pole, the train must travel its own length.
So, Distance travelled = 200 m
80 1000
200
m/sec
m/sec
60 60
9
200
So, time taken to cross the pole =
9 seconds.
200
9

Speed = 80 km/hr =

3.

A train running at a speed of 72 km/hr passes a pole on the platform in 15 seconds. Find the length of
the train in metres.
Sol: Speed of the train = 72 km/hr
= 72
Length of the train

4.

5
18

20m /s e c

= Speed of the train x time taken


= 20 x 15 = 300 m.

A ship sails to Vizag at a speed of 10 knots/hr and sails back to the s ame point at the rate of 15 knots/hr.
Find the average speed for the whole journey.

Get all banking study material on BankExamsToday.com

Page 92

www.BankExamsToday.com, best study material for bank exams


CRT
Sol: Here s 1 = 10 and s 2 = 15
Average speed

2s1s2
km/hr
s1 s2

2 10 15
10 15

12k no ts /hr.

Sheela started to a bakery with the speed of 5 km an hour and returns with a speed of 3 km/hr. if she
takes 8 hours in all, find the distance in km between the bakery and her house.

co
m

5.

Sol: Here s 1 = 5, s 2 = 3 and T = 8


s1s 2
s1 s 2

od
ay
.

The distance between the bakery and her house = T

5 3
5 3

= 8
6.

15km .

Sujay starts his journey from Bombay to Kolkata and simultaneously Niteesh starts from Kolkata to
Bombay. After crossing each other they finish their remaining journey in 6

1
hours and 4 hours
4

Sol:

Sujay's speed
Niteesh's speed

So, Niteeshs speed =

5
4

4
1
6
4

16
25

25
4

Sujays speed

5
4

40

4
5

50km/hr.

A bike during its journey travels 20 min at a speed of 15 km/hr, another 25 min at 30 km/hr and another
30 min at 15 km/hr. Find the average speed of the bike.

Ex

7.

T2
T1

am
sT

respectively. What is Niteeshs speed if Sujays speed is 40 km/hr?

w
w

.B

an
k

20
25
30
, T2 =
, T3 =
, s 1 = 15, s 2 = 30 and s 3 = 15
60
60
60
s T s T
s T
So, Average speed of the bike = 1 1 2 2 3 3
T1 T2 T3

Sol:T 1 =

15

15

20
25
30
30
15
60
60
60
20 25 30
60 60 60
20
25
30
30
15
60
60
60
20 25 30
60 60 60

300 750 450


75

Get all banking study material on BankExamsToday.com

1500
75

20km/hr.

Page 93

www.BankExamsToday.com, best study material for bank exams


CRT
8.

By walking at

3
of her usual speed, Rani is 5 minutes late to the conference. Find her usual time to
4

cover the distance.

b
a

We have, change in time =

3
4

original time =

originaltime

change in tim e
b
1
a

5
4
3

Two scooterists do the same journey by traveling at the rates of 9 km/hr and 8 km/hr respectively. Find
the length of the journey when one takes 20 minutes longer than the other.
Sol: Here, change in speed = 9 8 = 1
Product of speed = 9 x 8 = 72
Length i.e. distance, d =?
We have,

20
60

am
sT

Difference of time = 20 min =

Product of speed Differenceof speed


d
Differenceof time
72

20
60

72
d

1
20
60

Ex

24km.

Without stopping, a train travels certain distance with an average speed of 100 km/hr and with
stoppages; it covers the same distance with an average speed of 80 km/hr. How many minutes per hour
the strain stops?

an
k

10.

15 m inutes.
1

od
ay
.

9.

a
b

co
m

Sol: Here, change in time = 5 and

Sol: Here, s 1 = 100 and s 2 = 60


s2

.B

s1

11.

s1

w
w

Stoppage time/hr =

1
hr
5

12min.

A train 500 m long crosses a pole in 6 seconds. Find the speed of the train in km/hr?

Sol: Speed of the train =

12.

100 80
100

Lengthof the train


tim e takingin crossingthe pole

500 18
6
5

500
m /s
6

300 km/hr.

A train 150 m long passes a bridge in 24 seconds moving with a speed of 54 km/hr. Find the length of
the bridge.
Sol: Speed of the train =

Lengthof the train Lengthof the bridge


tim e takingin crossingthe bridge

Get all banking study material on BankExamsToday.com

Page 94

www.BankExamsToday.com, best study material for bank exams


CRT
5
150 Lengthof the bridge
54
18
24
150 Lengthof the bridge 24 15
Lengthof the bridge 360 150 210 m.

A train 120 m long is running with a speed of 68 km/hr. In what time wi ll it pass a man walking at 4
km/hr in the opposite direction to that of train?

co
m

13.

Sol: Here, L 1 = 120, L 2 = 0, s 1 = 68, s 2 = 4


L 1 + L 2 = 120 + 0 = 120 m
5
18

Required time =
14.

L1
s1

L2
s2

120
20

20 m /sec.

6 sec.

od
ay
.

s 1 + s 2 = 68 + 4 = 72 km/hr = 72

Two trains of length 120 m and 80 m running on parallel tracks in the same direction with a speed of 48
km/hr and 50 km/hr respectively. In what time will they pass each other?

5
m /sec.
18

s 1 - s 2 = 50 - 48 = 2 km/hr = 2
Required time =

200
5
2
18

200 18
2 5

360 sec.

Two trains of lengths 213 m and 205 m run on parallel tracks. When running in the same direction the

Ex

15.

L1 L2
s2 s1

am
sT

Sol: Here, L 1 = 120, L 2 = 80, s 1 = 48, s 2 = 50


L 1 + L 2 = 120 + 80 = 200 m

faster train crosses the slower one in 9

1
seconds. When running in opposite direction with the same
2

an
k

speeds, they pass each other completely in

.B

L 1 + L 2 = 213 + 205 = 418


19 11 30
2
2
2
19 11 8
T1 - T2 =
2
2
2

w
w

1
seconds. Find the speed of each train.
2

19
11
and T 2 =
2
2

Sol: Here, L 1 = 213, L 2 = 205, T 1 =

T1 + T2 =

15
4

Speed of the faster train =


=
Speed of the slower train =

L1

L2

1
T1

418 T1 T2
2
T1T2
L1

L2
2

1
T1

1
T2

418
2

15
19 11
2
2

60 m /s .

1
T2

418 T1 T2
2
T1T2

Get all banking study material on BankExamsToday.com

418
2

4
19
2

11
2

16 m /s.

Page 95

www.BankExamsToday.com, best study material for bank exams


CRT
16.

A train starts from Hyderabad at 9 A.M. with a speed of 50 km/hr and another train starts from there on
the same day at 1 P.M. in the same direction with a speed of 70 km/hr. Find at what distance from
Hyderabad both the trains will meet and also find the time of their meeting.
Sol: s 1 = 50, s 2 = 70, T = time from 9 A.M. to 1 P.M. = 4 hours.
s1 s2 T
km
s2 s1

=
s1 T
hr
s2 s1

Time of their meeting =

50 4
20

10hr after 1 P.M.

700km .

od
ay
.

i.e. at 11 P.M. on the same day.


17.

50 70 4
70 50

co
m

Distance of meeting point from Hyderabad =

Vijay Wada is at a distance of 340 km from Hyderabad. A train starts from Hyderabad to Vijay Wada at
4 A.M. with a speed of 60 km/hr. Another train starts from Vijay Wada to Hyderabad at 5 A.M. with a
speed of 80 km/hr. At what distance from Hyderabad will the two trains cross each othe r and also find
the time when they cross each other?

am
sT

Sol:s 1 = 60, s 2 = 80, T = time from 4 A.M. to 5 A.M. = 1 hr

d s2 T
km
s1 s2

Distance of meeting point from Hyderabad = s1


= 60

d s2 T
hr
s1 s2

420
120

180km .

3hr after 4 A.M.

Ex

Time of their meeting =

340 (80 1)
420
km 60
60 80
140

i.e. at 7 A.M.

Two trains start at the same time from Chennai and Cochin and proceed towards at a rate of 85 km/hr
and 100 km/hr respectively. When they meet, it is found that one train has travelled 30 km more than the
other. Find the distance between Chennai and Cochin.

an
k

18.

.B

Sol: s 1 = 85, s 2 = 100, d = 30

Distance between Chennai and Cochin = d


85 100
100 85

30

185
15

km.

2 185 370 k m.

w
w

= 30

s1 + s2
s2 s1

Get all banking study material on BankExamsToday.com

Page 96

www.BankExamsToday.com, best study material for bank exams


CRT
Time and Work

5.

Usually, we need to complete a particular job in a reasonable time.


We need to complete any project earlier or later depending upon the requirements.
Accordingly, the men on duty have to be increased or decreased.
This means, the time allowed and the men engaged for a project are inv ersely proportional to each other.
Again it means, more the number of men involved, the lesser is the time required to finish a job.
You also come across the situations where time and work or men and work are in direct proportion to
each other.

co
m

1.
2.
3.
4.

od
ay
.

Note: If a person can complete a work in n days then he can do 1/n part of the work in one day.
The amount of work done be a person in 1 day is called his efficiency.

Formulae:
1.

If A can do a piece of work in n days, then at a uniform rate of working, A will finish

2.
3.
4.

am
sT

day.

1
th work in one
n

1
th of work is done by A in one day, then A will take n days to complete the full work.
n
1
n
If A does th of a work in m hours, then A will take
hours.
m
n

If

If A does three times faster work than B, then

Ex

a) The ratio of work done by A and B is 3 : 1


b) The ratio of time taken by A and B is 1 : 3

an
k

A, B and C can do a piece of work in T 1 , T 2 and T 3 days respectively. If they have worked for D 1 , D 2
and D 3 days respectively, then
a) Amount of work done by A =

D1
T1

b) Amount of work done by B =

D2
T2

c) Amount of work done by C =

D3
T3

w
w

.B

5.

d) Amount of work done by A, B and C together =

If A can do a piece of work in X days and B can do the same work in Y days, then both of them working

6.

together will do the same work in

7.

XY
days.
X+Y

If A, B and C, while working alone, can complete a work in X, Y and Z days respectively, then they will
together complete the work in

8.

D1 D2 D3
+
+
.
T1
T2
T3

XYZ
days.
XY + YZ + ZX

A and B working together can complete a piece of work in X days. If A working alone can comp lete the
work in Y days, then B working alone will complete the work in

Get all banking study material on BankExamsToday.com

XY
days.
Y X

Page 97

www.BankExamsToday.com, best study material for bank exams


CRT

9.

If A and B working together can complete a piece of work in X days, B and C in Y days, C and A in Z
days, then
a) A, B and C working together will finish the job in

d)

XY

od
ay
.

c)

2XYZ
days.
YZ ZX

co
m

2XYZ
days.
YZ ZX
2XYZ
days.
B alone can finish the job in
YZ ZX XY
2XYZ
C alone can finish the job in
days.
XY YZ ZX

b) A alone can finish the job in

XY

10. If A and B working together can complete a work in X days and B is k times efficient than A, then
a) The time taken by A working alone, to complete the work is (k + 1) X.

am
sT

b) The time taken by B working alone, to complete the work is

x.

c) The time taken by both A and B working together to complete the work is

x
1

11. If A working alone takes a days more than A and B working together. If B worked alone, he takes b
hours more to complete the job than A and B working together then both A and B working together can
12. If A can complete

ab days.

a
c
b c X
part of a work in X days, then part of the work will be done in
days.
d
b
a d

Ex

finish the job in

13. If a men and b women can do a piece of work in n days, then c men and d women can do the work in

an
k

nab
days.
bc ad

14. If A can complete a work in X days and B is k times efficient than A, then the time taken by both A and

.B

B working together to complete the work is

w
w

15. If A is k times more efficient than B and hence able to finish the work in l days less than B, then
a) A working alone can finish the work in

b) B working alone can finish the work in

1
kl

days.
days.

c) A and B working together, can finish the work in

kl
k

days.

16. There are two groups of people with same efficiency. In one M 1 persons can do W 1 works in D 1 time
and in the other M 2 persons can do W 2 works in D 2 time. Then the relationship between the two groups
is M1D1W2 M2D2W1
17. There are two groups of people with same efficiency. In one M 1 persons can do W 1 works in D 1 time
working t 1 hours a day and in the other M 2 persons can do W 2 works in D 2 time working t 2 hours a day.
Then the relationship between the two groups is
Get all banking study material on BankExamsToday.com

Page 98

www.BankExamsToday.com, best study material for bank exams


CRT
M1D1t1W2

M2D2 t 2W1

Level 1 Examples

od
ay
.

co
m

Example 1:
If A can do a work in 10 days, B can do it in 20 days and C in 30 days in how many days will the three together do
it?
Soln:
The efficiencies are A = 1/10, B = 1/20 and C = 1/30
So work done per day by the three = 1/10 + 1/20 + 1/30 = 11/60 => No of days = 60/11 = 5.45 days.
Example 2:
If A and B can do a work in 10 days , B and C can do it in 20 days and C and A can do it in 40 days in what time all
the three can do it?
Soln:
A+B = 1/10
B+C = 1/20
C+A = 1/40

w
w

.B

an
k

Ex

am
sT

Adding all the three we get 2(A+B+C) = 7/40 => A+B+C = 7/80 => No of days = 80/7 days.
Note:
If all the people do not work for all the time then the principle below can be used:
mA + nB + oC = 1. (1 is the total work)
Here, m=no of days A worked
n=no of days B worked
o=no of days C worked
A,B,C = efficiencies
Example 3:
If A can do a work in 12 days, B can do it in 18 days and C in 24 days. All the three started the work. A left after
two days and C left three days before the completion of the work. How many days are required to complete the
work? Soln:
Let the total no of days be x.
A worked only for 2 days, B worked for x days and C worked for x-3 days.
So, mA + nB + oC = 1
2(1/12) + x(1/18) + (x-3)(1/24) = 1
12 + 4x + 3(x-3) = 72
x = 69 / 7 days.
Note:
The ratio of dividing wages = ratio of efficiencies = ratio of parts of work done
Example 4:
A can do a work in 10 days and B can do it in 30 days and C in 60 days. If the total wages for the work is Rs. 1800
what is the share of A?
Soln:
Ratio of wages = 1/10 : 1/30 : 1/60 = 6 : 2 : 1 (Multiplying each term by LCM 60)
So total 9 equal parts in Rs. 1800 => each part = Rs. 200 => share of A = 6 parts = Rs. 1200.
Note:
When pipes are used filling the tank they are treated similar to the men working but some outlet pipes emptying the
tank are present whose work will be considered negative.
Example 5:
A pipe can fill a tank in 5 hrs but because of a leak a the bottom it takes 1 hr extra. In what time can the leak alone
empty the tank?
Soln:
Let the filling pipe be A.
A = 1 / 5.
But with the leak L, A L = 1 / 6 ( A-L because leak is outlet)
So, 1/L = 1 / 5 1/ 6 = 1/30 => Leak can empty the tank in 30 hrs.

Get all banking study material on BankExamsToday.com

Page 99

www.BankExamsToday.com, best study material for bank exams


CRT

Level 2 Examples
1.

A can complete a piece of work by working alone in 5 days and B while working alone can finish the
same work in 10 days. If both of them work together, then in how many days, the work will be finished.
Sol: Here, X = 5, Y = 10

XY
days
X Y

am
sT

Working together, A and B will finish the work in


=

2.

od
ay
.

co
m

Example 6:
A pipe A can fill the tank in 10 hrs, B can fill it in 20 hrs and C can empty in 40 hrs. All are opened at the same time.
After how many hours shall the pipe B be closed such that the tank can be filled in 10 hrs?
Soln:
Let the pipe B be closed after x hrs.
Then A worked for 10 hrs, B worked for x hrs and C worked for 10 hrs.
mA + nB oC = 1 (since C is outlet)
10(1/10) + x(1/20) 10(1/40) = 1
x = 5 hrs.
Example7:
A can do a work in 10 days. Then the efficiency of A is given by A = 1 / 10.
Note:
Number of days required to do a work = work to be done / work per day

5(10)
5 10

10
days.
3

A, B and C can complete a piece of work in 5, 10 and 13 days respectively. In how many days would all
of them complete the same work working together?

Ex

Sol: Here, X = 5, Y = 10, Z = 13

Sol: Here, X = 10, Y = 15

w
w

So, B alone will complete the work in

32
days.
49

10(15)
XY
days =
Y X
15 10

30 days.

Madhu and Anil can do a piece of work in 12 days, Anil and Sunil in 15 days, Sunil and Madhu in 20
days. How long would each take separately to do the same work?

4.

XYZ
days
YZ ZX
5 10 13
=
5(10) 10(13) 13(5)

A and B working together take 10 days to complete a piece of work. If A alone can do this work in 15
days, how long would B take to complete the same work?

.B

3.

XY

an
k

So, the work will be completed in

Sol: Here, X = 12, Y = 15 and Z = 20

2XYZ
days
YZ ZX
2 12 15 20
=
12(15) 15(20) 20(12)
2XYZ
Anil alone can do the work in
days
YZ ZX XY
2 12 15 20
=
15(20) 20(12) 12(15)

Madhu alone can do the work in

XY

Get all banking study material on BankExamsToday.com

Page 100

7200
240

30 days.

7200
360

20 days.

www.BankExamsToday.com, best study material for bank exams


CRT
Sunil alone can do the work in

7200
120

60 days.

Mukesh can do a piece of work in 16 days. If Nita works thrice as fast as Mukesh, how long would they
take to finish the work by working together?

co
m

5.

2XYZ
days
YZ ZX
2 12 15 20
=
12(15) 15(20) 20(12)

XY

Sol: Here, X = 16 and k = 3

6.

days

16
1 3

4 days.

A and B together can do a piece of work in 4 days. If A does twice as much work as B in a given time,
how long A alone would take to do the work?
Sol: Here, X = 4 and k = 2
k

Time taken by A, working alone =

1
k

1
2

6 days

am
sT

7.

od
ay
.

Required time =

A alone would take 7 days more to complete the work than if both A and B worked together. If B
worked alone, he took 1

3
days more to complete the job both worked together. What time would they
4

take if both A and B worked together?


3
4

7
4

Ex

Sol: Here, a = 7, b = 1

Time taken by A and B working together =

Sol: Here, k = 4 and l = 12

7
4

7
days.
2

kl

days =

4 12
4 1

16 days.

.B

Time taken by B working alone =


A can do

1
5
of a work in 15 days. In how many days he can finish of the work?
6
4

w
w

9.

A is four times as good a workman as B and takes 18 days less to do a piece of work than B takes. Find
the time in which B alone can complete the work.

an
k

8.

ab days =

Sol: Here, a = 5, b = 4, c = 1, d = 6, X = 15
Required time =

10. If 12 persons can complete

4 1 15
5 6

2 days.

1
th of a work in 4 days, then find the number of persons required to
5

complete the remaining work in 16 days.


Sol: Here M 1 = 12, D 1 = 4, W 1 =
We have, M1D1W2
12 x 4 x

4
1
and M 2 =?, D 2 = 16, W 2 =
5
5

M2D2W1

4
1
= M 2 x 16 x
5
5

M 2 = 10.

Get all banking study material on BankExamsToday.com

Page 101

www.BankExamsToday.com, best study material for bank exams


CRT
11. If 5 persons can cut 15 trees in 6 days working 10 hours a day. Then, in how many days can 12 persons
cut 36 trees working 6 hours a day.

co
m

Sol: Here M 1 = 5, D 1 = 6, W 1 = 15, t 1 = 10


M 2 = 12, D 2 =?, W 2 = 36, t 2 = 6
We have, M1D1t1W2 M2D2 t 2W1
5 x 6 x 10 x 36 = 12 x D 2 x 6 x 15
D 2 = 10 days.

od
ay
.

12. 8 men or 12 women can do a work in 21 days. In how many days, 5 men and 3 women would complete
the work?
Sol: Here, a = 8, b = 12, c = 5, d = 3 and n = 21
Required no. of days =

nab
bc ad

21 8 12
12(5) 8(3)

21 8 12
84

Exercise

24 days.

P can do a work in 12 days and Q can do it in 36 days. In how many days the work can be done if they do it together?
1. 24
2. 9
3. 10
4. 7

2.

Madan and Raju can do a work in 15 days, if they work together. If Madan alone can do it in 20 days, in how many
days Raju alone can do the work?
1. 8

4
7

2. 60

am
sT

1.

3. 10

4. 44

P and Q can do a work in 9 days. Q and R can do it in 12 days, R and P in 18 days. In how many days all of them can
do the work?
1. 8
2. 9
3. 15
4. 5

4.

Shweta can do a work in 40 days. She worked for 5 days and Srikanth finished the remaining work in 21 days. In
how many days can both of them do the work?
1. 18
2. 19
3. 16
4. 24

5.

Rajeev can do a work in 50 days, which Sanjay can do in 40 days. Rajeev started the work and was joined by Sanjay
after 20 days. In how many days the total work is completed?
3. 33

1
3

4. 14

Pavan and Venu undertook a piece of work for Rs.600. Pavan alone can do the work in 24 days and Venu alone can
do it in 12 days. What is the share of Venu?
1. Rs.360
2. Rs.400
3. Rs.200
4. Rs.240

6.

2. 15

w
w

1. 10

.B

an
k

Ex

3.

7.

P and Q can do a piece of work in 12 days. P, Q and R together can finish it in 8 days. In how many days will R
finish that work?
1.

24
5

2. 24

3. 20

4. 4

8.

P can do a piece of work in 80 days. He works at it for 10 days and then Q alone finishes the work in 42 days. Both
together could complete the work in how many days?
1. 30
2. 48
3. 24
4. 25

9.

P and Q together finish a work in 30 days. They worked for 20 days and then Q left. The remaining work was done
by P alone in 15 days. In how many days P alone can finish the total work?

Get all banking study material on BankExamsToday.com

Page 102

www.BankExamsToday.com, best study material for bank exams


CRT
10.

1. 35
2. 30
3. 45
4. None
P can complete a job in 9 days, Q in 10 days and R in 15 days. Q and R started the work and were forced to leave
after 3 days. Find the time taken by P to complete the remaining work.
1. 4
2. 5
3. 7
4. None
P does half as much work as Q in three fourth of the time. If together they take 18 days to complete a work, how
much time shall P take to do it?
1. 35
2. 30
3. 45
4. None

12.

P can complete a job in 15 days, Q in 10 days and R in 20 days. Q and R started the work together and were forced to
leave after 4 days. Find the time taken by P to complete the remaining work?
1. 5
2. 12
3. 6
4. None

13.

A certain number of men complete a piece of work in 50 days. If there were 8 men more the work could be finished
in 10 days less. How many men were there originally?
1. 30
2. 32
3. 36
4. 40

14.

If 20 men can build a wall of 120 meters long in 6 days, what length of a similar wall can be built by 25 men in 3
days?
1. 95
2. 60
3. 75
4. 70

15.

P and Q can do a piece of work in 12 days, Q and R in 15 days, R and P in 20 days. How long the work runs if they
do it altogether.
1. 10 days
2. 15 days
3. 12 days
4. 5 days

16.

If 17 labourers can dig a ditch 26 meters long in 18 days, working 8 hours a day, how many more labourers should be
engaged to dig a similar ditch of 52 meters length in 16 days, each labourer working 9 hours a day?
1. 34
2. 17
3. 30
4. None

17.

P and Q can do a piece of work in 30 and 70 days respectively. They worked together for 10 days and then Q goes
away. In how many days will P finish the remaining work?
2. 15

an
k

1. 11

Ex

am
sT

od
ay
.

co
m

11.

3. 16

4. 15

5
7

If 5 men or 6 women can do a piece work in 38 days, in how many days will 3 men and 4 women will do the same
work?
1. 20
2. 17
3. 30
4. None

19.

2 men and 3 boys can do a piece of work in 8 days, 3 men and 2 boys can do it in 7 days. In how many days can 1
man and 2 boys do the same work?
1. 24
2. 17
3. 12
4. 14

w
w

P does two-fifth of work in 8 days. He then calls Q and they together complete the remaining work in 6 days. How
long would Q alone take to complete the entire work?
1. 20
2. 17
3. 18
4. None

20.

.B

18.

21.

A job can be completed by 10 men in 20 days and by 40 women in 15 days. How many days it takes for 5 men and 5
women to finish the work?
1. 40
2. 24
3. 30
4. 35

22.

After completing half of a work in 10 days P was joined by Q and they worked together for 3 days. P completed the
remaining work in 4 days. In how many days Q alone can do the entire work?
1. 25
2. 20
3. 30
4. 18

Get all banking study material on BankExamsToday.com

Page 103

www.BankExamsToday.com, best study material for bank exams


CRT
23.

10 men can do a work in 40 days. They started together but 4 men left after a few days. As a result the work was
completed in 60 days. After how many days the 4 men left?
1. 15
2. 20
3. 10
4. None

24.

80 men can finish


1. 38

2
of work in 30 days. How many men are required to finish the remaining work in 60 days?
5

2. 60

3. 30

4. 40

The work done by 1 man and 3 boys are equal. A work can be done by 3 men and 5 boys in 8 days. How many boys
should assist 9 men to finish the work in 4 days?
1. 4
2. 7
3. 1
4. None

26.

P and Q can do a job in 40 days and 36 days respectively. If they did a job for Rs.2660, what should be the share of
Q? (in Rs.)
1. 1260
2. 1400
3. 1520
4. 1600

27.

8 men and 20 boys can construct a house in 5 days working for 6 hours a day. 5 men and 8 boys can construct two
such houses in 16 days working for 7.5 hours a day. In how many days can 12 men and 6 boys construct 5 such
houses working for 10 hours a day?
1. 12
2. 18
3. 24
4. 30

28.

P can do a piece of work in 30 days and Q alone can do it in 40 days. They begin together but 5 days before the
completion of work, P leaves off. What is the total number of days to complete the work?
1. 20
2. 30
3. 25
4. 15

29.

P and Q can do a piece of work in 12 days and 20 days respectively. They started together and then P left after for
some days. If Q finishes the remaining work in 12 days, how many days has P worked on it before leaving?
1. 3 days
2. 4 days
3. 6 days
4. 8 days

30.

P is twice as good a workman as Q. Working together they can finish a work in 16 days. How many days will P take
to finish the work?
1. 4 days
2. 8 days
3. 24 days
4. 48 days

31.

P and Q can do a work in 10 and 15 days respectively. If they work at it on an alternate day, P starting first, in how
many days the work will be completed?
1. 6 days
2. 10 days
3. 12 days
4. 15 days

32.

A contractor agreed to complete a work in 120 days. He engaged 150 men for the work and after 70 days, only half
of the work was completed. How many additional men must employ to complete the work in agreed time?
1. 60 men
2. 90 men
3. 150 men
4. 210 men

od
ay
.

am
sT

Ex

an
k

.B

w
w

A camp has provision of food for 600 persons for 41 days. After 37 days, 360 persons left the camp. How long the
food will last for the remaining persons?
1. 5 days
2. 8 days
3. 10 days
4. 12 days

33.

co
m

25.

34.

P can do a job in 10 days, Q can do it in 12 days and R can do it in 15 days. All begin together, but P leaves the work
after 2 days and Q leaves 3 days before the work is completed. How long did the work last?
1. 9 days
2. 8 days
3. 7 days
4. 6 days

35.

P and Q can do a piece of work in 45 days and 40 days respectively. They began to work together but P leaves after
x days and Q finishes the remaining work in (x + 14) days. After how many days did P leaves?
1. 19 days
2. 11 days
3. 9 days
4. 12 days

Get all banking study material on BankExamsToday.com

Page 104

www.BankExamsToday.com, best study material for bank exams


CRT
Shekhar has done

1
rd of a job in 8 days, Kamalakar completes the rest of the job in 8 days. In how many days
3

would Shekhar and Kamalakar together had completed the work?


1. 24 days
2. 16 days
3. 20 days
37.

4. 8 days

1
A garrison has provisions for certain number of days. After 15 days,
of the men left and if it is found that the
4

provisions will now last just as before, how long was that?
1. 50 days
2. 60 days
3. 70 days

4. 30 days

co
m

36.

If 4 labourer reap 40 acres in 30 days, how many acres will 18 labourer reap in 12 days?
1. 49 acres
2. 70 acres
3. 72 acres
4. 65 acres

39.

6 men can prepare 6 toys in 5 days working 4 hrs a day. In how many days can 10 men can prepare 15 toys working
5 hrs per day?
1. 4
2. 5
3. 6
4. 7

40.

2 men and 3 boys can do a piece of work in 10 days and 3 men together with 2 boys can do the same job in 8 days. In
how many days can 2 men and a boy do the work?
1
days
2

2. 18

Answers

an
k

Ex

11) 4
12) 3
13) 2
14) 3
15) 1
16) 2
17) 4
18) 3
19) 4
20) 1

3. 25 days

21) 3
22) 2
23) 3
24) 2
25) 3
26) 1
27) 2
28) 1
29) 1
30) 3

w
w

.B

1) 2
2) 2
3) 1
4) 4
5) 3
6) 2
7) 2
8) 1
9) 3
10) 1

1
days
2

am
sT

1. 12

od
ay
.

38.

Get all banking study material on BankExamsToday.com

Page 105

4. 25

1
day
2

31) 3
32) 1
33) 3
34) 3
35) 3
36) 4
37) 2
38) 3
39) 3
40) 1

www.BankExamsToday.com, best study material for bank exams


CRT

Mensuration 2D

1. Triangle
2. Quadrilateral
3. Polygon
4. Circle
5. Sector of a circle
6. Rectangular Paths
7. Circular paths
I. Triangle

Perimeter (P) : P = a + b + c = 2s
1
altitude =
2

(b). Equilateral

s (s a) (s

b) (s

any side
c)

is the

h
r

length of

Ex

base

dropped on that side =

b
B

an
k

1
Area (A) : A =
2

am
sT

(a). Any triangle


a, b and c are three sides of the triangle; h is the altitude and AC
base.

od
ay
.

co
m

Types of Plane Figures

a is the length of each side

3 2
a
4

w
w

Area (A) : A =

.B

Perimeter (P) : P = 3a

(c). Right-angled

b, c are the lengths of the two legs


Perimeter (P) : P = a + b + c = 2s
Area (A) : A =

1
2

product of two legs

90o
A

(d). Isosceles
a is the length of two equal sides
b is the base

B
a

Get all banking study material on BankExamsToday.com

A
b/2

Page 106
b/2

www.BankExamsToday.com, best study material for bank exams


CRT
BD is the perpendicular dropped on base such that it divides the base equally AD = CD =
Perimeter (P) : P = 2a+b
b
4a2
4

b2

(e). Right-angled Isosceles


Perimeter (P) : P = 2 a
1
Area (A) : A =
2

(a)

( 2 +1)

Hypotenuse - h

od
ay
.

II. Quadrilateral
(a). Any Quadrilateral

AC is the diagonal = d, DE and BF are two perpendicular drawn on the


diagonal (AC) P1, and P2 are the lengths of the two perpendiculars

1
2

d (p1+p2) =

1
2

drawn on that diagonal)

am
sT

Perimeter (P) : P = sum of the four sides.


Area (A) : A =

co
m

Area (A) : A =

any diagonal

(sum of

rs

an
k

Ex

P1

(b). Rectangle
l = length
b = breadth
d = diagonal

b
2

P2
B

Perimeter (P) : P = 2(l + b) = 2(l+ d2 l2 ) =


2(b+Error! Objects cannot be created from editing
d2

2A

.B

codes.) = 2

w
w

d
l = b Error! Objects
Area (A): A = l b = l
cannot be created from editing field codes., when p and A are known and l and b are unknown.
The two values of x will give l and b.
a
D
O
A
B
(c). Square
A
90oa
a = length of side
OB
d = diagonal
A O
a
d
O
B
C
Perimeter (P) : P = 4a = 2d 2
90oa
B
A
B
OB
d2
p2
A
A
=
Area (A) : A = a2 =
A
2
16
A
a

O
B
A

Get all banking study material on BankExamsToday.com

Page 107

C
B
A
a
O
B
A
B
A

field

www.BankExamsToday.com, best study material for bank exams


CRT

D
C
B
A

Area (A) : A =

1
2

d1

d2 =

d2
2
d1
2

A
d1
2

a2

d2
2

d1
2

d2
2

a2

(e). Trapezium

h=

1
2

(sum of parallel sides)

(perpendicular distance between parallel sides)

am
sT

1
(a + b)
2

a
D
=aC h
B
A

(f). Parallelogram
b is the base
h is the perpendicular distance between the base and its opposite side

D
C
B

an
k

C
B

h
b
a
A

b
a
A

B
C
B

h
b
a
A

Ex

Area (A) : A = b h = base (perpendicular distance between the base


and its opposite sides)
= 2 area of ABD (or BCD)
III. Polygon

a
D
C
B
A

B
A

a
A

D
C
B

a and b are two parallel sides, h is the height


Area (A) : A =

90oO
aDC
BAO
a
90oO
D
aDC
C
BA
B
A

od
ay
.

Perimeter (P) : P = 4a = 2

d12

a
D
C
B
A

d2

C
B
A

co
m

(d). Rhombus
a = each side
d1 = one diagonal
d2 = another diagonal
h = height

a
D
C
B
A

Polygon is a n-sided closed figure bounded only by line segments.

.B

In a polygon if the internal angle at each vertex is less than 180 o then the
polygon is a convex polygon, else a concave polygon.

Convex Polygon:

ii.

Area of a regular polygon =

w
w

i.

1
2

perimeter

Number of diagonals in a polygon =

distance from the center of the polygon to any side.

n(n 3)
2

Sum of all interior angles of a polygon = (2n-4)

90o

iv.

Each interior angle of n-sided regular polygon =

n 2
n

v.

Sum of all exterior angles of n-sided regular polygon = 360o

vi.

Each exterior angle of n-sided regular polygon =

iii.

b
a
A

180o

360
n

B
A

IV. Circle
O is the center of the circle

Get all banking study material on BankExamsToday.com

Page 108
D
C

C
B
A

www.BankExamsToday.com, best study material for bank exams


CRT
OA = OC = OB = OD = radius of circle = r
AC = BD = diameter of circle = d = 2r
Circumference (or Perimeter) C = 2 r = d
d2
4

Area of circle A = r2 =

If C = circumference, A = area then


C2
A
and
4
C

co
m

r
2

V. Sector of a circle

r2

360o

Length of the Arc AB =

360

VI. Rectangular Paths

N
K

w
w

Case - I

L
Rectangle
C
B
A
D
N
K

B
A
D
C
N
B
K
A
D
N
K

L
M
N
K

an
k

A
D
N
D
K
N
K

.B

2r

Ex

Case - I

am
sT

Area of sector AOB =

od
ay
.

A=

Path
way

M
N
K

A
D
N
K
D
N
K

K
N
K

Case - II
L
C
B
A
WD
wN
W
K
M
N
K

Pathway is outside the rectangle

The length of rectangle AB = l, Breadth BC = b and , Width of path way = W, then


Area of Pathway = 2W (l+b+2w) (shaded portion)

Case II
Path way is inside the rectangle
Area of Pathway = 2W(l+b-2W) (shaded portion)

Get all banking study material on BankExamsToday.com

Page 109

L
M
N
M
K
N
K

B
A
D
N
K
C
B
A
D
N
K

www.BankExamsToday.com, best study material for bank exams


CRT

VII. Circular Pathway

A
W

r
R
W

r
W
W
O C

od
ay
.

A
W

co
m

OAC is a circle of radius = r, there is pathway, outside the circle of width = W


Area of circular pathway =

W (2r+W)

am
sT

When, the pathway is inside the circle,

W (2r - W)

Area of circular pathway =

Examples:

If three sides of a triangle are 5, 6 and 7 cm respectively, find the area of triangle.
=

s(s

a)(s

b)(s

c)

an
k

Sol: Area of

Ex

1.

Now, s =

b
2

6
2

5)(9

=9

6)(9

7)

4 3 2

216 6 6 cm 2 .

w
w

2.

9 (9

.B

Area =

ABC is an equilateral triangle of side 24 cm. Find the in radius of the triangle.

Sol: In a equilateral triangle, the altitude, median and perpendicular are equal.
AD =

3 /2 x 24 = 12 3

GD (in radius) = 1/3 x 12 3 = 4 3 cm


3.

The base and other side of an isosceles triangle is 10 and 13 cm respectively. Find its area.
Sol: Area of Isosceles
Given,

b
4a2
4

b2

base b = 10 Other side a = 13

Get all banking study material on BankExamsToday.com

Page 110

www.BankExamsToday.com, best study material for bank exams


CRT
10
4 (13)2 102
4
10
=
24 = 60 cm 2 .
4

Area (A)

4.

10
676
4

100

In a right-angled triangle, the length of two legs are 12 and 5 cm. Find the length of hypotenuse and its area.

122

Hypotenuse =

52 =

co
m

Sol: In a right angled triangle,


(Hypotenuse) 2 = (one leg) 2 + (other leg) 2
= 12 2 + 5 2
169 = 13 cm.

Area
5.

1
2

(leg)1 (leg)2

od
ay
.

In a right angled triangle,

1
2
12 5 = 30 cm .
2

If the perimeter and diagonal of a rectangle and 14 and 15 cm respectively. Find its area.

am
sT

Sol: In a rectangle,

(Perimeter)2
(14)2
= (diagonal)2 + 2 x Area ;
= (5) 2 + 2 x Area
4
4

2 x Area =

Area =

49

25

= 12 cm 2 .

Find the length of the diagonal and the perimeter of a square plot if its area is 900 square metres.
d2
2

p2
16

an
k

Sol: In a square, A =

Ex

6.

196
- 25
4

(Diagonal) 2 = 2 x Area = 900


Diagonal (d) = 2 900 30

2 = 42.42 metres

.B

(Perimeter) 2 = 16 x Area = 16 x 900

7.

16 900 = 120 metres.

w
w

Perimeter (P) =

A field in the shape of a rhombus has the distances between pairs of opposite vertices as 14 m and 48 m.
What is the cost (in rupees) of fencing the field at Rs.20 per metre?

Sol: The diagonals are 14 m and 48 m


Sides of rhombus =

14
2

48
2

625 = 25

Perimeter of rhombus = 4 x 25 = 100 m.


Cost of fencing the field = 100 x 20 = Rs.2000
8.

In a trapezium, the length of parallel sides are 20 and 25 metres respectively and the perpendicular distance
between the parallel sides is 12 metres. Find the area of trapezium.

Get all banking study material on BankExamsToday.com

Page 111

www.BankExamsToday.com, best study material for bank exams


CRT
Sol: One parallel side a = 20 metres. Second parallel side b = 25 metres. Height (perpendicular distance
between a and b) = 12 metres.
Area =

b) h

1
(20
2

25) 12 = 270 m .

The distance between a pair of opposite vertices of a quadrilateral is 32 units . The lengths of the
perpendiculars drawn on to this diagonal from the other two vertices are 4 1/3 units and 6 2/3 units
respectively. Find the area (in sq units) of the quadrilateral?
13
3

Sol: Area of quadrilateral = 1/2 x 32 x

= 178 sq units.

od
ay
.

20
3

co
m

9.

1
(a
2

10.

am
sT

D
o

In the above parallelogram ABCD,

A = x + 30 and

D = x 40 , what is the measure of

DCB ?

Sol: In a parallelogram, sum of adjacent angles is equal to 180 o

Sol:

an
k

In a circle of radius 49 cm, an arc subtends an angle of 36 o at the centre. Find the length of the arc and the
area of the sector.
Length of the arc

.B

11.

Ex

x + 30 + x 40 = 180
x = 95 o
o
DAB = x + 30 = 95 + 30 = 125
o
DCB = DAB = 125
(opposite angles of a parallelogram are equal)

2 r
360

2 22 49 36
= 30.8 cm
7 360

r2
360

22 49 49 36
= 754.6 cm 2
7 360

A rectangular plot of dimensions 13 m x 17 m is surrounded by a garden of width 5 m. What is the area (in
sq m) the garden?

12.

w
w

Area of the sector

Sol: Let ABCD be the rectangular plot of given dimension. The shaded part is the surrounding garden. Now,
the plot ABCD together with the garden forms another rectangular form PQRS. Dimensions of PQRS, as can
be seen from the diagram, are:
Length PQ = width of garden + AB + width of garden
= 5 + 17 + 5 = 27 m
Similarly, breadth = PS = 5 + 13 + 5 = 23 m
Area of garden
= Area of PQRS Area of ABCD
= (27 x 23) (17 x 13) = 621 221 = 440 sq m.

Get all banking study material on BankExamsToday.com

Page 112

www.BankExamsToday.com, best study material for bank exams


CRT
13.

There is a rectangular field of length 100 m and breadth 40 m. A carpet of 2 m width is to be spread from the
centre of each side to the opposite side. What is the area of the carpet?

There is an equilateral triangle of which each side is 3 m. With all the three vertices as centres, circles with
radius 1.5 cm are described (i) Calculate the area common to all the circles and the triang le. (ii) Find the
area of the remaining portion of the triangle.
Sol: (i) Area of each sector =

1
6

r2

od
ay
.

14.

co
m

Sol: Area of the carpet ABCD


= 40 m x 2 m = 80 m 2
Area of the carpet EFGH
= 100 m x 2 m = 200 m 2
But the common area of two carpets
= 2 x 2 = 4m 2
So, area of the carpet
= 200 + 80 4 = 276 m 2

So area common to the all the circles and triangle = 3


=

1
2

22
2
1.5 1.5 = 3.53 m
7

1
6

r2

1
2

r2

But area of the triangle

am
sT

(ii) Area of the shaded portion = Area of the triangle Area common to the triangle and the circles
3 2
a
4

So area of the shaded portion

9 3 2
m
4

9 3 2
m 3.53 m 2 = 3.89 m 2 3.53 m 2 = 0.36 m 2
4

Ex

Answers

3
(3)2
4

Find the area of a triangle having sides 3 m, 4 m and 5 m.


1. 60 sq m
2. 10 sq m
3. 12 sq m

4. 6 sq m

2.

Find the area of a triangle whose base is 4.6m and height is 67 cm.
1. 154.10 sq m
2. 15410 sq m
3. 15.410 sq m

4. None

3.

Find the area of an equilateral triangle each of whose sides measures 6 cm.
1. 36 sq cm
2. 3 3 sq cm
3. 9 3 sq cm
4
Length of the side of an equilateral triangle is
cm. Find its height.
3
1. 2 cm
2. 4 cm
3. 6 cm

.B

w
w

4. 12 sq cm

4. None

4.

an
k

1.

5.

Height of an equilateral triangle is 4 3 cm. Find its area.


1. 4 3 sq cm

2. 2 3 sq cm

3. 16 3 sq cm

4. 8 3 sq cm

6.

An isosceles right-angled triangle has two equal sides of length 6 m each. Find its area
1. 8 sq m
2. 36 sq m
3. 18 sq m
4. None

7.

The perimeter of an isosceles triangle is 80 cm. If the length of the equal sides is given by 0.15 m, find the length of
the base.
1. 40 m
2. 50 m
3. 12 m
4. 90.5 m
The perimeter of an isosceles triangle is 42 cm. If the base is 16 cm, find the length of equal sides.

8.

Get all banking study material on BankExamsToday.com

Page 113

www.BankExamsToday.com, best study material for bank exams


CRT
1. 13 cm

2. 8 cm

3. 21 cm

4. 29 cm

The two adjacent sides of a parallelogram are 5 m and 6 m respectively, and if the diagonal connecting the ends is 9
m, find the area of the parallelogram (approximately).
1. 29 sq m
2. 28 sq m
3. 58 sq m
4. 50 sq m

10.

Find the area of a quadrilateral of whose diagonal is 38 cm long and the lengths of perpendiculars from the other two
vertices are 31 cm and 19 cm, respectively.
1. 950 sq cm
2. 475 sq cm
3. 138 sq cm
4. 276 sq cm

11.

Find the area of a parallelogram whose two adjacent sides are 130 m and 140 m and one of the diagonals is 150 m
long.
1. 8400 sq cm
2. 16,800 sq cm
3. 2100 sq cm
4. None

12.

Find the diagonal of a rectangle whose sides are 8 cm and 6 cm.


1. 14 cm
2. 5 cm
3. 20 cm

4. 10 cm

13.

Find the perimeter of a rectangle of length 12 m and breadth 6 m.


1. 18 m
2. 72 m
3. 36 m

4. 144 m

14.

Calculate the area of a rectangular field whose length is 12.5 cm and breadth is 8 cm.
1. 10 sq cm
2. 100 sq cm
3. 200 sq cm
4. 1 sq cm

15.

Calculate the area of a rectangular field whose one side is 16 cm and the diagonal is 20 cm.
1. 192 sq cm
2. 96 sq cm
3. 294 sq cm
4. 72 sq cm

16.

A rectangular carpet has an area of 120 sq m and perimeter of 46 m. Find the length of its diagonal.
1. 34 m
2. 51 m
3. 93 m
4. 17 m

17.

The perimeter of a rectangle is 82 cm and its area is 400 sq m. Find the length of the rectangle.
1. 8 m
2. 16 m
3. 32 m
4. 64 m

18.

If the area of a square field be 6050 sq m, find the length of its diagonal.
1. 220 m
2. 110 m
3. 55 m

4. None

19.

Find the area of a square with perimeter 48 m.


1. 288 sq m
2. 72 sq m

4. 96 sq m

20.

Find the diagonal of a square field whose side is of 6 m length.


1. 12 2 m
2. 6 2 m
3. 2 m

4. 3 2 m

Perimeter of a square field is 16 2 cm. Find the length of its diagonal.


1. 16 cm
2. 4 cm
3. 8 cm

4. 64 cm

od
ay
.

am
sT

Ex

an
k

.B

w
w

3. 144 sq m

21.

co
m

9.

22.

The area of a rhombus is 156 sq m. If one of its diagonals is 13 m, find the length of the other diagonal.
1. 12 m
2. 6 m
3. 48 m
4. 24 m

23.

Find the area of a rhombus whose one side is 13 cm and one diagonal is 24 cm.
1. 60 sq cm
2. 120 sq cm
3. 240 sq cm
4. 74 sq cm

24.

If the perimeter of a rhombus is 73 cm and one of its diagonals is 27.5 cm, find the other diagonal and the area of the
rhombus.
1. 24 cm, 330 sq cm
2. 20 cm, 115 sq cm
3. 30 cm, 660.8 sq cm
4. 40 cm, 100.5 sq cm

Get all banking study material on BankExamsToday.com

Page 114

www.BankExamsToday.com, best study material for bank exams


CRT
25.

In a rhombus, the lengths of two diagonals are 18 m and 24 m. Find its perimeter.
1. 15 m
2. 30 m
3. 60 m
4. 120 m

26.

The diagonally of Rhombus are 12 cm and 5 cm respectively. Find the side of the Rhombus.
1. 5 cm
2. 6.5 cm
3. 6 cm
4. 8.5 cm

27.

What is the radius of a circular plot whose circumference is 176 m?


1. 14 m
2. 56 m
3. 88 m

28.

A circular plot covers an area of 154 sq m. How much wire is required for fencing the plot?
1. 44 m
2. 22 m
3. 88 m
4. 77 m

29.

Find the area of sector of a circle whose radius is 10 cm and the angle at the center is 36o.
2. 31

7
sq cm
3

3. 30

7
sq cm
3

co
m

3
sq cm
7

4. 31

3
sq cm
7

od
ay
.

1. 30

4. 28 m

30.

Find the area of sector of a circle whose radius is 12 cm and the length of the arc is 20 cm.
1. 60 sq cm
2. 240 sq cm
3. 120 sq cm
4. 64 sq cm

31.

Find the side of a regular hexagon whole area is 30 3 sq cm.


1. 5 cm

Find the area of a regular octagon whole side measures


1. 4( 2 +1) sq cm

4. None

8 cm.

3. 16 ( 2 +1) sq cm

4. ( 2 +1) sq cm

Find the sum of interior angles of a regular polygon of 12 sides. Also, find the value of each interior angle.
5
6

2. ,

5
6

3. 8 ,

Ex

1. 10 ,
34.

2. 8( 2 +1) sq cm

6
5

4. None

Find the sum of all the exterior angles of a regular polygon of 10 sides. Also, find the value of each exterior angle.
1. ,

2. 2 ,

an
k

33.

3. 3 5 cm

am
sT

32.

2. 2 5 cm

3. 3 ,

4. None

The length and breadth of a rectangle are increased by 20% and 5%, respectively. Find the percentage increase in its
area.
1. 25%
2. 26%
3. 13%
4. 15%

36.

Two poles 15 m and 30 m high stand upright in a play ground if their feet be 36 cm a part find the distance between
their tops.
1. 41 cm
2. 39 cm
3. 29 cm
4. 42 cm

w
w

If all the sides and the diagonals of a square are increased by 8% each, then find the percentage increase in its
perimeter?
1. 8%
2. 6%
3. 1%
4. None

37.

.B

35.

38.

Ratio of the areas of two squares is 16 : 9. Find the ratio of their diagonals.
1. 2 : 4
2. 9 : 16
3. 4 : 3

39.

The diagonal of a square is doubled. How many times will the area of the new square become?
1. 2 times
2. 4 times
3. 6 times
4. 8 times

40.

How many meters of a carpet 12 cm wide will be required to cover the floor of a room which is 600 cm long and 420
cm broad? Also, calculate the amount required in carpeting the floor if the cost of carpet is Rs.15 per meter.
1. Rs.3150
2. Rs.9000
3. Rs.1800
4. Rs.10,800

Get all banking study material on BankExamsToday.com

Page 115

4. 64 : 9

www.BankExamsToday.com, best study material for bank exams


CRT
A hall of length 24 cm and breadth 20 cm is to be paved with equal square tiles. What will be the size of the largest
tile so that the tiles exactly fit and also find the number of tiles required?
1. 60
2. 30
3. 480
4. 120

42.

A rectangular park 18 m x 12 m, is surrounded by a path 4 m wide. Find the area of the path.
1. 304 sq m
2. 152 sq m
3. 608 sq m
4. 864 sq m

43.

A park is square in shape with side 18 m. Find the area of the pavement 3 m wide to be laid all around it on its inside.
1. 360 sq m
2. 180 sq m
3. 90 sq m
4. None

44.

A playground measures 27 m x 13 m. From the center of each side a path 2 m wide goes across to the center of the
opposite side. Calculate the area of the path and the cost of constructing it at Rs.4 per sq m.
1. Rs.101
2. Rs.404
3. Rs.202
4. Rs.304

45.

A square field is surrounded by a path 2 m wide on its outside. The area of the path is 72 sq m. What is the area of the
field?
1. 15 sq cm
2. 121 sq m
3. 36 sq m
4. 18 sq m

46.

A circular park of radius 22 m has a path of width 1.4 m around it on its inside. Find the area of the path.
1. 178.45 sq m
2. 187.45 sq m
3. 187.54 sq m
4. None

47.

If the area of a square is 33 sq cm, then find the area of the circle formed by the same perimeter.
1. 21 sq cm
2. 66 sq cm
3. 33 sq cm
4. 42 sq cm

48.

Find the area of largest circle inscribed in a square of side 112 cm.
1. 4928 sq cm
2. 8856 sq cm
3. 9856 sq cm

49.

Find the side of the square inscribed in a circle whose circumference is 308 cm.
1. 25 2 cm
2. 49 2 cm
3. 36 2 cm
4. 16 2 cm

50.

The diameter of a wheel is 2 cm. If it rolls forward covering 10 revolutions, find the distance traveled by it.
1. 62.8 cm
2. 31.4 cm
3. 15.7 cm
4. 58.2 cm

4. None

w
w

1) 4
2) 2
3) 3
4) 1
5) 3
6) 3
7) 2
8) 1
9) 2
10) 1

11) 2
12) 4
13) 3
14) 2
15) 1
16) 4
17) 2
18) 2
19) 3
20) 2

.B

Answers

an
k

Ex

am
sT

od
ay
.

co
m

41.

21) 3
22) 4
23) 1
24) 1
25) 3
26) 2
27) 4
28) 1
29) 4
30) 3

Get all banking study material on BankExamsToday.com

31) 2
32) 3
33) 1
34) 2
35) 2
36) 2
37) 1
38) 3
39) 2
40) 1

Page 116

41) 4
42) 1
43) 2
44) 4
45) 2
46) 2
47) 4
48) 3
49) 2
50) 1

www.BankExamsToday.com, best study material for bank exams


CRT

MENSURATION : 3D
Cuboid :
b

= 2h(l + b)
= 2h(l + b) + 2lb = 2(lb + bh + hl)

Longest diagonal
Volume

= l2 b2 h2
= lbh

Cube:

od
ay
.

Lateral Surface Area


Total Surface Area

co
m

A right prism with a rectangular base is called a Cuboid.


The sides of the base are length (l) and breadth (b). The height is h.

Ex

am
sT

If the length, breadth and height of a cuboid are all equal, it is called a cube.
Then, if edge of the cube
=a
Longest diagonal
= 3a
Lateral Surface Area
= 6a 2
Total surface Area
= 6a 2
3
Volume
=a

Cylinder :

a
a

an
k

A cylinder can be considered to be a right prism except that instead of identical polygons a
cylinder has identical circles for its top and base and it has a single lateral surface also called
curved surface, instead of several rectangular surfaces.
The basic measurements are the radius of the base (or top) r and the height h.

w
w

.B

Curved Surface (Lateral Surface Area) = 2 rh


Total surface Area
= 2 rh+2 r 2 = 2 r(h + r)
2
Volume = r h
Hollow Cylinder:

The cross section of a hollow cylinder is a ring.


Volume of the material of a hollow cylinder = h(R 2 -r 2 )
Here R is outer radius and r is inner radius of the hollow
cylinder.

h
R

Cone:
A cone can be formed from the sector of a circle by rolling it and joining together its two
straight edges. If r is the radius of the cone, and R is the radius of the sector of angle , then
Get all banking study material on BankExamsToday.com

Page 117

www.BankExamsToday.com, best study material for bank exams


CRT

R
360

1.

r=

2.
3.
4.

Relation between r, l and h. (the radius, the slant height and height) is l 2 = h 2 +r 2
Curved Surface area of Cone = prl
Total Surface Area = rl + r 2 = r(l + r)

5.

Volume =

Sphere:

od
ay
.

All points on the surface of a sphere are at the same distance from the center
of the sphere. This distance is called the radius, r.
Surface Area of Sphere = 4 r 2
Volume of a Sphere =

co
m

1
2
r h
3

4
3
r
3

The sphere has only one surface and hence only one surface area.

The radius is r.
Curved Surface Area = 2 r 2
Total Surface Area = 2 r 2 + r 2 = 3 r 2
1
2

4 3
r
3

2
3
r
3

Examples:

A cuboid is 20 m x 10 m x 8 m. Find the length of diagonal, surface area and volume.


Sol: In a cuboid ,

an
k

1.

w
w

.B

Diagonal d
Surface are S
Volume

2.

Ex

Volume =

am
sT

Hemisphere:

= l2 b2 h2 = 202 102 82 = 23.75


= 2 (20 x 10 + 10 x 8 + 8 x 20) = 880 m2
= l x b x h = 20 x 10 x 8 = 1600 m3.

A cube has edge 12 m. Find its length of diagonal, surface area and volume.

Sol: In a cube

3.

Diagonal d
Surface area S
Volume V

= Edge x 3 = 12 x 3 = 20.78 m
= 6 x (Edge)2 = 6 x (12)2 = 864 m2
= Edge3 = (12)3 = 1728 m3.

The base of a right prism is a regular pentagon of side 18 cm. If the height of the prism is 2/3rd of the side of the base,
how much is the lateral surface area (in sq cm) of the prism?
Sol: Perimeter of the base of the prism
Lateral surface area of a right prism

Get all banking study material on BankExamsToday.com

= number of sides x length of each side


= 5 x 18 = 90 cm.
= (Base perimeter) x (height)
Page 118

www.BankExamsToday.com, best study material for bank exams


CRT
2
18 = 1080 sq cm
3

= (90)

If the radius of a sphere is increased by 50%, find the increase percent in volume and the increase percent in the
surface area.

4
3

R 3 , New volume

19
6

Increase % in volume =

R3

4 R3

Increase % in surface area =

5 R2
4 R2

9 R3 .

100 % = 237.5%

3R
2

100 % = 125%.

9 R2 .

A cylinder with base radius of 8 cm and height of 2 cm is melted to form a cone of height 6 cm. Find the radius of the
cone?
Sol: Let the radius of the cone be r cm.
1
3

r2

r2

8 8 2

8 8 2 3
6

= 64

r = 8 cm.

Ex

Then,

A brick measures 10 cm x 5 cm x 3 cm. How many bricks will be required for a wall of 100 metre long 6 metre high

an
k

6.

3R
2

R2. New surface area = 4

Original surface area = 4

5.

4
3

am
sT

Original volume =

3R
.
2

co
m

150
R
100

Sol: Let original radius = R. Then new radius =

od
ay
.

4.

and 1.5 metre thick?

Sol: Volume of the wall = 100 m x 6 m x 1.5 m = 900 m3


1
m
10

.B

Volume of one brick =

900m3
= 60,00,000
3
3
m3
20000

What is the maximum length of a pencil which can be inscribed in a box of length 24 units, breadth 3 units and height
4 units?

7.

w
w

No. of bricks required =

1
3
3
m
m
m3
20
100
20000

Sol: Maximum length in a cuboid is its diagonal


Length of main diagonal is
=

8.

length2 breadth2 height2

24

576

589 units

The height and base-radius of a right circular cone are 10 cm and 24 cm respectively. What is the area of the curved
surface area (in sq cm) if the cone?

Get all banking study material on BankExamsToday.com

Page 119

www.BankExamsToday.com, best study material for bank exams


CRT
Sol: Curved surface area of a cone =

rl,

L2 = h2 + r2 = 102 +242
l = 26 (10 and 24 are in the ratio of 5 : 12; hence l will be the 2 x 13 = 26
Hence, curved surface area = rl =
x 24 x 26 624 sq cm.

Exercise

co
m

R and l being radius and slant height.


It is given that height h = 10 cm and radius = 24 cm.

For any regular solid, No. of faces + No. of vertices = No. of _______ + 2
1. Faces
2. Vertices
3. Edges

4. Sides

2.

Find the volume of a cuboid whose dimensions are 20 m, 15 m and 3 m.


1. 900 m3
2. 800 m3
3. 700 m3

4. 450 m3

3.

Find the length of the longest bamboo stick that can be placed in a room of 16 m long, 11 m broad and 8 m high.
1. 441 m
2. 21 m
3. 210 m
4. 35 m

4.

The area of a side of a box is 150 sq cm. The area of the other side of the box is 20 sq cm. If the area of the upper
surface of the box is 30 sq cm, then find the volume of the box.
1. 300 m3
2. 200 m3
3. 900 m3
4. 900 m3

5.

Cube is a special type of cuboid in which each face is a ________.


1. Rectangle
2. Rhombus
3. Square

6.

Find the volume of a cube whose sides measure 3 cm.


1. 12 cm3
2. 6 cm3
3. 9 cm3

7.

Find the volume of a cube whose surface area is 384 sq cm.


1. 512 cm3
2. 192 cm3
3. 128 cm3

8.

The diagonal of a cube is 4 3 cm. Find its total surface area and volume.
1. 16 sq cm, 64 cm3
2. 96 sq cm, 16 cm3
3. 6 sq cm, 4 cm3
4. 96 sq cm, 64 cm3
If the volumes of two cubical blocks are in the ratio of 27 : 8, what will be the ratio of their sides?
1. 2 : 3
2. 3 : 2
3. 3 : 8
4. 27 : 2

10.

4. 64 cm3

am
sT

Ex

.B

an
k

4. 27 cm3

The volumes of two cubes are in the ratio of 1 : 27. Find the ratio of their surface areas.
1. 9 : 1
2. 1 : 3
3. 2 : 3
4. 1 : 9
The sides of two cubes are in the ratio of 3 : 4. Find the ratio of their surface areas.
1. 9 : 16
2. 9 : 4
3. 3 : 16
4. 16 : 9

11.

4. Circle

w
w

9.

od
ay
.

1.

12.

A right circular cylinder is a solid with circular ends of equal radius and the line joining their centres _____________
to them.
1. Parallel
2. Perpendicular
3. Both 1 and 2
4. None

13.

The diameter of the base of a right circular cylinder is 42 cm and the height is 10 cm. Find the volume of the
cylinder.
1. 1386 cm3
2. 13860 cm3
3. 13680 cm3
4. 8610 cm3

Get all banking study material on BankExamsToday.com

Page 120

www.BankExamsToday.com, best study material for bank exams


CRT
14.

15.

Find the area of the curved surface of the right circular cylinder if the diameter of the base is 28 cm and the height is
20 cm.
1. 1760 cm2
2. 176 cm2
3. 560 cm2
4. 880 cm2
A cylinder of height 20 cm has base of radius 8 cm. Find the total surface area of the cylinder.
1. 1480 sq cm
2. 1408 sq cm
3. 160 sq cm
4. 704 sq cm
Two circular cylinders of equal heights have their radii in the ratio of 3 : 4. Find the ratio of their volumes.
1. 16 : 9
2. 4 : 3
3. 9 : 16
4. 27 : 64

17.

Two circular cylinders of equal heights have their curved surface areas in the ratio of 4 : 5. Find the ratio of their
volumes.
1. 16 : 25
2. 5 : 4
3. 25 : 16
4. 4 : 25

18.

Two circular cylinders of equal curved surface areas have their heights in the ratio of 5 : 7. Find the ratio of their
volumes.
1. 25 : 49
2. 7 : 25
3. 5 : 49
4. 7 : 5

19.

The heights and curved surface areas of two right circular cylinders are in the ratio 3 : 5 and 7 : 10 respectively. Find
the ratio of their radii.
1. 6 : 7
2. 7 : 6
3. 21 : 50
4. 50 : 21

20.

The radii of two right circular cylinders are in the ratio of 2 : 3 and their curved surface areas are in the ratio of 4 : 5.
Find the ratio of their heights.
1. 15 : 8
2. 6 : 5
3. 8 : 15
4. 5 : 6

21.

A right circular cone is a solid obtained by rotating a __________ triangle around its height.
1. Right-angled
2. Acute-angled
3. Obtuse-angled
4. All the three

22.

Find the slant height of a cone whose volume is 8624 cm3 and radius of the base is 14 cm.
1. 6 cm
2. 196 cm
3. 42 cm
4. 1960 cm

23.

A tent is of diameter 6 m at the base and its height is 4 m. Find the slant height.
1. 5 m
2. 6 m
3. 10 m
4.

an
k

Ex

am
sT

od
ay
.

co
m

16.

42 m

A tent is of radius 3 m and its slant height is 5 m. Find the canvas required in sq m.
1. 15 sq m
2. 8 sq m
3. 47.10 sq m
4. 42.70 sq m

25.

How many persons can the tent accommodate, at the most, if each person requires 18m 3 of air, if diameter is 12 m at
the base and its height is 8 m?
1. 15
2. 16
3. 17
4. 18

w
w

Two right circular cones of equal volumes have their heights in the ratio of 25 : 16. Find the ratio of their radii.
1. 5 : 4
2. 4 : 5
3. 125 : 64
4. None

26.

.B

24.

27.

The volumes of two cones are in the ratio of 4 : 5 and their heights are in the ratio of 16 : 125. Find the ratio of their
radii.
1. 25 : 8
2. 5 : 2
3. 25 : 4
4. 8 : 25

28.

The heights of two cones are in the ratio of 3 : 2 and their radii are in the ratio of 5 : 4. Find the ratio of their
volumes.
1. 75 : 32
2. 45 : 16
3. 15 : 8
4. 225 : 64

29.

The volumes of two cones are in the ratio of 2 : 7 and their diameters are in the ratio of 3 : 5. Find the ratio of their
heights.

Get all banking study material on BankExamsToday.com

Page 121

www.BankExamsToday.com, best study material for bank exams


CRT
1. 63 : 50

2. 50 : 63

3. 6 : 35

4. 10 : 21

3. 38088 cm3

4. 30808 cm3

30.

The radius of a sphere is 21 cm. Find its volume.


1. 38808 cm3
2. 38880 cm3

31.

Find the curved surface area of the hemisphere, if its diameter is 56 cm.
1. 2464 sq cm
2. 9856 sq cm
3. 4829 sq cm

32.

The length, breadth and height of a Cuboid are increased by 25%, 20% and 33 1/3%. Find the percentage increase in
the volume.
1. 78 1/3%
2. 100%
3. 78%
4. None

33.

The sides of a cube are decreased by 20% each. Find the percentage decrease in the volume.
1. 48.8%
2. 40%
3. 22%
4. None

34.

The diameter of a sphere is increased by 20%. Find the percentage increase in its volume.
1. 172.8%
2. 800%
3. 72.8%
4. None

35.

The radius of a right circular cylinder is decreased by 5% and the height is increased by 10%. What is the percentage
change in its volume?
1. 0.725% decrease 2. 0.725% increase
3. 7.25% decrease
4. None

36.

The radius of a hemisphere is decreased by 10%. Find the percentage decrease in the surface area.
1. 18%
2. 19%
3. 81%
4. None

37.

Each edge of a cube is increased by 30%. What is the percentage increase in its surface area?
1. 96%
2. 31%
3. 169%
4. 69%

38.

The radius and height of a cylinder are increased by 20% and 15%. Find the percentage increase in the surface area.
1. 38%
2. 35%
3. 5%
4. 45%

39.

Find the number of lead balls of radius 1 cm each that can be made from a sphere of radius 5 cm.
1. 25
2. 5
3. 125
4. None

40.

A sphere of maximum possible volume is carved out of a cubical wooden block. Approximately what percentage of
the wood is unused?
1. 48%
2. 50%
3. 96%
4. None

w
w

Answers

.B

an
k

Ex

am
sT

od
ay
.

co
m

4. 4928 sq cm

1) 3
2) 1
3) 2
4) 4
5) 3
6) 4
7) 1
8) 4
9) 2
10) 4

11) 1
12) 2
13) 2
14) 1
15) 2
16) 3
17) 1
18) 4
19) 2
20) 2

Get all banking study material on BankExamsToday.com

21) 1
22) 4
23) 1
24) 3
25) 2
26) 2
27) 2
28) 2
29) 2
30) 1

Page 122

31) 4
32) 2
33) 1
34) 3
35) 1
36) 2
37) 4
38) 1
39) 3
40) 1

www.BankExamsToday.com, best study material for bank exams


CRT

Interest

Read this please:

co
m

Raju borrowed Rs.1000 from Ramesh. After some days Raju paid back Rs.1200 to Ramesh. Means Raju paid
Rs.200 excess than the amount he borrowed from Ramesh.

od
ay
.

This excess amount is called interest.


The total amount of money borrowed by Raju from Ramesh is called the Principal or Sum.
The money paid back to Ramesh, which is the combination of both Principal and interest is called
the Amount.
So, Amount = Principal + Interest
Rate of interest: The interest is usually charged according to a specified term, which is expressed as some per
cent of the principal and is called the rate of interest for the fixed period of time.
the
the
the
the

fixed
fixed
fixed
fixed

period
period
period
period

is
is
is
is

a year, the rate of interest is charged annually.


six months, the rate of interest is charged semi -annually.
three months, the rate of interest is charged quarterly.
a month, the rate of interest is charged monthly.

am
sT

If
If
If
If

Example: If the rate of interest is 10% per annum, then the interest payable on Rs.100 for one year is Rs.10.
Simple Interest: When the interest is payable on the principal only, it is called the simple interest. It is the
interest calculated on the principal for the entire period it is borrowed. It is denoted by S.I.

an
k

Ex

Example: S.I. on Rs.100 at 10% per annum will be Rs.10 each year.
At the end of one year, the total amount will be Rs.100+10 = Rs.110.
At the end of second year, the total amount will be Rs.100+10+10 = Rs.120 and so on.
Formulae:
1.
If P stands for Principal, R the rate per cent per annum, T the number of years, A the amount and S.I. the
simple interest then

3.

100 S.I.
R T
100 S.I.
R
%
P T
100 S.I.
T=
years
P R
RT
Amount, A P 1
100
P

4.

T R
100

w
w

2.

.B

S.I. =

5.

100 A
.
100 (R T)

6.

If a certain sum in T years at R% per annum amounts to Rs.A, then the sum will be P

7.

The annual payment that will discharge a debt of Rs.A due in T years at R% per annum =
100 A
.
RT(T 1)
100T
2

Get all banking study material on BankExamsToday.com

Page 123

www.BankExamsToday.com, best study material for bank exams


CRT
If a certain sum is invested in n types of investments in such a manner that equal amount is obtained on
each investment where interest rates are R 1 ,R 2 ,.,R n respectively and time periods and time periods are
T 1 ,T 2 ,., T n respectively, then the ratio in which the amounts ar e invested is

8.

1
1
1
:
: ........................
.
100 R1T1 100 R 2 T2
100 RnTn

If a certain sum of money becomes n times itself in T years at S.I., then the rate of interest per annum is,
100(n
T

1)

%.

co
m

9.

If a certain sum of money becomes n times itself at R% per annum S.I. in T years, the n T

10.

100(n
R

1)

years.
11. If a certain sum of money becomes n times itself in T years at a simple interest, then the time T ' in which

If the rate of interest (R) changes from R 1 to R 2 and P, T are constant, then

12.

Change in S.I. =

PT
100

R2) .

(R 1

If Principal (P) changes from P 1 to P 2 and R, T are constant, then


Change in S.I. =

RT
100

P2 ) .

(P1

If Rate (R) Changes from R 1 to R 2 and time (T) changes from T 1 to T 2 but principal (P) is constant, then

14.

Change in S.I. =

P
(R1T1
100

R2T2 ) .

If a debt of Rs.X is paid in n number of installments and if the value of each installment is a, then the

15.

borrowed (debt) amount is given by X

na

Ra
100 b

n(n 1)
.
2

b
b
b
b

=
=
=
=

Ex

Where, R is the rate of interest per annum


b is the no. of installments per year

1, when each installment is paid yearly.


2, when each installment is paid half-yearly.
4, when each installment is paid quarterly.
12, when each installment is paid monthly.

an
k

Note:

.B

If a certain sum of money P lent out at S.I. amounts to A 1 in T 1 years and to A 2 in T 2 years, then,
A1T2 ~ A 2 T1
A1 ~ A 2
100%
and R =
T2 ~ T1
A1T2 ~ A 2 T1

w
w

P=

17.

am
sT

13.

16.

T years.

od
ay
.

m 1
n 1

it will become m times itself is given by T' =

If a certain sum of money P lent out for a certain time T amounts to A 1 at R 1 % per annum and to A 2 at R 2 %
per annum, then
A1 ~ A 2
A 2R1 ~ A1R 2
100years.
and T =
R1 ~ R 2
A 2R1 ~ A1R 2

P=

18.

If an amount P 1 lent at S.I. rate of interest R 1 % p.a. and another amount P 2 at S.I. rate of R 2 % p.a., then the
rate of interest for the whole sum is R

19.

P1R 1
P1

P2R 2
.
P2

If a certain sum of money is lent out in n parts in such a manner that equal sum of money is obtained as
S.I. on each part where interest rates are R 1 ,R 2 ,,R n respectively and time periods are T 1 ,T 2 ,..,T n
respectively, then the ratio in which the sum will be divided in n parts is given by

Get all banking study material on BankExamsToday.com

Page 124

www.BankExamsToday.com, best study material for bank exams


CRT
1
1
1
:
: ........................
.
R1T1 R 2 T2
RnTn

If there is a change in principal (P), rate of interest (R) and time (T), then the value of S.I also changes and
is given by
P1
P2

A1
A2

P1
P2

P1
P2

Out of a certain sum P,


say,

T1
T2
R1
R2

T1
T2

1
1
part is invested at R 1 %, part is invested at R 2 % and the remainder 1
y
x

A
R1
x

100
R3
R2
y
z

Examples:

Find the S.I. on Rs.3500 for 3 years at 5% per annum.


Sol: Here, P = Rs.3500, T = 3 years and R = 5%
Rs.525.

Find the Principal, if S.I. = 624 for 2 years at 3% per annum.


100 S.I.
R T

100 624
2 3

an
k

Sol: P

Rs.10400.

At what rate per annum will a sum of Rs.4000 amount to Rs. Rs.6000 in 5 years?

.B

3.

3500 3 5
100

Ex

S.I. =
2.

Sol: S.I. = 6000 4000 = 2000

Sol: T

5.

100 2000
4000 5

10%.

In what time will Rs.1500 earn an interest of Rs.150 at 2% per annum?

4.

100 S.I.
P T

w
w

1
y

1
part at R 3 %. If the annual income from all these investments is Rs.A, the original sum is given by
z
P

1.

1
x

od
ay
.

21.

R1
R2

co
m

S.I1
S.I2

am
sT

20.

100 S.I.
P R

100 150
1500 2

5years.

Sunny borrowed Rs.2000 from his friend Praveen at 10% per annum for 5 years. Find the interest and
money returned by Sunny to Praveen.
Sol: S.I. =

2000 5 10
1000
100

A = P + S.I. = 2000 + 1000 = Rs.3000

Get all banking study material on BankExamsToday.com

Page 125

www.BankExamsToday.com, best study material for bank exams


CRT

Sol: P

7.

1
years?
2
100 420
100 420
5
105
100 (2
)
2

What sum will amount to Rs.420 at 2% per annum in 2


100 A
100 (R T)

100 420
1
100 (2 2 )
2

Rs.400

Find the annual installment that will discharge a debt of Rs.16200 in 5 years at 4% per annum.
100 A
RT(T 1)
100T
2
100 16200
100 16200
=
4 5 (5 1)
80
100(5)
500
2
2

Sol: Annual installment =

Rs.3000.

od
ay
.

8.

100 16200
540

co
m

6.

A sum of Rs.3454 is divided among three such parts that amount obtained on these three parts of money
after 3, 4 and 5 years, respectively at rate of 5% per annum remains equal. Find such three parts of the
sum.
Sol: The three parts will be in the ratio

am
sT

1
1
1
:
: ........................
100 R1T1 100 R 2 T2
100 RnTn

1
1
1
:
:
1 0 0 (5 3) 1 0 0 (5 4) 1 0 0 (5 5)

1 69000 1 69000 1 69000


:
:
115
120
125

1
1
1
:
:
115 120 125

600:575:552

[Since the L.C.M. of 115, 120 and 125 is 69000.]

Ex

Ratio = 600: 575: 552


Sum of proportional = 600 + 575 + 552 = 1727
600
3454 Rs.1200
1727
575
3454 Rs.1150
2 nd part =
1727
552
3454 Rs.1104.
3 rd part =
1727

A certain sum of money quadruples itself in 6 years S.I. Find the rate percent per annum.
Sol: R =

Sol: T

11.

1)

%=

100(4
6

n 1
100 years
R

= 50%.

(2 1)
4
100 16
6
6

16

2
years
3

A sum of money put on S.I. doubles itself in 10 years. In how many years would it quadruple itself?
Sol: Required Time, T ' =

12.

1)

In what time a sum of money will double itself at a rate of S.I. of 6% per annum?

10.

100(n
T

w
w

9.

.B

an
k

1 st part =

4 1
2 1

10 = 30 years.

If simple interest on Rs.500 increases by Rs.20, when the rate % increases by 8% per annum, find the time.
Sol: Change in S.I. =

PT
100

(R 1

R2)

Get all banking study material on BankExamsToday.com

Page 126

www.BankExamsToday.com, best study material for bank exams


CRT
20

1
years.
2

RT
(P1 P2 )
100
4R
(3500 2000)
100

60

4R
1500
100

1%.

co
m

60

If the S.I. on a certain sum at 3% per annum for 5 years is Rs.90 more than the interest on the same sum
for 2 years at 6% per annum. Find the sum.
Sol: Change in S.I. =

P
(R1T1
100

P
100

90
15.

40T

If the S.I. on Rs.3500 be more than the interest on Rs.2000 by Rs.60 in 4 years, then find the rate per cent
per annum.
Change in S.I. =

14.

20

(3 5

R2T2 )

6)

90

P
100

od
ay
.

13.

500T
100

Rs.3000.

A sum of Rs.4 is lent to be paid back in 3 equal monthly installments of Re.1 each. Find the rate percent.
Sol: Here, X = Rs.4, a = Re.1, n = 3, b = 12, R = ?
na

Ra
100 b

R 1
100 12
R
4 3
3
1200
1200 R
4
400
1200 R 1600
R 400%.
3(1)

3(3 1)
2

16.

an
k

Ex

n(n 1)
2

am
sT

Requiredformula, X

Anitha deposits Rs.8000 in S.B.I at 2% per annum and Rs.6000 at 5% per annum in ICICI bank. Find the
rate of interest for the whole sum.
P1R 1
P1

P2R 2
P2

8000 2 6000 5
8000 6000
19000
5
1
%.
14000
14

If a sum of Rs.3800 is divided into two such parts that the S.I. on the first part for 1

17.

w
w

.B

Sol: Required rate, R

1
years at the rate of
2

2% per annum, equals the S.I. on the second part for 4 years at the rate of 4% per annum, then find two
such divisions of the sum.
Sol: Required Ratio =

1 st part: 2 nd part =

1
1
2 1
2

1
1
:
R1T1 R 2 T2

1
4 4

1 1
:
3 16

16 : 3

Sum of proportionals = 16 + 3 = 19

Get all banking study material on BankExamsToday.com

Page 127

www.BankExamsToday.com, best study material for bank exams


CRT
So, 1 st part =
2 nd part =

3
3800 Rs.600 .
19

If Rs.50 amounts to Rs.85 in 2 years, what Rs. 340 amount to in 15 years at the same rate?
A1
A2

P1
P2

85 50
A2 340

P1 R1
P2 R2

85 R 2
340 R 15

35
A2 340
A2

T1
we get
T2

170
340 15

35
A2 340

1
2 15

340 30 35 1050

A2 1050 340 1390


19.

co
m

Sol: By using,

From a certain sum Krishna invested,

od
ay
.

18.

16
3800 Rs.3200
19

1
1
th at 2%,
th at 4% and the rest at 6%. If the annual income of
4
5

1
x

1 1
,
4 y

1
1
and
z
5

1
x

1840 100
2 4
6
4 5 20
11

R1
x
1
y

100
R3
R2
y
z
1
4

1
5

20 5 4
20

11
20

an
k

Original sum =

Ex

Sol: Required formula is, P

am
sT

Krishna is Rs.1840 from all these investments, then find the original sum.

Find the S.I. on Rs.500 at 6% p.a. from 3 rd May to July 15th on the same year.
1. Rs.9
2. Rs.6
3. Rs.4

4. None

1.

w
w

Exercise

.B

1840 100
1840 100
2 4 66
10 16 66
4 5 20
20
1840 100
20 Rs.40,000.
92

2.

Uday borrowed a Sum of Rs.10000 from a bank for 6 years at 8% p.a. Find the amount returned by him.
1. Rs.14800
2. Rs.12600
3. Rs.13300
4. None

3.

The Principal that will yield Rs.60 as S.I. at 6% p.a. in 5 years is ________.
1. Rs.175
2. Rs.350
3. Rs.200

4.

Calculate the Sum of money that will produce Rs.1700 interest in 7 years at 8% S.I. p.a.
1. Rs.2950
2. Rs.3120
3. Rs.2800
4. None
If the S.I. on a certain Sum of money after 6 years is 3/8 of the Principal, then find the rate% p.a.
1. 5%
2. 6%
3. 4%
4. None

5.

Get all banking study material on BankExamsToday.com

Page 128

4. None

www.BankExamsToday.com, best study material for bank exams


CRT
Ramesh borrowed Rs.5000 from Ganesh at S.I. If Ganesh got Rs.500 more than his capital after 5 years, then find the
rate% p.a.
1. 2%
2. 3%
3. 4%
4. None

7.

The rate% at which Rs.1200 amounts to Rs.1400 in 4 years is _________.


1. 5%
2. 4%
3. 6%
4. None
The S.I. on a certain Sum of money is Rs.256 and the rate of interest p.a. equals the number of years, then find the
rate% p.a.
1. 13%
2. 14%
3. 16%
4. Data insufficient

8.

co
m

6.

If S.I. on a certain Sum of money for 2 years is 1/5 th of the Sum then find the rate% p.a.
1. 9%
2. 10%
3. 8%
4. Data insufficient

10.

If S.I. on a certain Sum of money for 2 years is 4/25 th of the Sum and the rate% is same as the time period, then find
the rate% p.a.
1. 2%
2. 3%
3. 4%
4. None

11.

If a certain Sum of money borrowed at 5% p.a. S.I. amounts to Rs.1020 in 4 years, then find the Sum of money
borrowed.
1. Rs.850
2. Rs.925
3. Rs.750
4. None

12.

In what time will Rs.1200 amounts to Rs.1344 at 6% p.a?


1. 2 years
2. 3 years
3. 2 years

13.

In what time will Rs.8100 produce the same income at 3% as Rs.225 in 4 years at 3%?
1. 1/7 years
2. 1/9 years
3. 1/6 years
4. None

14.

If Rs.1000 be invested at 5% and the interest be added to the Principal every 10 years then find the time in which it
will amount to Rs.2000.
1. 16 2/3 years
2. 16 years
3. 1/6 years
4. None

15.

If Rs.500 amounts to Rs.725 at 9% S.I. in some time, what will be Rs.600 amount to at 11% in the same time?
1. Rs.870
2. Rs.930
3. Rs.910
4. None

16.

Sanjay lends Rs.10000 for 2 years at 20% p.a. S.I. After 1 year, he gets Rs.6000. How much will he get in the next
year?
1. Rs.5900
2. Rs.6400
3. Rs.7200
4. Data insufficient

17.

What Principal will amount to Rs.15000 at 10% p.a. in 5 years?


1. Rs.10,000
2. Rs.8700
3. Rs.10,500

am
sT

w
w

.B

an
k

Ex

4. None

4. Data insufficient

The annual payment that will discharge a debt of Rs.47,250 due in 3 years at 5% S.I. is _____
1. Rs.8000
2. Rs.10000
3. Rs.15000
4. None

18.

od
ay
.

9.

19.

Find the annual installment that will discharge a debt of Rs.4200 due in 5 years at 10% S.I.
1. Rs.700
2. Rs.750
3. Rs.800
4. None

20.

If the amount obtained by Mahesh by investing Rs.1500 for 2 years at 8% p.a. is equal to the amount obtained by
Suresh by investing a certain Sum for 2 years at 5% p.a. S.I., then the find the Sum invested by Suresh.
1. Rs.1636 4/11
2. Rs.1636
3. Rs.1636 1/2
4. None

21.

A man invests Rs.3965 in the names of his three daughters Nitu, Lekha and Ratna in such a way that they get the
same amount after 2, 3 and 4 years respectively. If the rate of interest is 5% p.a. then find the amounts invested for
Nitu, Lekha and Ratna.
1. Rs.1380, Rs.1320, Rs.1265
2. Rs.1330, Rs.1360, Rs.1380

Get all banking study material on BankExamsToday.com

Page 129

www.BankExamsToday.com, best study material for bank exams


CRT
3. Rs.1265, Rs.1320, Rs.1340

4. None of these

A Sum of money at S.I. becomes 4 times in 24 years. Find the rate% p.a.
1. 13 %
2. 12 %
3. 11 %

4. None

23.

In how many years will a Sum of money triples itself at 10% p.a. S.I.?
1. 15 years
2. 19 years
3. 20 years

4. None

24.

A Sum of money doubles itself in 8 years. In how many years will it triple?
1. 16 years
2. 15 years
3. 14 years

4. None

25.

A Sum of money put at S.I. at a certain rate for 4 years. Had it been put at 2% higher rate, it would have fetched
Rs.56 more. Find the Sum.
1. Rs.680
2. Rs.700
3. Rs.720
4. None

26.

If the interest on Rs.800 be more than the interest on Rs.400 by Rs.40 in 2 years then find the rate% p.a.
1. 5%
2. 5 %
3. 6%
4. None

27.

If the difference between S.I. on a certain Sum for 4 years at 2 % p.a. and the S.I. on the same Sum for the same
period at 3% p.a. is Rs.60 then find the Sum.
1. Rs.3000
2. Rs.2900
3. Rs.3100
4. None

28.

If a certain Sum of money at S.I. amounts to Rs.2800 in two years and Rs.3250 in 5 years, then find rate % p.a.
1. 4%
2. 6%
3. 5%
4. None

29.

If a certain Sum of money amounts to Rs.1760 in two years and Rs.2000 in 5 years at S.I. then find the Sum.
1. Rs.1960
2. Rs.1590
3. Rs.1600
4. None

30.

If a certain Sum is invested for a certain time, it amounts to Rs.450 at 7% p.a. but when invested at 5% p.a. it
amounts to Rs.350 then find the Sum.
1. Rs.60
2. Rs.100
3. Rs.120
4. None

31.

A certain Sum is invested for 40 years, it amounts to Rs.400 at 10% p.a. but when invested at 4% p.a. it amounts to
Rs.200. Find the time.
1. 36 years
2. 37 years
3. 37 years
4. 36 years

32.

If a Sum of Rs.20 is lent to be paid back in 10 equal month installments of Re.1 each then find the rate%.
1. 266 2/3 %
2. 265 %
3. 266%
4. None

od
ay
.

am
sT

Ex

an
k

.B

A Sum of Rs.7700 is to be divided among three brothers Veeru, Vinay and Ram in such a way that S.I. on each part
at 5% p.a. after 1, 2 and 3 years respectively remains equal. The share of Veeru is more than that of Vinay by ____.
1. Rs.2800
2. Rs.2500
3. Rs.3000
4. None
If S.I. on a certain Sum of money for 4 years at 5% p.a. is same as the S.I. on Rs.560 for 10 years at the rate of 4%
p.a. then find the Sum.
1. Rs.1190
2. Rs.1120
3. Rs.1210
4. None

34.

w
w

33.

co
m

22.

35.

Mr. Hari invested an amount of Rs.12000 at S.I. rate of 10% p.a. and another at 20%. The total interest at the end of
one year on the total amount invested became 14% p.a. Find the total amount invested.
1. Rs.20,000
2. Rs.20,800
3. Rs.21,000
4. None

36.

Mr. Gupta deposits Rs.3000 in a bank at 10% p.a. and Rs.5000 in another bank at 8% p.a. Find the rate% for the
whole Sum.
1. 8 %
2. 8 %
3. 8%
4. None

Get all banking study material on BankExamsToday.com

Page 130

www.BankExamsToday.com, best study material for bank exams


CRT
A person invested 2/3rd of his capital at 3%, 1/6th at 6% and the remaining at 12%. If his annual income is Rs.25 then
find the capital.
1. Rs.490
2. Rs.510
3. Rs.500
4. None

38.

The S.I. on a Sum of money will be Rs.600 after 10 years. If the Principal is trebled after 5 years, what will be the
total interest at the end of 10th year?
1. Rs.1200
2. Rs.1190
3. Rs.1210
4. None

39.

Rs.1500 is invested at 10% S.I. and interest is added to the Principal after every 5 years. In how many years will it
amount to Rs.2500?
1. 6 1/9 years
2. 6 years
3. 7 years
4. None

40.

Anitha lends some money to Navitha at 5% p.a. S.I. Navitha lend the amount to Radhika on the same day at 8 %
p.a. In this transaction after a year Navitha earned a profit of Rs.350. Find the Sum of money lent by Anitha to
Navitha.
1. Rs.9000
2. Rs.10,000
3. Rs.10,200
4. None

41.

Poornima borrowed Rs.1000 to build a hut. She pays 5% S.I. She lets the hut to Srikanth and receives the rent of
Rs.12 per month from him. In how many years, Poornima ought to clear off the debt?
1. 10 years
2. 10 years
3. 10 years
4. None

42.

The rate of interest on a Sum of money is 4% p.a. for the first two years and 6% p.a. for the next 4 years, 8% p.a. for
the period beyond 6 years. If the S.I. occurred by the Sum for a total period of 9 years is Rs.1120 then find the Sum.
1. Rs.2400
2. Rs.2200
3. Rs.2000
4. None

43.

Find the simple Interest on Rs.5000 for 219 days at 10% p.a.
1. Rs.300
2. Rs.250
3. Rs.350

44.

Mr. Shyam borrowed Rs.25000 from a bank at the rate of 6% p.a. What is the interest paid by him at the end of the
5th year if simple interest is calculated?
1. Rs.7000
2. Rs.7500
3. Rs.7250
4. None

45.

The simple interest on a sum of money is 1/9 of the principal, and the number of years is equal to the rate % p.a. Find
the rate % p.a.

4. None

w
w

Answers

2. 3

.B

1. 2%

an
k

Ex

am
sT

od
ay
.

co
m

37.

1) 2
2) 1
3) 3
4) 4
5) 2
6) 1
7) 4
8) 4
9) 2
10) 4

11) 1
12) 3
13) 2
14) 1
15) 2
16) 3
17) 1
18) 3
19) 1
20) 1

1
%
3

21) 1
22) 2
23) 3
24) 1
25) 2
26) 1
27) 1
28) 2
29) 3
30) 2

Get all banking study material on BankExamsToday.com

3. 2.5%

4. 3%

31) 3
32) 1
33) 4
34) 2
35) 1
36) 2
37) 3
38) 1
39) 1
40) 2

Page 131

41) 1
42) 3
43) 1
44) 2
45) 2

www.BankExamsToday.com, best study material for bank exams


CRT

Compound Interest: In this method, the interest for each period is added to the principal before; interest is
calculated for the next period. So, the principal grows as the interest is added to it. It is denoted by C.I.

1.

co
m

Formulae:

If a principal P is given on C.I. at the rate of interest R% p.a., then the Amount A after t years is
t

C.I. = A P
= P1

3.

R
.
100

R
100

R
100

P 1

A
P

Rate of interest (R) = 100

1
1

1 %p.a.

am
sT

2.

P1

od
ay
.

Note: S.I. and C.I. for 1 year at a given rate of interest per annum are always equal.
If the interest is compounded half-yearly, then
a) Amount, A
b) C.I. = P 1

P1

R
100 2

R
100 2

2t

1 %p.a.

If the interest is compounded quarterly, then


P1

R
100 4

.B

a) Amount, A

R
100 4

w
w

b) C.I. = P 1

4t

A
P

c) Rate (R) = 4 100

6.

1
2
t

an
k

5.

1 .

A
P

c) Rate (R) = 2 100

2t

Ex

4.

4t

1 .
1
4
t

1 %p.a.

If the interest is compounded n times a year, then


R
100 n

a) Amount, A

P1

b) C.I. = P 1

R
100 n

c) Rate (R) = n 100

n t

A
P

n t

.
1 .

1
n
t

1 %p.a.

Get all banking study material on BankExamsToday.com

Page 132

www.BankExamsToday.com, best study material for bank exams


CRT
If the rate of interest is different for different years, say R 1 %, R 2 %, R 3 % for first, second and third years
respectively, then

If the time is in the form of fraction, say

Amount, A

P 1

b) C.I. S.I. =

R
100

R
100

S.I.
3

R
100

R
100

R
100

if P and R are given

if S.I. and R are given

If a certain sum becomes n times in t years at C.I.,


a) Then the same sum becomes n t in mt years.
b) Rate of C.I., R

100

1
(n) t

1 %.

an
k

If a certain sum of money at C.I. amounts to Rs.x in A years and to Rs.y in B years, then th e rate of
interest p.a. is
y
x

1
B A

100%.

If a loan of Rs.P at R% C.I. p.a., is to be repaid in n equal yearly installments, then the value of each

.B

13.

if P and R are given

The difference between the C.I. and the S.I. on a certain sum of money for 3 years at R% p.a., is

b) C.I. S.I. =

12.

R S.I.
if S.I. and R are given
2 100

a) C.I. S.I. = P

11.

The difference between the C.I. and the S.I. on a certain sum of money for 2 years at R% p.a., is
a) C.I. S.I. = P

10.

y
years, then
z

y
R
z
.
100

R
100

od
ay
.

9.

R3
.
100

R2
1
100

am
sT

8.

R1
1
100

P1

co
m

Amount, A

Ex

7.

installment = Rs.

w
w

100
100 R

100
100 R

........

100
100 R

Examples

Ravi invested Rs.25000 at C.I. rate 4% p.a., for a period of 3 years. What amount will he receive at the end
of 2 years?

1.

Sol: P = Rs.25000, R = 4%, t = 3 years, A =?


A

P1

R
100

25000 1

2.

4
100

25000

26
25

26
25

26
25

Rs.28,121.60.

Find the C.I. on Rs.2000 for 2 years at 5% p.a.


Sol: P = Rs.2000, R = 5%, t = 2 years, C.I. =?

Get all banking study material on BankExamsToday.com

Page 133

www.BankExamsToday.com, best study material for bank exams


CRT
C.I.

R
100

= P 1

5
100

= 2000 1

2000(1.025 1)

2000(0.1025)

1
Rs.205.

Ram invested Rs.5548 for 3 years at C.I. and received an amount of Rs.6750 on maturity. What is rate
percent?

co
m

3.

21
20

2000

15
14

100

1
3

3375
100
2774

1%

1
3 3

15
14

1
3

1%

1%

100

1%

1
%.p.a.
7
1
Find the amount of Rs.16000 in 1 years at 10% p.a., C.I. payable half-yearly.
2
1
3
Sol: P = Rs.16000, R = 10%, t = 1 =
years, A =?
2
2

P1

2t

R
100 2

an
k

Amount, A

50
7

1 %p.a.

Ex

100
14

10
100 2

.B

16000 1
16000

(21)3
8000

3
2

16000

21
20

2 9261 Rs.18,522.

Find the C.I. on Rs.8192 at 50% p.a., compounded quarterly for 1 year.

w
w

5.

6750
5548

100

4.

am
sT

A
P

Rate of interest (R) = 100

od
ay
.

Sol: P = Rs.5548, A = Rs.6750, t= 3 years, R =?

Sol: P = Rs.8192, R = 50%, t = 1 year, C.I. =?

C.I. = P 1
8192 1
8192

6.

R
100 4

50
100 4

4t

4 1

6561 4096
4096

1
8192

8192
2465
4096

9
8

1
Rs.4930

Find the C.I. on Rs.2000 at 12% p.a. for 2 months compounded monthly.
Sol: P = Rs.2000, R = 12%, t = 2 months =

2
12

Get all banking study material on BankExamsToday.com

1
years , C.I. =?
6

Page 134

www.BankExamsToday.com, best study material for bank exams


CRT

2000 1

12 t

R
100 12

20001.0201 1

1
6

2000

101
100

Lakshmi invests Rs.50000 in a mutual fund which gives interest at 2% p.a., during first year, 5% during
the second year and 8% during the third year. How much does she get at the end of the third year?
Sol: P = Rs.50000, R 1 = 2%, R 2 = 5%, R 3 = 8%
R2
1
100

50000 1

2
5
8
1
1
100
100
100

51 21 27
50 20 25

Rs.57834.

1
years at 4% p.a.?
2

1
years, R = 4%.
2

R
100

= P 1

4
100

26
25

26
25

an
k

93750

93750

93750 3226
31250

.B

y
R
z
100

Ex

Required C.I.

1
4
2
100

51
1
50

Rs.9678

w
w

Find the difference between C.I. and S.I. on a sum of Rs.6250 put for 2 years at 4% p.a.
Sol: P = Rs.6250, t = 2 years, R = 4%

For 2 years, C.I. S.I. = P

10.

50000 28917
25000

am
sT

What will be the C.I. on Rs.93750 for 2


Sol: P = Rs.93750, t= 2

R3
100

R1
1
100

P1

50000

9.

2000 0.0201 Rs.40.2.

Amount at the third year, A

8.

od
ay
.

7.

12

12
100 12

co
m

C.I. = P 1

R
100

6250

if P and R are given

4
100

6250

1
25

1
25

Rs.10

The difference between C.I. and S.I. on a certain sum of money for 3 years, at 4% p.a. is Rs.76. Find the
sum.
Sol: C.I. S.I. = Rs.76, R = 3%
For 3 years, C.I. S.I. = P
76 = P

R
100

4
100

R
100

4
100

if P and R are given

Get all banking study material on BankExamsToday.com

Page 135

www.BankExamsToday.com, best study material for bank exams


CRT
76 = P

1
25

76 = P

1
25

1
25

1
25

1 (3 25)
25 25 25

76
15625

76 = P

Rs.15625.

Certain sum of money placed at C.I. doubles itself in 3 years. In how many years will it amount to eight
times itself?

od
ay
.

11.

co
m

76 = P

Sol: Here, n = 2, t = 3 years and m = 3


Then the same sum becomes nm in mt years = 3 x 3 = 9 years.
12.

At what percent C.I. does a sum of money become eight -fold in 3 years.

13.

100 (n) t

100 (8)3

100%.

A certain sum of money at C.I. amounts to Rs.800 in two years and to Rs.1152 in four years. Find the rate
of interest per annum.
Sol: x = Rs.800, y = Rs.1152, A = 2 and B = 4
1
B A

y
x

100%

Ex

Required rate of interest, R

an
k

1152
800

.B
w
w

14.

am
sT

Sol: Required Rate percent is, R

24
20

1
4 2

1
2 2

100%

100%

576
400

1
2

24 20
20

100%

100

1
3

20%.

If a sum of Rs.18120 is to be paid back in two equal installments at 1 % per annum, what is the amount
of each installment?

4
%
3
P

1
3

Sol: P = Rs.18120, R = 1 % =

Each installment = Rs.

100
100 R

100
100 R
18120

18120
2

100
100

100
4
3

100

4
3

Get all banking study material on BankExamsToday.com

100
304
3

Page 136

100
304
3

www.BankExamsToday.com, best study material for bank exams


CRT

18120
300
304

300
304

18120
300 604
304 304

18120
300
300
1
304
304

18120 304 304 304 304


=
10
300 604

Exercise

Rs .9241.60.

co
m

1.

Find the C.I. on Rs.12,000 at 10% for 2 years.


1. Rs.3000
2. Rs.3246

2.

On what sum of money will be C.I. be Rs.328 in 2 years at 5% p.a.?


1. Rs.3200
2. Rs.4000
3. Rs.3500

4. Rs.4500

3.

At what rate% p.a. will Rs.400 amount to Rs.441 in 2 years?


1. 7%
2. 9%
3. 11%

4. 5%

4.

The C.I. on Rs.10,000 for 4 years is Rs.641. Find the rate% p.a.
1. 10%
2. 11%
3. 12%

5.

In what time will Rs.1200 amount to Rs.1323 at 5% p.a. C.I.?


1. 3 years
2. 2 years
3. 6 years

6.

Find the C.I. on Rs.24,000 at 10% p.a. for 1 year 6 months the interest is being compounded half-yearly?
1. Rs.12,000
2. Rs.15,000
3. Rs.14,000
4. None

7.

Find the rate% p.a. if Rs.2000 amount to Rs.2315.25 in 1 years interest being compounded six monthly?
1. 10%
2. 11%
3. 15%
4. 20%

8.

The difference between S.I. and C.I. on a certain sum of money for 2 years at 4% p.a. is 20. Find the sum.
1. Rs.12,500
2. Rs.13,000
3. Rs.14,000
4. None

9.

The difference between S.I. and C.I. is Rs.1395 for 3 years is at 10%. Find the sum.
1. Rs.45,000
2. Rs.90,000
3. Rs.4500
4. None

10.

Find the difference between S.I. and C.I. on Rs.3125 for 3 years at 4% p.a.
1. Rs.15.20
2. Rs.60.00
3. Rs.20

4. 19%
4. 9 years

w
w

.B

an
k

Ex

am
sT

od
ay
.

4. None

4. Rs.22

The C.I. and S.I. on a certain sum of money for 2 years are Rs.816 and Rs.800 at the same rate. Find the sum and the
rate%.
1. Rs.10,000, 6%
2. Rs.4000, 4%
3. Rs.10,000, 4%
4. None

11.

3. Rs.2000

12.

The C.I. on a sum of money for 2 years at 5% p.a. is Rs.246. Find the corresponding S.I.
1. Rs.240
2. Rs.900
3. Rs.680
4. None

13.

At a C.I. a sum becomes double itself in 3 years. In 12 years, it will become how many times?
1. 15 times
2. 16 times
3. 18 times
4. 20 times

14.

At a C.I. a sum becomes double itself in 7 years. In how many years it will become 32 times?
1. 30 years
2. 40 years
3. 35 years
4. 39 years

15.

The annual increase in the price of a TV is 10% if the present value is Rs.15215. What was it 2 years ago?

Get all banking study material on BankExamsToday.com

Page 137

www.BankExamsToday.com, best study material for bank exams


CRT
1. Rs.12000

17.

3. Rs.12500

4. None

A building of worth Rs.81,000 is constructed on a land worth Rs.1,21,000. After how many years will the value of
both the same if the building appreciates at 10% p.a. and the land depreciates at 10% p.a.?
1. 2 years
2. 3 years
3. 5 years
4. 7 years
A sum of Rs.13,360 is borrowed at 8 % p.a. C.I. and paid back in equal installments. What is the value of each
installment?
1. Rs.7579
2. Rs.7569
3. Rs.7000
4. Rs.6500

co
m

16.

2. Rs.15000

What is the nominal % p.a. when interest is payable half-yearly that would give an effective rate of 8% p.a.?
1. 7.8%
2. 8.7%
3. 9%
4. None

19.

A loan was repaid in two annual installments of Rs.121 each. If the rate of interest be 10% p.a. compounded
annually, the sum borrowed was ____.
1. Rs.200
2. Rs.210
3. Rs.300
4. Rs.310

20.

A sum of Rs.8448 is to be divided between A and B who are respectively 18 and 19 years old in such a way that if
their share be invested at 6.25% p.a. C.I. they shall receive equal amounts on attaining the age of 21 years. Find the
present share of each and how much each will receive at the age of 21?
1. Rs.4096, Rs.4352
2. Rs.5000, Rs.3448
3. Rs.6000, Rs.2448
4. None

21.

A sum of money is borrowed and paid in two equal installments of Rs.729 allowing 8% C.I. What was the sum
borrowed?
1. Rs.13,640
2. Rs.12,000
3. Rs.1300
4. None

22.

A sum of money put at C.I. for 2 years at 20%, it would fetch more Rs.482 more, if the interest were payable halfyearly, then if it were payable half-yearly, than if it were payable yearly. Find the sum?
1. Rs.25,000
2. Rs.20,000
3. Rs.14,000
4. None

23.

At what rate% C.I. does a sum of money becomes four-fold in 2 years?


1. 150%
2. 100%
3. 200%

24.

If the C.I. on a certain sum for 2 years is Rs.60.60 and the S.I. is Rs.60 then find the rate of interest p.a.
1. 2%
2. 3%
3. 4%
4. None

25.

If the C.I. on a certain sum for 2 years is Rs.105 and S.I. is Rs.100 then find the sum.
1. Rs.300
2. Rs.400
3. Rs.500
4. None

26.

Find the difference between S.I. and C.I. on Rs.1250 for 2 years at 4% p.a.
1. Rs.3
2. Rs.4
3. Rs.2

am
sT

Ex

w
w

.B

an
k

4. 400%

4. None

On a certain sum of money, the S.I. for 2 years is Rs.200at 7% p.a. Find the difference in C.I. and S.I.
1. Rs.3
2. Rs.4
3. Rs.7
4. None

27.

od
ay
.

18.

28.

The difference of C.I. and S.I. on a certain sum at 5% for 2 years is Rs.1.50. Find the sum.
1. Rs.700
2. Rs.600
3. Rs.500
4. None

29.

Find the difference between the S.I. and C.I. on a certain sum at 3% p.a. for 3 years is Rs.27.27. Find the sum.
1. Rs.12000
2. Rs.15000
3. Rs.10000
4. None

30.

Find the difference between the C.I. and S.I. on Rs.8000 for 3 years at 5% p.a.
1. Rs.61
2. Rs.63
3. Rs.65

31.

If a sum of money at C.I. amounts to thrice itself in 3 years, then in how many years will it be 9 times itself?
1. 9 years
2. 6 years
3. 7 years
4. None

Get all banking study material on BankExamsToday.com

Page 138

4. None

www.BankExamsToday.com, best study material for bank exams


CRT
32.

At what rate% C.I. does a sum of money becomes 16 times in 4 years?


1. 75%
2. 100%
3. 50%

33.

A certain sum of money at C.I. grows up to Rs.12,960 in 2 years and up to Rs.13,176 in 3 years. Find the rate% p.a.
1. 1 1/3%
2. 2 1/3%
3. 1 2/3%
4. None

34.

What sum of money at C.I. will amount to Rs.650 at the end of 1st year and to Rs.676 at the end of the second year?
1. Rs.825
2. Rs.925
3. Rs.625
4. None

35.

A sum of Rs.1260 is borrowed from a money lender at 10% p.a. compounded annually. If the amount is paid back in
two equal installments, find the annual installment.
1. Rs.726
2. Rs.626
3. Rs.526
4. None

36.

A tree increases annually by 1/8th of its height. By how much will it increase after 2 years, if it stands today 64 cm
high?
1. 72 cm
2. 74 cm
3. 78 cm
4. 81 cm

37.

Find the least number of completed years in which a sum of money put out at 20% C.I. will be more than doubled.
1. 3
2. 4
3. 5
4. 6

38.

A man borrows Rs.4000 from a bank at 7 % C.I. At the end of every year, he pays Rs.1500 as part repayment of
loan and interest. How much does he still owe to the bank after three such installments?
1. Rs.123.25
2. Rs.125
3. Rs.400
4. Rs.469.18

39.

If in a certain no. of years Rs.3000 amount to Rs.4320 at C.I., in half that time Rs.3000 will amount to _____.
1. Rs.3400
2. Rs.3600
3. Rs.3800
4. Rs.3520

40.

Rs.3757 is to be divided between A and b such that As share at the end of 7 years be equal to Bs share at the end of
9 years. If rate% be 10% p.a. C.I. then find the Bs share.
1. Rs.1700
2. Rs.1400
3. Rs.1500
4. Rs.2057

w
w

1) 2
2) 1
3) 4
4) 1
5) 2
6) 1
7) 1
8) 1
9) 1
10) 1

.B

Answers

an
k

Ex

am
sT

od
ay
.

co
m

4. None

11) 3
12) 1
13) 2
14) 3
15) 3
16) 1
17) 2
18) 1
19) 2
20) 1

Get all banking study material on BankExamsToday.com

21) 3
22) 2
23) 2
24) 1
25) 3
26) 3
27) 3
28) 2
29) 3
30) 1

Page 139

31) 2
32) 2
33) 3
34) 3
35) 1
36) 4
37) 2
38) 4
39) 2
40) 1

www.BankExamsToday.com, best study material for bank exams


CRT

Clocks

co
m

In a clock the most important hands are the minutes hand and the hours hand. Whatever may be the shape of the dial they
move in a circular track.

od
ay
.

Minute Spaces: The circumference of a dial of a clock or watch is divided into 60 equal parts. These parts are
called minute spaces.
Hands: The clock has two hands. 1. Hour hand 2. Minute hand

Hour hand: The hour hand or short hand indicates time in hours.
In an hour, the hour hand covers 5 minute spaces.

am
sT

Minute hand: The minute hand or long hand indicates time in minutes.
In an hour, the minute hand covers 60 minute spaces.

The total angle of 360 degrees in a watch is divided into 1 sectors, one for each hour.
So one hour sector = 360 / 12 = 30 degrees.

Ex

For every one hour (60 min),

So for every minute,

an
k

The minutes hand moves through 360 deg.


The hours hand moves through 30 deg.

w
w

.B

The minutes hand moves through 6 deg


The hours hand moves through 0.5 deg.
They move in same direction. So their relative displacement for every minute is 5.5 deg.
This 5.5 deg movement constitutes the movements of both the hands.
So for every minute both the hands give a displacement of 5.5 deg.
Note1:

In every hour, both the hands coincide once.


In every hour, the hands are straight (points in opposite directions) once. In this position, the hands are 30
minutes apart.
In every hour, the hands are twice at right angles. In this position, the hands are 15 minute spaces apart.

The minute hand moves through 6 o in each minute whereas the hour hand moves through

Thus, in one minute, the minute hand gains 5

1
in each minute.
2

1
than the hour hand.
2

0o .

When the hands coincide, the angle between them is

When the hands point in opposite directions, the angle between them is 180 .
The hands are in the same straight line, when they are coincident or opposite to each other. So, the angle
between the two hands is

0 o or 180o .

Get all banking study material on BankExamsToday.com

Page 140

www.BankExamsToday.com, best study material for bank exams


CRT

The minute hand moves 12 times as fast as the hour hand.

Note2:

3.

co
m

2.

Between every two hours i.e., between 1 and 2, 2 and 3 and so on the hands of the clock coincide with each other
for one time except between 11, 12 and 12, 1.
In a day they coincide for 22 times.
Between every two hours they are perpendicular to each other two times except between 2, 3 and 3, 4 and 8, 9 and
9, 10.
In a day they will be perpendicular for 44 times.
Between every two hours they will be opposite to each other one time except between 5, 6 and 6, 7.
In a day they will be opposite for 22 times.

od
ay
.

1.

Level 1 Examples:
1.

At what time between 5 and 6 will the hands of the clock coincide?

2.

am
sT

Soln: At 5 the angle between the hands is 150 deg.


To coincide, they collectively have to travel this distance. Every minute they travel 5.5 deg.
So no. of minutes required to coincide = 150 / 5.5 = 300 / 11 = 27 3/11 min.
At what time between 6 and 7 will the hands be perpendicular?

What is the angle between the hands of the clock at 3.45?

an
k

3.

Ex

Soln: At 6 the angle between the hands is 180 deg.


To form 90 deg they have to cover 90 deg (out of 180 if 90 is covered 90 will remain)
So no. of minutes required = 90 / 5.5 = 180 / 11 = 16 4/11 min.
But they will be perpendicular for two times. The second one will happen after the minutes hand crosses the hours
hand and then for 90 deg.
So it has to travel 180+90 = 270 deg.
So time = 270 / 5.5 = 540 / 11 = 49 1/11 min.

.B

Soln: At 3, the angle between the hands = A = 90 deg.


In 45 min the hands will move angle of B = 45 X 5.5 deg (since 5.5 deg for 1 min)
B = 247.5 deg.
Required angle = A ~ B = 157.5 deg.

w
w

4. What is the angle between the hands at 4.40?


Soln: At 4 the angle between the hands, A = 120 deg.
In 40 min, B = 40 X 5.5 = 220 deg.
The required angle = A ~ B = 100 deg.
A clock loses 5 min for every hour and another gains 5 min for every hour. If they are set correct at 10 am on
Monday then when will they be 12 hrs apart?
Soln: For every hour watch A loses 5 min and watch B gains 5 min.
So for every hour they will differ by 10 min.
For 12 hrs (720 min) difference between them the time required = 720 / 10 = 72 hrs
So they will be 12 hrs apart after 3 days i.e., at 10 am on Thursday.

5.

Get all banking study material on BankExamsToday.com

Page 141

www.BankExamsToday.com, best study material for bank exams


CRT

Level 2 Examples
1.

At what time between 6 and 7 O clock are hands of a clock together?


60H
11

60 6
11

360
11

32

8
11

Hands of a clock are together at 32

8
minutes past 6 O clock.
11

At what time between 7 and 8 O clock will the hands of a clock be at right angle?
Sol: Here, H = 7
(5H 15)

12
11

= (5 x 7 15)

od
ay
.

2.

co
m

Sol: Here, H = 6

12
11

6
9
and 21
11
11
6
9
Hands of a clock are at right angle at 54
minutes past 7 and 21
minutes past 7.
11
11
3.

am
sT

= 54

Find at what time between 3 and 4 O clock will the hands of a clock be in the same straight line but not
together.
Sol: Here, H = 3 < 6

12
12
540
= (5 x 3 + 30)
=
11
11
11

49

Ex

(5H + 30)

1
.
11

4.

an
k

The hands will be in the same straight line but not together at 49

1
minutes past 3 O clock.
11

Find the time between 5 and 6 O clock when the two hands of a clock are 5 minutes apart.
Sol: Here, H = 5 and M = 5

12
12
8
9
= (5 x 5 5)
= 32
and 21
11
11
11
11
8
9
The hands will be 5 minutes apart at 32
past and 21
past 5 O clock.
11
11

5.

w
w

.B

(5H M)

Find the angle between two hands of a clock at 25 minutes past 8 O clock.

Sol: Here, H = 8 and M = 25


The required angle = 30 H

M
M
+ degrees
2
5

= 30 8
=
6.

5
25
+ degrees
2
5

205
= 102.5 o .
2

The minute hand of a clock overtakes the hour hand at intervals of 45 minutes. How much a day does the
clock gain or lose?

Get all banking study material on BankExamsToday.com

Page 142

www.BankExamsToday.com, best study material for bank exams


CRT
Sol: Here, M = 45
60 24
M

720
11

45

60 24
45

720
11

45

60 24
45

225
32
11

7200
11

654

6
minutes.
11

od
ay
.

Exercise

minutes

co
m

720
M
11

The clock gains or losses in a day by

At what time between 3 and 4 O clock will the hands of a clock at right angles?
1. 3hr, 32 8/11 hrs. min
2. 3.15hr, 32 11/8 hrs. min
8
3. 3.10hr, 31 /11 hrs. min
4. None

2.

What is the angle between the two hands of a clock when the clock shows 3hrs 25mins?
1. 45 o
2. 46o
3. 46 o
4. 47 o

3.

By how many degrees does an hour hand move in one quarter of an hour?
1. 5o
2. 7.5o
3. 10o

4.

At what time between 6 and 7 oclock, are the hands of a clock together?
1. 6 hours, 32 8/11 minutes
2. 6 hours, 33 6/11 minutes
5
3. 6 hours, 34 /11 minutes
4. 6 hours, 29 7/11 minutes

5.

At what time between 5 and 6 are the hands of a clock co-incident?


1. 22 minutes past 5
2. 30 minutes past 5
3. 278/11 minutes past 5
4. 27 3/11 minutes past 5

6.

In accurate clock shows 8 oclock in the morning. Though how many degrees will the hour hand rotate when the
clock shows 2 oclock in the afternoon?
1. 144o
2. 150o
3. 168o
4. 180o

7.

How many times do the hands of a clock point towards each other in a day?
1. 24
2. 20
3. 21

w
w

.B

an
k

Ex

4. 12.5o

4. 22

At what time between 9 to 10 will the hands of a watch be together?


1. 45 minutes past 9
2. 50 minutes past 9
3. 49 1/11 minutes past 9 4. 48 2/11 minutes past 9

8.

am
sT

1.

9.

The angle between the minute hand and the hour hand of a clock when the time is 8.30 is ?
1. 80o
2. 75o
3. 60o
4. 105o

10.

At what time between 3 and 4o oclock are the hands of a clock in the opposite direction?
1. 3 hours, 48 6/11 minutes
2. 3 hours, 49 1/11 minutes
4
3. 3 hours, 50 /11 minutes
4. 3 hours, 47 2/11 minutes

11.

When the clock shows 3 hours 14 minutes, what is the angle between the hands of the clock?
1. 10o
2. 12o
3. 13o
4. 14o

Get all banking study material on BankExamsToday.com

Page 143

www.BankExamsToday.com, best study material for bank exams


CRT
At what angle the hands of a clock are inclined at 15minutes past 5?
1. 72 o
2. 67o
3. 58 o

4. 64o

13.

How many times in a day, the hands of a clock are on a straight line?
1. 22
2. 24
3. 44

4. 48

14.

The angle between the two hands of a clock is 70 o , when the hour hand is between 7 and 8 what time does the watch
show?
1. 7 hours, 50 10/11 minutes
2. 7 hours, 25 5/11 minutes
8
3. 7 hours, 42 /11 minutes
4. Both (1) & (2)

15.

How many times, the minute hand of clock overlaps with the hour hand from 9.00 am to 4.00 pm in a day?
1. 5
2. 6
3. 7
4. 8

16.

At what time between 5 and 6 oclock will the hands of a clock be at an angle of 62o?
1. 5 hours, 17 2/11 minutes
2. 5 hours, 38 6/11 minutes
3. 5 hours, 16 minutes
4. Both (2) & (3)

17.

At what time between 7 oclock and 8 oclock will both the hands of a clock in same line?
1. 7 Past, 5 5/11 minutes
2. 7 Past, 5 11/5 minutes
5
3. 7 Past, 11 /11 minutes
4. None

18.

At what time, in minutes between 3 oclock and 4 oclock, both the needles will coincide each other?
1. 51/11
2. 124/11
3. 134/11
4. 164/11

19.

At what time between 4 & 5 oclock are the hands of a clock in the opposite directions?
1. 52 3/11 minutes, Past 4 oclock
2. 54 6/11 minutes, Past 4 oclock
7
3. 51 /11 minutes, Past 4 oclock
4. 53 9/11 minutes, Past 4 oclock

20.

A watch, which gains uniformly, was observed to be 4 minutes slow at 6am on a Monday. On the subsequent
Thursday at 7pm it was noticed that the was 6 minutes fast. When did the watch show the correct time?
1. 5pm Tuesday
2. 4pm Tuesday
3. 6pm Tuesday
4. 3pm Tuesday

21.

The angle between the hands of a clock is 20o and the hour hand is in between 2 and 3. What is the time shown by
the clock?
1. 2hrs, 7 3/11 minutes
2. 2hrs, 15 5/11 minutes
6
3. 2hrs, 14 /11 minutes
4. Both (1) & (3)

22.

At what time between 9 and 10 oclock will be hands of a watch be together?


1. 45 minutes past 9
2. 50 minutes past 9
3. 49 1/11 minutes past 9
4. 49 2/11 minutes past 9

od
ay
.

am
sT

Ex

an
k

.B

w
w

Which of the following can be the time shown by the clock when the hour hand is in between 4 and 5 and the angle
between the two hands of the clock is 60o?
1. 16 4/11 minutes past 4
2. 18 9/11 minutes past 4
8
3. 32 /11 minutes past 4
4. 36 5/11 minutes past 4

23.

co
m

12.

24.

The minute hand of a clock overtakes the hour hand at intervals of 66 minutes of the correct time. How much in a
day does the clock gain or lose?
1. 10 113/121 minutes loss
2. 11 115/121 minutes gain
109
3. 11 /121 minutes loss
4. 10 104/121 minutes gain

25.

At what time between 5.30 and 6 will the hands of a clock be at right angles?
1. 43 5/11 minutes past 5
2. 43 7/11 minutes past 5

Get all banking study material on BankExamsToday.com

Page 144

www.BankExamsToday.com, best study material for bank exams


CRT
3. 40 minutes past 5

4. 45 minutes past 5

A clock which gains uniformly is 2 minutes slow at noon on Monday and is 4minutes 48 seconds fast at 2pm on the
following Monday when was it correct?
1. 2 pm on Tuesday
2. 2 pm on Wednesday
3. 3 pm on Thursday
4. 1 pm on Friday

27.

The minute hand of a clock overtakes the hour hand at intervals of 62 minutes of the correct time. How much in a
day does the clock gain or lose?
1. 80 80/341 minutes
2. 80 70/341 minutes
90
3. 80 /341 minutes
4. 80 60/341 minutes

28.

A watch which gains uniformly was observed to be 6 minutes slow at 9.00am on a Tuesday and 3 minutes fast at
12.00noon on the subsequent Wednesday. When did the watch show the correct time?
1. 9.00 am on Tuesday
2. 12.00 am on Wednesday
3. 3.00 am on Wednesday
4. 6.00 am on Wednesday

29.

A watch which gains uniformly was observed to be 5minutes slow at 12noon on Monday. It was noticed 10 minutes
fast at 6.00 p.m. on the next day. When did the watch show the correct time?
1. 9pm the same day
2. 9.30 pm the same day
3. 10.30 pm, the same day
4. 10 pm the same day

30.

At what time between 4 and 4:30 will the hands of a clock be at right angle?
1. 5 5/11 minutes past 4
2. 5 11/5 minutes past 4
11
3. 5 /5 minutes past 5
4. 5 11/5 minutes past 3

31.

A watch showed 5 minutes past 3 oclock on Sunday evening when the correct time was 3 oclock. It loses
uniformly and was observed to be 10minutes slow on the subsequent Tuesday at 9pm when did the watch show the
correct time?
1. 8am Monday
2. 10am Monday 3. 7am Monday 4. 9am Monday

32.

A watch showed 10minutes past 6 oclock on Thursday morning when the correct time was 6 oclock. It loses
uniformly and was observed to be 15minutes slow at 8 oclock on Saturday morning. When did the watch show the
correct time?
1. 1 oclock on Friday afternoon
2. 12 oclock noon on Friday
3. 4 oclock on Friday evening
4. 2 oclock on Friday morning

33.

Find the time between 2 and 3 oclock at which the minute hand and the hour hand are coincide each other?
1. 10 10/11 minutes past 2
2. 12 10/11 minutes past 2
11
3. 10 /10 minutes past 2
4. 27 3/11 minutes past 2

w
w

Find the time between 2 & 3 oclock at which the minute hand and hour hand are on the same straight line but are
facing opposite directions?
1. 43 7/11 minutes past 2 oclock
2. 42 7/11 minutes past 2 oclock
11
3. 43 /7 minutes past 2 oclock
4. None

34.

.B

an
k

Ex

am
sT

od
ay
.

co
m

26.

35.

There are 2 clocks on a wall, both set to show the correct time at 8am. One clock loses two minutes in an hour while
the other gains one minute in one hour. By how many minutes do the two clocks differ at 12noon on the same day?
1. 6minutes
2. 9minutes
3. 12minutes
4. 15minutes

36.

A watch which loses uniformly was observed to be 12 minutes fast at 4am on 6 th of a month. It showed 20minutes
less than the correct time at 6pm on the 10th of the same month when did the watch show the correct time?
1. 9:15am on the 7th
2. 9:05am on the 8th
th
3. 9:35am on the 9
4. 9:20am on the 7th

Get all banking study material on BankExamsToday.com

Page 145

www.BankExamsToday.com, best study material for bank exams


CRT
37.

The minute hand of a clock overtakes the hour hand at intervals of 65minnutes of the correct time. How much a day
does the clock gain or lose?
1. 10 10/43 minutes in 24hours
2. 10 43/10 minutes in 24hours
10
3. 11 /43 minutes in 24hours
4. None

38.

The hands of a correct clock coincide after every ___________


2. 61 min

3. 64

6
min
11

4. 65

5
min
11

co
m

1. 60 min

If a clock takes 33 seconds to strike 12, how much time will it take to strike 6 ______
1. 15
2. 33/2
3. 18
4. 6

40.

A man go out in between 5 p.m. and 6 p.m. When he comes back in between 5 p.m. and 7 p.m. he observes that the
two hands of a clock have interchanged their position. Find when the man did go out?
1. 34

7
min past 5
13

2. 36

5
min past 5
13

3. 32

4
min past 5
13

Answers
21) 4
22) 3
23) 3
24) 3
25) 2
26) 2
27) 1
28) 3
29) 4
30) 1

am
sT

11) 3
12) 2
13) 3
14) 4
15) 2
16) 4
17) 1
18) 4
19) 2
20) 2

w
w

.B

an
k

Ex

1) 1
2) 4
3) 2
4) 1
5) 4
6) 4
7) 4
8) 3
9) 2
10) 2

od
ay
.

39.

Get all banking study material on BankExamsToday.com

Page 146

4. 37

2
min past 5
13

31) 4
32) 4
33) 1
34) 1
35) 3
36) 1
37) 1
38) 4
39) 1
40) 3

www.BankExamsToday.com, best study material for bank exams


CRT

Calendars

co
m

Here you mainly deal in finding the day of the week on a particular given date.
The process of finding this depends on the number of odd days.
Odd days are quite different from the odd numbers.
Odd Days: The days more than the complete number of weeks in a given period are called odd days.
Ordinary Year: An year that has 365 days is called Ordinary Year.

od
ay
.

Leap Year: The year which is exactly divisible by 4 (except century) is called a leap year.
E.g. 1968, 1972, 1984, 1988 and so on are the examples of Leap Years.
1986, 1990, 1994, 1998, and so on are the examples of non leap years.
Note: The Centuries divisible by 400 are leap years.

am
sT

Important Points:

an
k

Explanation:

Ex

An ordinary year has 365 days = 52 weeks and 1 odd day.


A leap year has 366 days = 52 weeks and 2 odd days.
Century = 76 Ordinary years + 24 Leap years.
Century contain 5 odd days.
200 years contain 3 odd days.
300 years contain 1 odd day.
400 years contain 0 odd days.
Last day of a century cannot be Tuesday, Thursday or Saturday.
First day of a century must be Monday, Tuesday, Thursday or Saturday.

.B

100 years = 76 ordinary years + 24 leap years


= 76 odd days + 24 x 2 odd days
= 124 odd days = 17 weeks + 5 days
100 years contain 5 odd days.

w
w

No. of odd days in first century = 5


Last day of first century is Friday.

No. of odd days in two centuries = 3


Wednesday is the last day.

No. of odd days in three centuries = 1


Monday is the last day.
No. of odd days in four centuries = 0
Sunday is the last day.
Since the order is continually kept in successive cycles, the last day of a century cannot be Tuesday,
Thursday or Saturday.
So, the last day of a century should be Sunday, Monday, Wednesday or Friday.
Therefore, the first day of a century must be Monday, Tuesday, Thursday or Saturday.

Get all banking study material on BankExamsToday.com

Page 147

w
w

.B

an
k

Ex

am
sT

od
ay
.

co
m

www.BankExamsToday.com, best study material for bank exams


CRT

Get all banking study material on BankExamsToday.com

Page 148

www.BankExamsToday.com, best study material for bank exams


CRT
Working Rules:
Working rule to find the day of the week on a particular date when reference day is given:
Step 1: Find the net number of odd days for the period between the reference date and the given date
(exclude the reference day but count the given date for counting the number of net odd days) .

co
m

Step 2: The day of the week on the particular date is equal to the number of net odd days ahead of the
reference day (if the reference day was before this date) but behind the reference day (if this date was
behind the reference day).
Working rule to find the day of the week on a particular date when no reference day is given

od
ay
.

Step 1: Count the net number of odd days on the given date

For 0 odd days Sunday


For 1 odd day Monday
For 2 odd days Tuesday
.
.
.
.
.
.
.
.
.
For 6 odd days - Saturday

Examples:

.
.
.

am
sT

Step 2: Write:

Ex

1. If 11 th January 1997 was a Sunday then what day of the week was on 10 th January 2000?

an
k

Sol: Total number of days between 11 th January 1997 and 10 th January 2000
= (365 11) in 1997 + 365 in 1998 + 365 in 1999 + 10 days in 2000
= (50 weeks + 4 odd days) + (52 weeks + 1 odd day) +
(52 weeks + 1 odd day) + (1 week + 3 odd days)
Total number of odd days = 4 + 1 + 1 + 3 = 9 days = 1 week + 2 days

.B

Hence, 10 th January, 2000 would be 2 days ahead of Sunday i.e. it was on Tuesday.

w
w

2. What day of the week was on 10 th June 2008?


Sol: 10 th June 2008 = 2007 years + First 5 months up to May 2008 + 10 days of June

2000 years have 0 odd days.


Remaining 7 years has 1 leap year and 6 ordinary years

2 + 6 = 8 odd days

So, 2007 years have 8 odd days.


No. of odd days from 1 st January 2008 to 31 st May 2008 = 3+1+3+2+3 = 12
10 days of June has 3 odd days.
Total number of odd days = 8+12+3 = 23

23 odd days = 3 weeks + 2 odd days.


Hence, 10 th June, 2008 was Tuesday.

Get all banking study material on BankExamsToday.com

Page 149

www.BankExamsToday.com, best study material for bank exams


CRT

PROBABILITY
Experiment
An operation which results in some well-defined outcomes is called an experiment.

co
m

Random Experiment
An experiment whose outcome cannot be predicted with certainty is called a random experiment. In other words, if an
experiment is performed many times under similar conditions and the outcome of each time is not the same, then this
experiment is called a random experiment.
Example:
a) Tossing of a fair coin
b) Throwing of an unbiased die
c) Drawing of a card from a well shuffled pack of 52 playing cards

od
ay
.

Sample Space
The set of all possible outcomes of a random experiments is called the sample space for that experiment. It is usually
denoted by S.
Example:

am
sT

a) When a die is thrown, any one of the numbers 1, 2, 3, 4, 5, 6 can come up. Therefore. Sample space
S = {1, 2, 3, 4, 5, 6}
b) When a coin is tossed either a head or tail will come up, then the sample space w.r.t. the tossing of the
coin is
S = {H, T}
c) When two coins are tossed, then the sample space is

Ex

Sample point / event point


Each element of the sample spaces is called a sample point or an event point.
Example:
When a die is thrown, the sample space is S = {1, 2, 3, 4, 5, 6} where 1, 2, 3, 4, 5 and 6 are the sample
points.

an
k

Discrete Sample Space


A sample space S is called a discrete sample if S is a finite set.
Event
A subset of the sample space is called an event.

w
w

.B

Problem of Events
Sample space S plays the same role as universal set for all problems related to the particular experiment.

is also the subset of S and is an impossible Event.


S is also a subset of S which is called a sure event or a certain event.

Types of Events
A. Simple Event/Elementary Event
An event is called a simple Event if it is a singleton subset of the sample space S.
Example:
a) When a coin is tossed, then the sample space is
S = {H, T}
Then A = {H} occurrence of head and B = {T} occurrence of tail are called Simple events.
b) When two coins are tossed, then the sample space is
S = {(H,H); (H,T); (T,H); (T,T)}
Then A = {(H,T)} is the occurrence of head on 1 st and tail on 2nd is called a Simple event.

Get all banking study material on BankExamsToday.com

Page 150

www.BankExamsToday.com, best study material for bank exams


CRT
B. Mixed Event or Compound Event or Composite Event
A subset of the sample space S which contains more than one element is called a mixed event or when two or more events
occur together, their joint occurrence is called a Compound Event.

co
m

Example:
When a dice is thrown, then the sample space is
S = {1, 2, 3, 4, 5, 6}
Then let A = {2, 4 6} is the event of occurrence of even and B = {1, 2, 4} is the event of occurrence of exponent of 2 are
Mixed events

od
ay
.

Compound events are of two type:


a) Independent Events, and
b) Dependent Events

C. Equally likely events


Outcomes are said to be equally likely when we have no reason to believe that one is more likely to occur than the other
Example:
When an unbiased die is thrown all the six faces 1, 2, 3, 4, 5, 6 are equally likely to come up.

am
sT

D. Exhaustive Events
A set of events is said to be exhaustive if one of them must necessarily happen every time the experiments is performed.
Example:
When a die is thrown events 1, 2, 3, 4, 5, 6 form an exhaustive set of events.
Important
We can say that the total number of elementary events of a random experiment is called the exhaustive number of cases.

.B

an
k

Ex

E. Mutually Exclusive Events


Two or more events are said to be mutually exclusive if one of them occurs, others cannot occur. Thus if two or more
events are said to be mutually exclusive, if not two of them can occur together.
Hence, A1, A2, A3,, An are mutually exclusive if and only if Ai Aj =
i j
Example:
a) When a coin is tossed the event of occurrence of a head and the event of occurrence of a tail are mutually
exclusive events because we cannot have both head and tail at the same time.
b) When a die is thrown, the sample space is S = {1, 2, 3, 4, 5, 6}
Let
A is an event of occurrence of number greater than 4 i.e., {5, 6}
B is an event of occurrence of an odd number {1, 3, 5}
C is an event of occurrence of an even number {2, 4, 6}
Here, events B and C are Mutually Exclusive but the event A and B or A and C are not Mutually Exclusive.

w
w

F. Independent Events or Mutually Independent events


Two or more event are said to be independent if occurrence or non-occurrence of any of them does not affect the
probability of occurrence of or non-occurrence of their events.
Thus, two or more events are said to be independent if occurrence or non-occurrence of any of them does not influence the
occurrence or non-occurrence of the other events.
Example:
Let bag contains 3 Red and 2 Black balls. Two balls are drawn one by one with replacement.
Let A is the event of occurrence of a red ball in first draw.
B is the event of occurrence of a black ball in second draw.
then probability of occurrence of B has not been affected if A occurs before B. As the ball has been
replaced in the bag and once again we have to select one ball out of 5(3R + 2B) given balls for event B.
G. Dependent Events
Two or more events are said to be dependent, if occurrence or non-occurrence of any one of them affects the probability of
occurrence or non-occurrence of others.
Example:
Let a bag contains 3 Red and 2 Black balls. Two balls are drawn one by one without replacement.
Let A is the event of occurrence of a red ball in first draw
Get all banking study material on BankExamsToday.com

Page 151

www.BankExamsToday.com, best study material for bank exams


CRT

od
ay
.

co
m

B is the event of occurrence of a black ball in second draw.


In this case, the probability of occurrence of event B will be affected. Because after the occurrence of
event A i.e. drawing red ball out of 5(3R + 2B), the ball is not replaced in bag. Now, for the event B, we will have to draw 1
black ball from the remaining 4(2R + 2B) balls which gets affected due to the occurrence of event A.
H. Complementary Events
Let S be the sample space for a random experiment and let E be the event. Also, Complement of event E is denoted by E or
E, where E means non occurrence of event E.
Thus E occurs if and only if E does not occur.
n(E) + n(E) = n(S)
Occurrence of an Event
For a random experiment, let E be an event
Let E = {a, b, c}. If the outcome of the experiment is either a or b or c then we say the event has occurred.
Sample Space : The outcomes of any type
Event
: The outcomes of particular type
Probability of Occurrence of an event
Let S be the same space, then the probability of occurrence of an event E is denoted by p(E) and is defined as
P(E) = n(E)/n(S) = number of elements in E/number of elements in S
P(E) = number of favourable/particular cases
total number of cases

w
w

.B

an
k

Ex

am
sT

Example:
a) When a coin is tossed, then the sample space is S = {H, T}
Let E is the event of occurrence of a head
E = {H}
b) When a die is tossed, sample space S = {1, 2, 3, 4, 5, 6}
Let A is an event of occurrence of an odd number
And B is an event of occurrence of a number greater than 4
A = {1, 3, 5} and B = {5, 6}
P(A) = Probability of occurrence of an odd number = n(A)/n(S) = 3/6 =
and P(B) = Probability of occurrence of a number greater than 4 = n(B)/n(S) = 2/6 = 1/3

Get all banking study material on BankExamsToday.com

Page 152

www.BankExamsToday.com, best study material for bank exams


CRT
PARTNERSHIP
Partnership: Two or more persons carry on a business and share the profits of the business at an agreed proportion.
This is called partnership.

co
m

Partners: Persons who have entered into partnership with one another are individually called partners. They are of
two types.

od
ay
.

a) Sleeping Partner: A person who invests the capital in the business but does not actively participate in the
conduct of business is called sleeping partner.
b) Working Partner: A person who takes part in running the business besides investing the capital is called
working partner. He gets salary for his work or some per cent of profit, in addition.
Firm name: The name under which the business is carried on is called firm name.
Note: The partnership may be simple or compound.

Simple Partnership: It is one in which the capital of each partner is in the business for same time.

am
sT

Compound Partnership: It is one in which the capitals of partners are invested for different periods.
Results:
1.

Three partners A, B and C invested their capitals in a business in the ratio of C1 : C2 : C3 and their profits
are in the ratio P1 : P2 : P3 , then the ratio of timing of their investments is

P1 P2 P3
:
:
.
t1 t 2 t 3

If capitals of two partners A, B be C be Rs. C1 and Rs. C 2 respectively for the periods t1 , t2 and t3
respectively, then
Profit of A : Profit of B : Profit of C = C1t1 : C2t2 :
C3t3

If there is a loss in the business, then


Loss of A : Loss of B : Loss of C = C1t1 : C2t2 :
(Note: Investment Ratio = Profit Ratio)

C3t3

w
w

4.

.B

3.

Ex

Three partners A, B and C invested their capitals in a busi ness. If the timing of their investments is in the
ratio t1 : t2 : t3 and their profits are in the ratio P1 : P2 : P3 , then the ratio of their capitals invested is

an
k

2.

P1 P2 P3
:
:
.
C1 C 2 C 3

Get all banking study material on BankExamsToday.com

Page 153

www.BankExamsToday.com, best study material for bank exams


CRT
SOLVED EXAMPLES
Shashi, Ravi and Kiran invested Rs.2000, Rs.5000 and Rs.4000 respectively in a business. The net profit for
the year was Rs.1210 which was divided in proportion to investments. Find the profit of each.
Sol: Here, C1 = 2000, C2 5000 and C3
Ratio of capitals = 2:5:4
Shashis profit =

2
5

4000 and P = 1210

x 1210 = 220

co
m

1.

5
x 1210 = 550
11
4
Kirans profit =
x 1210 = 440
11

od
ay
.

Ravis profit =

Note: You can solve in this way also.


Profit of Kiran = Total Profit - (Profit of Shashi + Profit of Ravi)

A and B are two partners in a business. A contributes Rs. 1000 for 6 months and B Rs. 600 for 5 months. If
total profit is Rs.600, find the profits of A and B.
Sol: Here, C1

1000, C2

am
sT

2.

600 and t1

6 , t2

5 and P = 600

Profit of A : Profit of B = C 1 t 1 : C 2 t 2
= 6000 : 3000
=2:1
2
x 600 = 400
3
1
Profit of B =
x 600 = 200
3

3.

an
k

Ex

Profit of A =

A, B and C are three partners in a business. A contributes Rs.1500 for 6 months and B Rs.1200 for 4 months
and C Rs.2000 for 2 months. Find the ratio of their shares in the profit.
Sol: Here, C1

1500, C2

1200 and C3

2000 and
C3t3

Raju, Suman and Sunil invested capitals in the ratio of 3 : 4 : 7. At the end of the business term, their profits
are in the ratio 1 : 3 : 5. Find the ratio of time for which they invested the capitals.

4.

w
w

.B

t1 6 , t2 4 and t 3 2
Profit of A : Profit of B : Profit of C = C1t1 : C2t2 :
= 1500(6) : 1200(4) : 2000(2)
= 9000 : 4800 : 4000
= 45 : 24 : 20.

Sol: Here, C1 : C2 : C3 = 3 : 4 : 7 and P1 : P2 : P3


Required ratio =

P1 P2 P3
:
:
C1 C2 C3

1 3 5
: :
3 4 7

Get all banking study material on BankExamsToday.com

1:3:5
28 : 63 : 60

Page 154

www.BankExamsToday.com, best study material for bank exams


CRT
5.

Phani, Deepa and Bindu start a business. If the ratio of their periods of investments are 1 : 4 : 5 and their
profits are in the ratio 4 : 8 : 10, find the ratio of their capitals.
Sol: P1 : P2 : P3

4 : 8 : 10 and t1 : t 2 : t 3 = 1 : 4 : 5

Required ratio =

P1 P2 P3
:
:
t1 t2 t3

4 8 10
: :
1 4 5

2 :1 :1.

co
m

Exercise

4:2:2

A and B started a business by investing Rs.8000 and Rs.6000 respectively. Find the share of each, out of an annual
income of Rs.4200.
1. Rs.2400; 1800
2. Rs.1800; 1500
3. Rs.1900; 1200
4. Rs.1550; 1200

2.

A started a business by investing Rs.4500 after 6 months B joined him by investing Rs.6000. Find the share of each,
out of an annual profit of Rs.5000.
1. Rs.2000; 3000
2. Rs.3000; 1500
3. Rs.3000; 2000
4. Rs.1500; 3000

3.

A, B and C started a business by investing Rs.30,000, Rs.20,000 and Rs.10,000 respectively. After 6 months, A
withdraws Rs.10,000 and B withdraw Rs.5000 and C invests Rs.15,000 more. Find the share of B out of an annual
profit of Rs.4800.
1. Rs.1500
2. Rs.1400
3. Rs.2200
4. Rs.2500

4.

A and B started a business by investing Rs.4000 and Rs.6000 respectively. After 5 months A invested Rs.3000 more
and B withdraws Rs.3000. Find the share of B out of a total annual profit of Rs.4000.
1. Rs.2300
2. Rs.1700
3. Rs.1500
4. Rs.2000

5.

Sharma and Shastri entered into a partnership investing Rs.12,000 and Rs.9,000 respectively. After 3 months Sharma
withdraws Rs.4,000 and Rechal joined them with an investment of Rs.16,000. Find the share of Rechal out of total
annual profit of Rs.10,000.
1. Rs.5000
2. Rs.3000
3. Rs.2000
4. Rs.4000

6.

A started a business with Rs.2400 and he is joined afterwards by B with Rs.3600. After how many months did B
joined if the profits at the end of the year are divided equally?
1. 8 months
2. 4 months
3. 1 month
4. None

7.

A started a business with Rs.16,000 and is joined afterwards by B with Rs.12,000. After how many months did B
joined, if As annual profit is double that of B.
1. 4 months
2. 8 months
3. 1 month
4. None

.B

w
w

A, B, C subscribed Rs.46,000. A subscribes Rs.12,000 more than B subscribes and B Rs.2000 more than C. Find the
share of A out of a total profit of Rs.23,000.
1. Rs.6000
2. Rs.5000
3. Rs.12000
4. Rs.24000

8.

an
k

Ex

am
sT

od
ay
.

1.

9.

A, B, C subscribed Rs.26,000. A subscribes Rs.4000 more than B and B Rs.2000 more than C. Find the share of B
out of a total profit of Rs.5200.
1. Rs.800
2. Rs.1200
3. Rs.1600
4. Rs.1000

10.

A, B, C subscribed Rs.30,000. A subscribes Rs.4000 more than B and B Rs.2000 less than C. Find the share of A out
of a total profit of Rs.6000.
1. Rs.2400
2. Rs.3000
3. Rs.2000
4. Rs.400

11.

A, B, C subscribed Rs.38,000. A subscribes Rs.2000 less than B and B Rs.4000 less than C. Find the share of C out
of a total profit of Rs.950.
1. Rs.450
2. Rs.400
3. Rs.200
4. Rs.275

Get all banking study material on BankExamsToday.com

Page 155

www.BankExamsToday.com, best study material for bank exams


CRT
A and B invests in a business in the ratio 4 : 5. If 10% of the total profit goes to a charity. If A share is Rs.4000 then
find the total profit.
1. Rs.10000
2. Rs.9000
3. Rs.12000
4. Rs.14000

13.

A is a working and B is a sleeping partner in a business. A puts in Rs.8000 and B Rs.7000. A receives 10% of the
profits for running the business the rest being divided in proportion to their investments. Find the total amount
received by A out of a total profit of Rs.500.
1. Rs.240
2. Rs.450
3. Rs.290
4. Rs.400

14.

A is a working and B is a Sleeping partner in a business. A puts in Rs.5000 and B Rs.4000. A receives 2/11 of the
profit for managing and the rest being divided in proportion to their capital. If A receives a total profit of Rs.2800.
Find the total profit.
1. Rs.4400
2. Rs.4000
3. Rs.3300
4. Rs.5500

15.

A, B and C contract a work for Rs.810 together. A and B are to do 5/9 of the work. Find the share of C.
1. Rs.400
2. Rs.360
3. Rs.200
4. Rs.519

16.

A and B started a business. A invested thrice as much as B and the period of his investment is two times the period of
investment of B. If B got Rs.5000 as profit, then find the total profit in the business.
1. 35000
2. 38000
3. 22000
4. None

17.

Padmanabhan and Sharavana started a joint firm. Padmanabhans investment was 2/3 of Sharavana and his
investment period is that of Sharavanas. If the difference between their profits is Rs.4000 then find the total profit.
1. Rs.54000
2. Rs.30000
3. Rs.20000
4. Rs.12000

18.

A, B and C enter into a partnership with shares in the ratio

am
sT

od
ay
.

co
m

12.

5 3 6
: :
after 4 months A increases his share by 50%.
2 4 5

4. Rs.1540

A and B enter into a partnership with the capitals in the ratio of 6 : 7. After 3 months A withdraws
and B reinvested
1. Rs.1050; 1550

1
of his capital
6

1
of his capital more. If the total profit at the end of a year is Rs.2600. Find the share of each.
7

an
k

19.

Ex

If the total profit at the end of the year Rs.3170 then find the profit share of B.
1. Rs.800
2. Rs.450
3. Rs.1700

2. Rs.1800;800

3. Rs.1300; 1300

4. None

A and B enter into a partnership with the capitals in the ratio of 4 :5. After 5 months A invested 25% of his capital
more and B withdraws 40% of his capital. The total gain at the end of 10 months was Rs.850. Find the share of A.
1. Rs.150
2. Rs.300
3. Rs.200
4. Rs.450

21.

A and B started a business with Rs.2000 and Rs.3000 respectively. If the total profit was Rs.1500 find the share of A.
1. Rs.500
2. Rs.600
3. Rs.900
4. Rs.1000

w
w

A and B started a business with Rs.1500 and Rs.2500 respectively. 20% of the total profit was given to A as salary.
At the end of the year, if the total profit was Rs.4000, find the total amount received by A.
1. Rs.2000
2. Rs.1200
3. Rs.800
4. Rs.3200

22.

.B

20.

23.

A and B started a business with Rs.2000 and Rs.3000 respectively. 10% of the total profit was given to A as salary.
At the end of the year, if B got Rs.1080 as his profit, then find the total profit.
1. Rs.5000
2. Rs.3000
3. Rs.2000
4. Rs.4000

24.

A and B started a business with Rs.3000 and Rs.5000 respectively. 20% of the total profit was given to A as salary.
At the end of the year, if A got a profit of Rs.1760, then find the total profit.
1. Rs.2000
2. Rs.3520
3. Rs.1800
4. Rs.3600

Get all banking study material on BankExamsToday.com

Page 156

www.BankExamsToday.com, best study material for bank exams


CRT
A started a business with Rs.6000. After 4 months B joined with Rs.8000. At the end of the year, if the total profit
was Rs.5100, find the share of A.
1. Rs.2700
2. Rs.2500
3. Rs.2000
4. Rs.2400

26.

A started a business with Rs.4000. After 4 months B joined with Rs.5000. Again after 2 months, C joined with
Rs.6000. At the end of the year if C got Rs.1557 as his share, find the share of B.
1. Rs.2076
2. Rs.1730
3. Rs.2000
4. Rs.5363

27.

A and B started a business with Rs.6000 and Rs.8000 respectively. After 4 months A invested Rs.1000 more and B
withdraws Rs.1000. At the end of the year if B got Rs.3300 as his share, find the share of A.
1. Rs.3000
2. Rs.6600
3. Rs.2700
4. None

28.

P started a business with Rs.7000. After a few months Q joined with Rs.10,000. At the end of the year, if Q got
Rs.2500 as his share out of a total profit of Rs.5500, after how many months Q joined?
1. 7 months
2. 5 months
3. 10 months
4. 2 months

29.

R and S started a business with Rs.7000 and Rs.8000 respectively. If the total profit at the end of the year was
Rs.4500 then find the Rs share.
1. Rs.2500
2. Rs.2250
3. Rs.1500
4. Rs.2100

30.

M and N started a business with Rs.4500 and Rs.5500 respectively. At the end of the year, if N got Rs.4400 as profit
then find Ms share of profit.
1. Rs.2500
2. Rs.3600
3. Rs.4500
4. Rs.8000

31.

Ram and Dolly started a business with Rs.6300 and Rs.7200. 10% of the profit was given to Ram as salary. At the
end of the year if Dolly got Rs.2592 as profit then find the total profit.
1. Rs.3240
2. Rs.2268
3. Rs.5000
4. Rs.5400

32.

Rambabu started a business with Rs.12,000, 4 months later Nagarjuna joined with a capital of Rs.15,000. At the end
of the year total profit was Rs.5500. Find Rambabus share.
1. Rs.3000
2. Rs.2500
3. Rs.4000
4. Rs.4800

33.

Srihari and Pratap started a business with Rs.3000 and Rs.4000 respectively. 6 months later, Sudheer joined with a
capital of Rs.5000. At the end of the year, if Srihari got a profit of Rs.3600 then find the total profit.
1. Rs.3200
2. Rs.3000
3. Rs.11400
4. Rs.9000

34.

A, B and C enter into partnership with investments in ratio 3 : 5 : 7 respectively. The period of investments is 10
months, 4 months and 5 months respectively. Find As share of profit out of the total of Rs.4250.
1. Rs.1500
2. Rs.850
3. Rs.1625
4. None

od
ay
.

am
sT

Ex

an
k

.B

w
w

A started the business with Rs.1.5 lakhs. After one year he allowed 10 people to join the business as silent partners
with each investing Rs.18,000. A claimed 1/5th of the profit as his salary. After another one year, if the total profit
earned is Rs.1 lakh, find As total share in the profit.
1. Rs.30,000
2. Rs.50,000
3. Rs.40,000
4. Rs.70,000

35.

co
m

25.

36.

A invested Rs.1 less than B for every Rs.8 invested by B. If A gets Rs.150 less than B, how much is the total profit
earned by them?
1. Rs.1200
2. Rs.1050
3. Rs.2250
4. None

37.

As investment is 20% more than Bs investment. If the total profit is Rs.1155, how much is Bs share of profit less
than that of A?
1. Rs.105
2. Rs.924
3. Rs.231
4. None

Get all banking study material on BankExamsToday.com

Page 157

www.BankExamsToday.com, best study material for bank exams


CRT
A, B and C rented a pasture. A puts 90 sheeps for 20 days. B puts 75 sheeps for 18 days and C puts 60 sheeps for 30
days. If the total rent is Rs.2640, Bs share of rent is?
1. Rs.264
2. Rs.720
3. Rs.810
4. None

39.

A and B rent a pasture. A puts 30 horses for 60 days while B puts some number of horses for 50 days. If the rent paid
by them are in ratio 9 : 10, how many horses did B put in pasture?
1. 60
2. 40
3. 20
4. 10

40.

Two partners invest Rs.3500 and Rs.4500 respectively in business. They agree that 50% of the profit would be
distributed equally between them and remaining profit in ratio of capital. If the difference between their shares of
profit is Rs.100, then find the total profit.
1. Rs.800
2. Rs.1600
3. Rs.2400
4. None

Answers
21) 2
22) 1
23) 3
24) 2
25) 1
26) 2
27) 1
28) 2
29) 4
30) 2

am
sT

11) 2
12) 1
13) 3
14) 1
15) 2
16) 1
17) 4
18) 2
19) 1
20) 4

w
w

.B

an
k

Ex

1) 1
2) 3
3) 2
4) 2
5) 4
6) 2
7) 2
8) 3
9) 3
10) 1

od
ay
.

co
m

38.

Get all banking study material on BankExamsToday.com

Page 158

31) 4
32) 1
33) 3
34) 1
35) 4
36) 3
37) 1
38) 2
39) 2
40) 2

www.BankExamsToday.com, best study material for bank exams


CRT
MIXTURES (OR) ALLIGATION
Alligation: It means linking.
a) It is a rule to find the ratio in which two or more ingredients at their respective prices should be mixed
to give a mixture at a given price.

Mean Price: The cost price of a unit quantity of mixture is called the mean price.

1.

Let us suppose Rs.a per unit be the price of the first ingredient (superior quality) is mixed with another
ingredient (cheaper quality) of price Rs.b per unit to form a mixture whose mean price is Rs.m per unit,
then the two ingredients must be mixed in the ratio:
C.P.of superior- Meanprice
Meanprice - C.P.ofcheaper

a m
m b

am
sT

Quantityof cheaper
Quantityof superior

2.

od
ay
.

Alligation Rule:

co
m

b) It is a rule to find the average price of a mixture when the prices of two or more ingredients which may
be mixed together and the proportion in which they are mixed are given.

This means, the two ingredients are to be mixed in the inverse ratio of the di fferences of their prices and
the mean price.

w
w

.B

an
k

Ex

Note: This can be remembered easily through the diagram below:

Formulae:

A vessel contains a litres of wine. From it b litres are withdrawn. The vessel is then filled with water. Next
b litres of the mixture are withdrawn and again the vessel is filled with water. If this process is repeated n
times then

1.

Wine left in the vesselafternth operation


Originalquantityof wine in the vessel

After n th the quantity of wine left in the vessel = a 1


2.

b
a

There are n vessels of equal size filled with mixtures of liquids A and B in the ratio a 1 : b 1 , a 2 :
b 2 ,,a n : b n , respectively. If the contents of all the vessels are poured into a single large vessel, then

Get all banking study material on BankExamsToday.com

Page 159

www.BankExamsToday.com, best study material for bank exams


CRT
a1
Q uantityof liquid A
Q uantityof liquidB

b1

a2

b2

a1

b1
b1

a2

b2
b2

................
................

an
an

bn

an

bn
bn

There are n vessels of sizes c 1 , c 2 , .. , c n filled with mixtures of liquids A and B in the ratio a 1 : b 1 , a 2 :
b 2 ,,a n : b n , respectively. If the contents of all the vessels are poured into a single large vessel, then
Q uantity of liquid A
Q uantity of liquid B

a1c1
a1 b1

a2c2
a2 b2

................

ancn
an bn

b1c1
a1 b1

b2c2
a2 b2

................

bncn
an bn

In what ratio the two varieties of coffee one costing Rs.30 per kg and the other Rs.35 per kg should be
blended to produce a blended variety of coffee worth Rs.32 per kg. How much should be the quantity of
second variety of coffee, if the first variety is 72 kg.
Sol:

an
k

The required ratio


is 3 : 2. i.e.

Ex

am
sT

1.

od
ay
.

SOLVED EXAMPLES

co
m

3.

a2

a1

Quantityof cheaperquality
Quantityof superiorquality

2
3

48 kg.

Salt at Rs.10 per kg is mixed with salt at Rs.15 per kg in the ratio 3 : 4. Find the price per kg of mixture.

w
w

2.

3
2

.B

So, Quantity of superior coffee = 72

of the two varieties of coffee

Sol: Let the mean price of the mixture be Rs.x.

Get all banking study material on BankExamsToday.com

Page 160

www.BankExamsToday.com, best study material for bank exams


CRT
Quantityof cheapersalt
Quantityof superiorsalt
3
4

3x

30

60 4x

90
7

6
Rs.12 .
7

100

4
5

100

co
m

20
100

40.96litres.

Two equal glasses are filled with mixture of milk and water. The proportion of milk and water in the
first glass is 4 : 3 and in the second glass is 5 : 2. The contents of the two glasses are emptied into a
single vessel. What is the proportion of milk and water in it?
Quantityof milk
Sol:
Quantityof water

4
4

5
3

9
5

9 : 5.

Three glasses of sizes 1 litres, 2 litres and 3 litres contain mixture of milk and water i n the ratio 2: 3, 3 :
7 and 4 : 11 respectively. The contents of all the three glasses are poured into a single vessel. Find the
ratio of milk to water in the resulting mixture.
2
2
3
2

1
3
1
3

3
3
7
3

2
7
2
7

4
4
11
4

3
11
3
11

Ex

Q uantityof m ilk
Sol:
Q uantityof water

am
sT

5.

90

A vessel contains 100 litres of wine. 20 litres of wine was taken out and replaced by water. Then, 20
litres of mixture was withdrawn and again replaced by water. The operation was repeated for fourth
time. How much wine is now left in the vessel?
Sol: Amount of wine left in the vessel = 1

4.

7x

od
ay
.

3.

15 x
x 10

15 x
x 10

2
5
3
5

6
10
14
10

12
15
33
15

12 18 24
30
18 42 66
30

54
126

Exercise

an
k

= 54 : 126 or 27 : 63.

How many kilograms of rice costing Rs.12/- per Kg should be mixed with another rice of 20 Kg at Rs.16/- per Kg to
obtain the mixture costing Rs.13/- per Kg?
1. 50 Kg
2. 60 Kg
3. 45 Kg
4. 40 Kg

2.

In what proportion must water be mixed with spirit to gain 16 2/3% by selling at its cost price?
1. 6:1
2. 5:2
3. 1:6
4. 2:3

w
w

How many kilograms of rice costing Rs.12/- per Kg should be mixed with another rice of 20 Kg at Rs.16/- per Kg to
obtain the mixture costing Rs.13/- per Kg?
1. 50 Kg
2. 60 Kg
3. 45 Kg
4. 40 Kg

3.

.B

1.

4.

Two equal glasses filled with mixtures of alcohol and water in the proportions of 2:1 and 1:1 respectively were
emptied into a third glass. What is proportion of alcohol and water in the third glass?
1. 5:7
2. 3:5
3. 5:3
4. 7:5

5.

A vessel of 90 litres is filled with milk and water. 55% of milk and 40% of water is taken out of the vessel. It is
found that the vessel is vacated by 45%. Find the initial quantity of water?
1. 30 litres
2. 35 litres
3. 40 litres
4. 60 litres

Get all banking study material on BankExamsToday.com

Page 161

www.BankExamsToday.com, best study material for bank exams


CRT
6.

9 litres are drawn from a cask full of wine, and it is then filled with water. Nine litres of the mixture are drawn and
the cask is again filled with water. The ratio of the wine now left to that of the water is 16:9. How much does the
cask hold?
1. 40 litres
2. 45 litres
3. 50 litres
4. 55 litres

7.

How much water must be added to 60 litres of milk at 1


litre?
1. 15 litres

3. 5 litres

4. 12 litres

co
m

2. 10 litres

1
2
litres for Rs.20 so as to have a mixture worth Rs.10 a
2
3

In what proportion must a grocer mix tea at Rs.1.02 per Kg and Rs.1.44 per Kg so as to make a mixture worth
Rs.1.26 per Kg?
1. 4:3
2. 2:3
3. 3:2
4. 3:4

9.

A vessel of 80 litre is filled with milk and water. 70% of milk and 30% of water is taken out of the vessel. It is found
that the vessel is vacated by 55%. Find the initial quantity of milk?
1. 30 Kg
2. 25 Kg
3. 35 Kg
4. 50 Kg

10.

300 grams of sugar solution has 50% sugar in it. How much sugar should be added to make it 40% in the solution?
1. 25 gm
2. 75 gm
3. 50 gm
4. 35 gm

11.

A trader has 50 Kg of rice, a part of which he sells at 14% profit and the rest at 6% loss. On the whole his loss is 4%.
What is the quantity sold at 14% profit?
1. 45 Kg
2. 25 Kg
3. 20 Kg
4. 5 Kg

12.

Vessel A contains milk and water in the ratio 7:3 and vessel B contains the same in the ratio 3:1. In what proportion
should quantities be taken from A and B to form a mixture in which milk and water are in the proportion 11:4?
1. 2:1
2. 3:2
3. 1:2
4. 2:3

13.

In what proportion must a grocer mix tea at Rs.6 per Kg and Rs.8 per Kg so as to make a mixture worth Rs.7 per Kg?
1. 1:1
2. 4:3
3. 2:3
4. 3:2

14.

In a Zoo, there are Monkeys and Parrots, If heads are counted, there are 450 and if legs are counted there are 1000.
How many Monkeys are there?
1. 150
2. 50
3. 200
4. 250

15.

How many kilograms of rice costing Rs.8/- per Kg should be mixed with another rice of 60 Kg at Rs.15/- per Kg to
obtain the mixture costing Rs.11/- per Kg?
1. 40 Kg
2. 20 Kg
3. 80 Kg
4. 60 Kg

16.

A man lend Rs.1500/- in 2 parts one at 5% and another at 8%. If at the end of the year he received Rs.93/- as interest.
Then the sum lent at 8%?
1. 900
2. 700
3. 600
4. 500

am
sT

Ex

an
k

.B

w
w

400 grams of sugar solution has 30% sugar in it. How much sugar must be added to make it 60% in the solution?
1. 80
2. 120
3. 300
4. 180

17.

od
ay
.

8.

18.

A man buys two buildings for Rs.45000/- and sells one so as to lose 12% and the other so as to gain 15% and on the
whole be neither gains no loses. What is the cost of building sold at 12% lose?
1. Rs.20000/2. Rs.22000/3. Rs.25000/4. Rs.23000/-

19.

Three vessels of equal capacity contain alcohol and water in the ratio of 3:2, 5:4 and 1:4 respectively. It all the
solutions are mixed, what is the ratio of alcohol to water in the resultant mixture?
1. 61:74
2. 5145
3. 50:73
4. 74:45

Get all banking study material on BankExamsToday.com

Page 162

www.BankExamsToday.com, best study material for bank exams


CRT
Two litres of pure milk is added to 6 litres of a milk solution containing 16 2/3% milk. What is the concentration of
milk in the resultant solution?
1. 12:5%
2. 25%
3. 33 1/3%
4. 37.5%

21.

In what ratio must 82 % of wine solution be mixed with pure wine to get a resultant solution of 6% of water?
1. 3:1
2. 2:1
3. 1:3
4. 1:2

22.

In what proportion must water be mixed with the spirit to gain 26 2/3% by selling it at cost price?
1. 4:15
2. 15:4
3. 1:12
4. 15:7

23.

Three equal glasses are filled with a mixture of spirit water. The proportion of spirit to water in the first glass is 2:3,
in the second glass is 3:4 and the third glass is 4:5. The contents of the three glasses are emptied into a single vessel.
What is the proportion of spirit and water in it?
1. 544:401
2. 401:544
3. 410:544
4. 401:454

24.

A grocer mixes 2 varieties of tea, one costing Rs.75/- per Kg and another Rs.100/- per Kg. In what proportion he
should mix these so by selling at Rs.88/- per Kg 10% is gained?
1. 1:4
2. 4:1
3. 3:4
4. 4:3

25.

What weight of rice worth Rs.4.20 per Kg should be mixed with 60 Kg of rice worth Rs.2.70 per Kg so that when the
mixture is sold at Rs.3.30 per Kg. There may be neither gain nor loss?
1. 50Kgs
2. 45Kgs
3. 55Kgs
4. 40Kgs

26.

A mixture of milk and water contains 10% water. How much water must be added to make 20% water in the new
mixture?
1. 5Ltrs
2. 6Ltrs
3. 8Ltrs
4. 10Ltrs

27.

In 70 litres of a mixture of milk and water, the quantity of water is 10%. How much water should be added so that
new mixture may contain 25%?
1. 7
2. 14
3. 21
4. 9

28.

A sum of Rs.39/- was divided among 45 boys and girls. Each girl gets 50paise, whereas a boy gets one rupee. Find
the number of girls?
1. 33
2. 32
3. 22
4. 12
A sum of Rs.34.000 was invested partly at 5% p.a. and the remaining at 3% p.a. simple interest for a period of 2
years. The total interest after the end of two years is Rs.2. 680. What was the amount interest at 5%?
1. 19,000
2. 14,000
3. 13,000
4. 16,000

od
ay
.

am
sT

Ex

an
k

Goods worth Rs.400 are purchased. One fourth of them are sold at 5% loss. At what profit percent should the
remaining be sold such that 10% profit is made on the whole?
1. 5%
2. 15%
3. 25%
4. 20%
A container has 40 litres of whisky. From this 4 litres of whisky is replaced by water. This process was replaced two
times. How much whisky is now contained in t he container?
1. 26.34 litres
2. 27.36 litres
3. 28 litres
4. 32.4 litres

31.

.B

30.

w
w

29.

co
m

20.

32.

Two qualities of tea are mixed in the ratio of 4:1 and the mixture is sold at Rs.72 per Kg for a profit of 12.5%. If the
tea of the second quality costs Rs.3.25 more per Kg than the tea of the first quality. What is the cost per Kg of the tea
of first quality?
1. 65.53
2. 63.35
3. 65.35
4. 63.53

33.

In what ratio must a solution of milk and water containing 42% milk be mixed with pure milk to get a resultant
solution containing 29% water?
1. 1:2
2. 2:1
3. 3:2
4. 1:1

Get all banking study material on BankExamsToday.com

Page 163

www.BankExamsToday.com, best study material for bank exams


CRT
How many litres of pre water must be mixed with 144 litres of pure spirit in order to gain 11% by selling the
resultant mixture at cost price of pure spirit?
1. 32
2. 48
3. 18
4. 16

35.

A rice trader mixes two verities of rice. The cost price of the first variety rice is twice the cost price of the second
variety. If the trader sells the mixture at the cost price of the other variety and makes a profit of 35% then what is the
ratio in which the verities of the rice are mixed?
1. 7:13
2. 13:14
3. 1:3
4. cannot be determined

36.

In what ratio a grocer mix two varieties of tea worth Rs.60 per kg and Rs.65 Per Kg so that by selling the mixture at
Rs.68.20 per Kg he may gain 10%
1. 3:2
2. 3:4
3. 3:5
4. 4:5

37.

A barrel has 50 litres of pure spirit. 5 litres of spirit is removed and replaced with the same amount of water. This
process is repeated two more times, what is the amount of spirit in the resultant mixture?
1. 36.45
2. 34.45
3. 36.54
4. 35.45

38.

A grocer purchased 20 Kg of sugar at the rate of Rs.15/- per Kg and 30 Kg of sugar at the rate of Rs.13/-. At what
price per Kg should be sell the mixture to earn 33 1/3% profit on the cost price?
1. Rs.28.00
2. Rs.20.00
3. Rs.18.40
4. Rs.17.40

39.

How many kilograms of wheat at Rs.9.30 per Kg must be mixed with 16 Kg of wheat at Rs.13.80 per Kg such that
the mixture when sold at Rs.12.43 per Kg gives a profit of 10%?
1. 20
2. 33
3. 30
4. 25

40.

A trader mixes two types of rice, one costing Rs.18/- per Kg and the other costing Rs.15/- per Kg. so that the mixture
is worth Rs.16.25 per Kg. In what ratio does he mix the two types of rice?
1. 5:3
2. 5:7
3. 3:2
4. 8:5

Ex

am
sT

od
ay
.

co
m

34.

11) 4
12) 3
13) 1
14) 2
15) 3
16) 3
17) 3
18) 3
19) 3
20) 4

.B

21) 4
22) 1
23) 2
24) 2
25) 4
26) 1
27) 2
28) 4
29) 4
30) 2

w
w

1) 2
2) 3
3) 2
4) 4
5) 4
6) 2
7) 1
8) 4
9) 4
10) 2

an
k

Answers

Get all banking study material on BankExamsToday.com

Page 164

31) 4
32) 2
33) 4
34) 4
35) 2
36) 1
37) 1
38) 3
39) 1
40) 2

www.BankExamsToday.com, best study material for bank exams


CRT
PROBLEMS ON AGES
In solving the problems related to ages, we come across three situations.
1. Age some years ago
2. Present Age
3. Age some years hence

co
m

Models:

If the age of A, t years ago, was n 1 times the age of B and at present the age of A is n 2 times that of B, then
finding the present Age of A and the present Age of B.

2.

If the present age of A is n 1 times the present age of B. If t years hence, the age of A would be n 2 times that of
B, then finding the present Age of A the present Age of B.

3.

If the age of A, t 1 years ago, was n 1 times the age of B and if t 2 years hence As age would be n 2 times that of B,
then finding the present Age of A and the present Age of B.

4.

The sum of present ages of A and B is S years. If t years ago, the age of A was n times the age of B, then
finding the present Age of A and the present Age of B.

5.

The sum of present ages of A and B is S years. If t years hence, the age of A would be n times the age of B, then
finding the present Age of A and the present Age of B.

6.

If the ratio of the present ages of A and B is a : b and t years hence, it will be c : d, then finding the present Age
of A and the present Age of B.

The age of Mr. Bindra is 3 times the age of his son. If 10 years ago, his age was 8 times the age of his son,
then at that time what would be the age of Mr. Bindra.

an
k

1.

Ex

SOLVED EXAMPLES

am
sT

od
ay
.

1.

present:
son = x+10
Mr. Bindra = 8x+10
3(x+10) = 8x+10
3x+30 = 8x+10
x=4
Mr. Bindra = 8x4+10 = 42

The age of Mr. Sinha is 7 times the age of his daughter. After 15 years, the age of Mr. Sinha is only twice the
age of his daughter. Find the present age of Mr. Sinhas daughter.

2.

w
w

.B

Sol: 10 years ago:


Son = x
Mr. Bindra = 8x

Sol: Present:
Daughter = x
Mr. Sinha = 7x

After 15 years:
Daughter = x+15
Mr. Sinha = 7x+15
7x+15 = 2(x+15)
7x+15 = 2x+30
x=3
Mr. Sinhas daughter = 3 years

Get all banking study material on BankExamsToday.com

Page 165

www.BankExamsToday.com, best study material for bank exams


CRT
12 years ago Lekhas father was 5 times older than her. After 12 years, the father will be 3 times older than
her. Find the present age of Lekha.

Sol: 12 years ago:


Lekha = x
Father = 5x

Sol: 2 years back:


B=x
A = 4x

The sum of the ages of father and a son is 44 years. After 8 years, the age of the father will be 5 times that of
son. Find their respective ages.
Present:
Son = x 8
Father = 5x-8
x-8+5x-8 = 44
x = 10
Son = 10-8 = 2
Father = 50-8 = 42

Ex

Sol: After 8 years:


Son = x
Father = 5x

6.

x+2+4x+2 = 36
x = 6.4
A-B = 25.6 6.4 = 19.2

am
sT

5.

Present:
B = x+2
A = 4x+2

co
m

3(x+24) = 5x+24
3x+72 = 5x+24
x = 24
Lekha = 24+12 = 36
The sum of the ages of A and B is 36 years. 2 years back, the age of A is 4 times the age of B. Find the
difference between the present ages of A and B.

The ratio of the present ages of father and son is 3 : 1. After 10 years, it will become 5 : 2. Find the present
age of the son.

Sol: Present:
Son = x
Father = 3x

an
k

4.

Present:
After 12 years:
Lekha = x+12
Lekha = x+24
Father = 5x+12
Father = 5x+24

od
ay
.

3.

After 10 years:
Son = x+10
Father = 3x+10

.B

3x 10
x 10

5
2

4 years ago Kalyani was thrice as old as Ramya. If the ratio of their present ages is 4 : 3 respectively. Find
the difference between their ages.

7.

w
w

6x+20 = 5x+50
x = 30
son = 30 years

Sol:

Present:
Kalyani = 4x
Ramya = 3x

4 years ago:
Kalyani = 4x-4
Ramya = 3x-4

3(3x-4) = 4x-4
9x-12 = 4x-4
x = 1.6
Kalyani Ramya = 1.6

Get all banking study material on BankExamsToday.com

Page 166

www.BankExamsToday.com, best study material for bank exams


CRT
Exercise
The ratio of Ps and Qs ages is 5:7. If the difference between the present age of Q and the age of P six years hence is
2, then what is the total of present ages of P and Q?
1. 52 yrs
2. 48 yrs
3. 56 yrs
4. None

2.

The ratio between the present ages of P and Q is 5:8. After 4 years, the ratio between their ages will be 2:3. What is
Qs present age?
1. 36 yrs
2. 20 yrs
3. 24 yrs
4. None

3.

The ratio of present ages of Ram and Shyam is 7:8, respectively. Four years hence this ratio becomes 9:10,
respectively. What is Rams present age in years?
1. 18
2. 14
2. 17
4. None

4.

One year ago the ratio of Karans and Ankits ages was 6:7, respectively. Four years hence this ratio would become
7:8. How old is Ankit?
1. 35 yrs
2. 30 yrs
3. 31 yrs
4. None

5.

The sum of the ages of A & B is 50 years and the ratio between their ages is 7:3 then the age of A is how many
years?
1. 15
2. 35
3. 20
4. 25

6.

The sum of the ages of P & Q is 72 years and the ratio between their ages is 2:7. Then the ratio between their ages 8
years ago is __________
1. 2:5
2. 5:2
3. 10:15
4. 1:6

7.

The ratio between the present ages of A & B in 4:5 and the sum of their ages after 5 years is 100 years. What will be
the ratio of their ages at that time?
1. 9:10
2. 10:9
3. 9:11
4. 11:9

8.

The ratio between the ages of a father and his son, 10 years ago was 9:2. The ratio between the present ages of the
father and his son is 2:1. What is the ratio of the ages of Father & son 10 years hence?
1. 2:1
2. 19:12
3. 12:7
4. 21:11

9.

The sum of the ages of a mother and her daughter is 50 years. Also, 5 years ago, the mothers age was 7 times the age
of the daughter. The present ages of the mother and daughter respectively are ________
1. 35 & 15 yrs
2. 38 & 12 yrs
3. 40 & 10 yrs
4. 42 & 8 yrs

10.

A man is 4 times as old as his son. Four years hence the sum of their ages will be 43 years. The present age of son is
_________
1. 5 yrs
2. 7 yrs
3. 8 yrs
4. 10 yrs

od
ay
.

am
sT

Ex

an
k

.B

w
w

Kalyani got married 6 years ago. Today her age is 1

11.

co
m

1.

times her age. Her sons age is _________


1. 2 yrs
2. 3 yrs

1
1
times her age at the time of marriage. Her sons age is
4
10

3. 4 yrs

4. 5 yrs

12.

The difference between the ages of two persons is 10 years, 15 years ago, the elder one was twice as old as the
younger one. The present age of the elder son is ________
1. 25 yrs
2. 35 yrs
3. 45 yrs
4. 55 yrs

13.

The ratio between the ages of A and B at present is 2:3. Five years hence the ratio of their ages will be 3:4. What is
the present age of A?
1. 10 yrs
2. 15 yrs
3. 25 yrs
4. None

Get all banking study material on BankExamsToday.com

Page 167

www.BankExamsToday.com, best study material for bank exams


CRT
Mahesh is as much younger to Anil as he is older to Prashant. If the sum of the ages of Anil and Prashant is 48 years,
what is the age of Mahesh?
1. 20 yrs
2. 24 yrs
3. 30 yrs
4. None

15.

Five years ago, Vinays age was one third of the age of Vikas and now Vinays age is 17 years. What is the present
age of Vikas?
1. 5 yrs
2. 36 yrs
3. 41 yrs
4. 51 yrs

16.

The age of Arivinds father is 4 times his age. If 5 years ago, fathers age was 7 times of the age of his son at that
time. What is Arivinds fathers present age?
1. 35 yrs
2. 40 yrs
3. 45 yrs
4. 55 yrs

17.

10 years ago, Anitha mother was 4 times older than her daughter, after 10 years, the mother will be twice of the
daughter. The present age of Anitha is ________
1. 5 yrs
2. 20 yrs
3. 45 yrs
4. 30 yrs

18.

A is twice as old as B. 12 years ago, A was five times old as B. Find the present age of A?
1. 16 yrs
2. 32 yrs
3. 24 yrs
4. 28 yrs

19.

The age of the father 8 years ago was 5 times the age of his son. At present the fathers age is 3 times that of his son.
Find the age of father?
1. 48 yrs
2. 36 yrs
3. 46 yrs
4. 58 yrs

20.

At present the age of the father is five times that of the age of his son. Three years hence, the fathers age would be
four times that of his son. Find the present ages of the father and the son?
1. 8: 32
2. 45: 9
3. 12;57
4. None

21.

The age of Mrs. Anjali is 5 times the age of his son. After 12 years the age of Mrs. Anjali will be only twice the age
of his son. Find the present age of Mrs. Anjaliss son?
1. 4 yrs
2. 16 yrs
3. 12 yrs
4. 18 yrs

22.

9 years ago Vimal was 5 times the age of Sudarshan. If the present age of Vimal is twice the age of Sudarshan, what
will be the total of their present ages?
1. 66 yrs
2. 54 yrs
3. 36 yrs
4. 46 yrs

23.

At present the age of the father is 6 times the age of his son, 4 years hence the fathers age would be 5 times that of
his son. What is the sum of the present ages of father and his son?
1. 116 yrs
2. 112 yrs
3. 114 yrs
4. 111 yrs

24.

At present the age of the father is 7 times the age of the son, 4 years hence the fathers age would be 4 times that of
his son. What is the sum of the present ages of father and his son?
1. 21 yrs
2. 24 yrs
3. 28 yrs
4. 32 yrs

od
ay
.

am
sT

Ex

an
k

.B

w
w

At present the age of the father is 7 times the age of his son, 6 years hence the fathers age would be 5 times that of
his son. What is the sum of the present ages of father and his son?
1. 80 yrs
2. 64 yrs
3. 96 yrs
4. None of these

25.

co
m

14.

26.

The sum of ages of A and B is 60 years. After 10 years A will be thrice as old as B. Find the difference of their
present ages?
1. 30 yrs
2. 40 yrs
3. 10 yrs
4. 50 yrs

27.

The sum of the ages of the father and his son is 41 years. After 17 years the fathers age will be twice the age of his
son. Then the respective ages of the father and son are ______ and ____ years.
1. 32, 9
2. 34, 7
3. 33, 8
4. 31, 10

Get all banking study material on BankExamsToday.com

Page 168

www.BankExamsToday.com, best study material for bank exams


CRT
The sum of the ages of a son and father is 56 years. After four years, the age of the father will be three times that of
the son. Their ages respectively are _____________
1. 12 & 44 yrs
2. 16 & 48 yrs
3. 16 & 42 yrs
4. 18 & 36 yrs

29.

The present ages of the father and son are in the ratio 6:1. After 5 years, the ratio will be 13:3. Find the present age of
the son?
1. 24 yrs
2. 28 yrs
3. 32 yrs
4. None

30.

The ratio of the present ages of P and Q is 8:5. After 6 years their ages are in the ratio of 3:2. Find the ratio of the
sum and difference of the present ages of P and Q.
1. 13:3
2. 39:19
3. 13:2
4. 13:5

31.

8 years ago, the ratio of the ages of Nagendra and Sunil was 3:2. If the ratio of their present ages is 7:5 respectively,
what is the sum of their present ages?
1. 96 yrs
2. 86 yrs
3. 76 yrs
4. 66 yrs

32.

The ratio of the ages of A and B at present is 4:2. 10 years earlier, the ratio was 3:2, then find the present ages of A
and B.
1. 40 & 30 yrs
2. 48 & 36 yrs
3. 64 & 48 yrs
4. 20 & 15 yrs

33.

The ages of Kanchan is thrice the age of the Chandan. After 12 years the age of Kanchan will become twice the age
of Chandan. Then the sum of their present ages is ____ years.
1. 42 yrs
2. 48 yrs
3. 46 yrs
4. 50 yrs

34.

The age of Jayshree is thrice the age of her younger sister. The product of their ages is 300 years. Then the Jayshrees
present age is ___________ years.
1. 30
2. 20
3. 10
4. 40

35.

At the time of the marriage, a man was 6 years elder than to his wife. 12 years after their marriage, his age is 6/5
times the age of his wife. What was the wifes age at the time of marriage?
1. 18 years
2. 24 years
3. 30 years
4. 36 years

36.

The difference of present ages of A and B is 12 years. 6 years back their ages were in the ratio 3:2, how old is B?
1. 32 years
2. 35 years
3. 30 years
4. 45 years

37.

Siddhikas age is
th

an
k

Ex

am
sT

od
ay
.

co
m

28.

1
6

th

of her fathers age. Siddhikas fathers age will be twice of Kamals age after 10 years. If

Ten years before, one is seven years more than the half the age of other. Now the brides age is 30 years. And also
one is nineteenths the age of other. What is grooms present age.
1. 35
2. 27
3. 40
4. 58

39.

3 years ago, the fathers age was twice the ages of his 4 daughters. In 3 years time, the fathers age will be equal to
sum of ages of the daughters. Find the present age of father.
1. 45 years
2. 36 years
3. 24 years
4. 39 years

w
w

38.

.B

Kamals 8 birthday was celebrated two years before, then what is Siddhikas present age?
1. 10 years
2. 15 years
3. 5 years
4. 3 years

40.

When I add 4 times my age 4 years from now to 5 times my age 5 years from now, I get 10 times my current age.
How old will I be 3 years from now?
1. 52
2. 40
3. 44
4. None

Get all banking study material on BankExamsToday.com

Page 169

www.BankExamsToday.com, best study material for bank exams


CRT
Answers
21) 1
22) 3
23) 2
24) 4
25) 3
26) 2
27) 3
28) 1
29) 4
30) 1

31) 1
32) 1
33) 2
34) 1
35) 1
36) 3
37) 3
38) 4
39) 2
40) 3

co
m

11) 2
12) 2
13) 1
14) 2
15) 3
16) 2
17) 2
18) 2
19) 1
20) 2

w
w

.B

an
k

Ex

am
sT

od
ay
.

1) 2
2) 4
3) 2
4) 4
5) 2
6) 4
7) 3
8) 2
9) 3
10) 2

Get all banking study material on BankExamsToday.com

Page 170

www.BankExamsToday.com, best study material for bank exams


CRT
PROBLEMS ON TRAINS
A train is said to have crossed an object (stationary or moving) only when the last coach (end) of the train
crosses the said object completely.
Hence, the distance covered by the train = length of train + length of object
train Length of object
Speed of train Speed of object

co
m

Time to cross an object moving in the direction of train = Length of

DIFFERENT TYPES OF OBJECTS


On the basis of various types of objects that a train has to cross, we find the following different cases:

2.
3.
4.

od
ay
.

1.

Time to cross +
length of train
t=
speedof train

Type of Object
Object is stationary and is of negligible length,
e.g., train crosses lamp post, pole, standing man, etc.
Object is stationary and is of some length,
e.g., train crosses bridge, a tunnel, platform, or
another train at rest.
Object is moving* and is of negligible length,
e.g., train crosses a running man, a running car, etc.
Object is moving* and has some length,
e.g., train crosses another running train.

t=

length of (train object)


speed of train

t=

length of train
speedof (train object)

t=

length of (train object)


speed of (train object)

am
sT

Case

(*if the object is moving in opposite direction, then denominator becomes speed of (train + object).

Ex

TWO TRAINS CROSSING EACH OTHER IN BOTH DIRECTIONS

Speed of Faster train =


Speed of Slower Train =

L1

L2
2

L1

L2
2

1
t1
1
t1

1
t2
1
t2

w
w

.B

an
k

Two trains are crossing each other


Length of one train = L 1
Length of second train = L 2
They are crossing each other in opposite direction in t 1 seconds.
They are crossing each other in same direction in t 2 seconds then,

SOLVED EXAMPLES
1)

A train travelling with uniform speed crosses a platform of length 800 metres in 20 seconds and a tunnel of length 1.1
kilometer in 25 seconds. What is the length of the train?
Sol: Let the length of the train be x metres.
Speed while crossing the platform =

(x

800)
m/sec.
20

Get all banking study material on BankExamsToday.com

Page 171

www.BankExamsToday.com, best study material for bank exams


CRT
1100)
25
(x 800) (x 1100)
=
20
25

Speed while crossing the tunnel

Since the train has uniform speed

(x

5 (x + 800)
x
The length of the train is 400 metres.

If 460 poles are arranged such that the distance between any two successive poles is 20 metres, and a train 180 m
long crosses them completely in three minutes, what is the speed of the train?

Speed of the train =


3)

459 20

180

3 60

= 52 m/s

od
ay
.

Sol: The total distance covered = distance between 460 poles + length of the train

co
m

2)

= 4(x + 1100)
= 4400 400 = 400 metres.

The train crosses a man standing on a platform 150 metre long in 10 seconds and crosses the platform completely in
22 seconds. Find the length of train and speed of train.
Sol: Let Lt and Vt be the length and the speed of the train
Lt L
Vt V

am
sT

Using the basic formula t =

For standing man, put L = 0 and V = 0, then, we get


10 =

Lt
_____________________ (1)
Vt

For the stationary platform, put V = 0, and L = 150 (given), then we get
Lt

150
Vt

Ex

22 =

From (1) & (2),

Lt
Vt

an
k

22 =

150
___________________ (2)
Vt

150
Vt

22 = 10 +

Since

Lt
Vt

10in1

w
w

.B

Vt = 12.5 m/sec
Speed of train = 12.5 metre/sec.

From (1)

10 =

Lt
12.5

Lt = 12.5
length of train = 12.5 metres.

4)

A toy train crosses 210 and 122 metre long tunnels in 25 and 17 seconds respectively. Find the length of train and
speed of train.
Sol: Let Lt and Vt be the length of train and speed of train respectively.
Using the basic formula t =
For tunnel, put V = 0
25

Lt L
Vt V

L t 250
Vt

Get all banking study material on BankExamsToday.com

Page 172

www.BankExamsToday.com, best study material for bank exams


CRT
Lt + 210
And

= 25Vt

17

L t 122
________________ (1)
Vt

Lt + 122 = 17 Vt __________________ (2)

co
m

Subtracting (2) from (1)

od
ay
.

210 122 = 8 Vt
Vt
= 11 m/sec
Putting Vt = 11 in equation (1),
Lt + 210 = 25 x 11
Lt = 275 210 = 65 m
Speed of the train
= 11 m/sec.
Length of the train = 65 m
5)

A train 75 metres long overtook a man who was walking at the rate of 6 km/hr and crossed him in 18 seconds. Again,
the train overtook a second person in 15 seconds. At what rate was the second person travelling?

Using the basic formula t =

Lt L
Vt V

am
sT

Sol: Let Vt be the speed of the train and V2 be the speed of the second person

For the first man, put L = 0, V = 6 km/hr = 6

5
3

an
k

Vt

5
m/sec, t = 18 sec.
3

Ex

75

18 =

5
18

75

Vt - 3 18
75

5 105
3 18 m/sec.

.B

Vt = 18

105

w
w

For the second person, put L = 0, Vt = 18 m/sec.


75

15 = 105
18

V2

105
18 - V2 = 5
15 18
V2 = 18 5 = 3 km/hr.

The speed of the second person is 3 km/hr.

Get all banking study material on BankExamsToday.com

Page 173

www.BankExamsToday.com, best study material for bank exams


CRT
6)

Two trains of lengths 190 m and 210 m respectively, are running in opposite directions on parallel tracks. If their
speeds are 40 km/hr and 32 km/hr respectively, in what time will they cross each other?
Sol: Using basic formula t =
Since

Lt L
Vt V

Lt = 190 m, L = 210 m, Vt = 40, V = -32

co
m

Vt V = 40 (-32) = 72 km/hr
5
18 = 20 m/sec
190 210
t=
20

Hence

t = 20 seconds
The two trains cross each other in 20 seconds.
7)

od
ay
.

= 72

When two trains were running in the same direction at 90 km/hr and 60 km/hr respectively, the faster train crossed a
man in the slower train in 27 seconds. Find the length of the faster train.

am
sT

Sol: It is given that the faster train crosses a man in the slower train in the same direction. It implies that a train
crosses a man moving in the same direction.
Speed of slower train = speed of the man (because the man is in the slower train)
Now, using the basic formula, we get t =

Lt
(90

60)

5
18

an
k

27 =

Ex

For the man, put L = 0

Lt L
, where L1 = length of faster train
Vt V

Lt = 30

5
18

27 = 225

w
w

.B

Hence, the length of the faster train is 225 meters.

Get all banking study material on BankExamsToday.com

Page 174

www.BankExamsToday.com, best study material for bank exams


CRT
BOATS AND STREAMS
Still Water: If the speed of the water in the river is Zero, then the water is called Still water. (Stationary Water)
Stream: If the water of the river is moving, it is called a stream. (Water in motion)

co
m

Upstream: If a boat or a swimmer moves against the stream i.e. in the direction opposite to that of the stream, it
is called upstream.
Downstream: If a boat or a swimmer moves with the stream i.e. along the direction of the stream, it is called
downstream.

am
sT

od
ay
.

Note: If the speed of a boat or a swimmer is given, it usually means the speed in still water.

Formulae:

If the speed of a boat or a swimmer be x km/hr and the speed of the stream or the current be y km/hr,
then

a.
b.

Speed of the boat or swimmer downstream = (x + y) km/hr.


Speed of the boat or swimmer downstream = (x - y) km/hr.

a.

an
k

2.

Ex

1.

Speed of the boat or swimmer in still water

b.

Speed of the stream

w
w

1
(Downstream speed - Upstream speed)
2

If a man is capable of rowing at the speed of x km/hr in still water, rows the same distance up and down a
stream which flows at a rate of y km/hr, then his average speed throughout the journey =
Upstream Downstream
Man's rate in still water

3.

1
(Downstream speed + Upstream speed)
2

.B

4.

y)(x
x

y)

km/hr.

A man can row a boat in still water at x km/hr. In a stream flowing at y km/hr, if it takes t hours more in
upstream than to go downstream for the same distance, then
The distance =

a.

(x

(x2

y2 )t
km.
2y

A man rows a certain distance downstream in t 1 hours and returns the same distance upstream in t 2 hours.
If the speed of the stream be x km/hr, then

Get all banking study material on BankExamsToday.com

Page 175

www.BankExamsToday.com, best study material for bank exams


CRT
t2
t2

The speed of the man in still water = y


b.

A man rows a boat in still water at x km/hr. In a stream flowing at y km/hr if it takes him t hours to row to
a place and come back, then
x2 y2
2x

The distance between the two places = t

km.

A boat or a swimmer takes n times as long to row upstream so as to row downstream the river. If the speed
of boat or swimmer be x km/hr and the speed of stream be y km/hr,
Then x

co
m

c.

t1
km/hr.
t1

n 1
.
n 1

od
ay
.

SOLVED EXAMPLES

The speed of a boat in still water is 10 km/hr. If the speed of the stream be 2 km/hr, then find its
downstream and upstream speeds.
Sol:
Speed of the boat (x) = 10 km/hr.
Speed of the stream (y) = 2 km/hr.
Downstream speed = x + y = 10 + 2 = 12 km/hr.
Upstream speed = x y = 10 2 = 8 km/hr.

2.

A boat is rowed down a river 32 km in 4 hours and up a river 15 km in 3 hours. Find the speed of the boat
and the river.
Sol: Speed of the boat downstream =

am
sT

1.

32
4

15
3

5 km/hr.

Ex

Speed of the boat upstream =


1
2
1
=
2
1
=
2
1
=
2

(Downstream speed + Upstream speed)

an
k

Speed of the boat

8 km/hr.

Sol: Average speed =

4.

13
2

6.5 km/hr.

(Downstream speed - Upstream speed)


(8 - 5) =

3
2

1.5 km/hr.

A man rows a boat at a speed of 16 km/hr in still water to a certain distance upstream and back to the
starting point in a river which flows at 8 km/hr. Find h is average speed for total journey.

w
w

3.

.B

Speed of the river

(8 + 5) =

Upstream Downstream (x y)(x y)


Man's rate in still water
x
(16 8)(16 8) 24 8
12 km/hr.
=
16
16

A man can row 4 km/hr in still water. If the river is running at 3 km/hr, it takes 12 hours more in upstream
than to go downstream for the same distance. How far is the place?
Sol: The required distance =

(x2

y2 )t
2y

(42

32 ) 12
2 3

Get all banking study material on BankExamsToday.com

10 km.

Page 176

www.BankExamsToday.com, best study material for bank exams


CRT
A motorboat covers a certain distance downstream in 8 hours but takes 10 hours to return upstream to the
starting point. If the speed of the stream be 14 km/hr, then find the speed of the motor boat in still water.
Sol: Speed of the motorboat in still water = y

t2
t2

t1
km/hr
t1

10 8
10 8

= 14

Sol: The required distance = t

x2 y2
km
2x

= 10
7.

82 42
2 8

10 48
16

30 km.

co
m

A man can row 8 km/hr in the still water. If the river is running at 4 km/hr, it takes him 10 hours to row to
a place and back. How far is the place?

A man can row at the rate of 16 km/hr in still water. If the time taken to row a certain distance upstream is
7 times as much as to row the same distance downstream, find the speed of the current.
Sol: Speed of the man =
16 =

n 1
speed of the current
n 1
7
7

am
sT

6.

126 km/hr.

od
ay
.

5.

1
speed of the current
1

So, speed of the current = 12 km/hr.

Ex

Exercise

The speed of a boat with the current is 18 km/hr and its speed against the current is 12 km/hr. What is the speed of
boat in still water?
1. 15 km/hr
2. 12 km/hr
3. 18 km/hr
4. 8 km/hr

2.

A mans rate against current is 11 km/hr and with the current is 15 km/hr. What is the speed of stream?
1. 8 km/hr
2. 10 km/hr
3. 2 km/hr
4. 5 km/hr

3.

A boat can row 12 km upstream in 48 minutes and 10 km with the current in 30 minutes. What is the speed of boat in
still water?
1. 1.5 km/hr
2. 2.5 km/hr
3. 3.5 km/hr
4. 4 km/hr

.B

w
w

A boat can row 36 km down the stream in 6 hours. if the speed of flow is 2 km/hr, find the time required for the boat
to travel the same distance against the flow.
1. 18 hrs
2. 15 hrs
3. 24 hrs
4. None

4.

an
k

1.

5.

The speed of a boat in still water is 17.5 km/hr and the rate of current is 2.5 km/hr. The distance traveled in
downstream in 25 minutes is____?
1. 18 km

2. 8

1
km
3

3. 5

1
km
2

4. 6 km/hr

6.

A man rows 3 km/hr in still water. If the river is running at 1 km/hr, it takes him 45 minutes to row to a place and
back. How far is the place?
1. 8 km
2. 1 km
3. 5 km
4. 6 km

7.

A boat goes 80 km downstream in 8 hours and 70 km upstream in 14 hours. Find the speed of boat in still water.
1. 2.5 km/hr
2. 10 km/hr
3. 5 km/hr
4. 7.5 km/hr

Get all banking study material on BankExamsToday.com

Page 177

www.BankExamsToday.com, best study material for bank exams


CRT
A man can row 80 km downstream in 5 hours and 30 km upstream in the same time. The speed of the stream is
______?
1. 6 km/hr
2. 1.5 km/hr
3. 5 km/hr
4. 2.8 km/hr

9.

A man can row 9 km/hr in still water and he finds that it takes him twice as long to row upstream as to row down the
river. Find the rate of the stream.
1. 1 km/hr
2. 2 km/hr
3. 3 km/hr
4. None

10.

A man can row 50 km downstream in 5 hours and 20 km upstream in 10 hours. Find the speed of the stream.
1. 4 km/hr
2. 8 km/hr
3. 3 km/hr
4. 6 km/hr

11.

A man can row with a speed of 15 km/hr in still water. If the stream flows at 5 km/hr, then his speed in downstream
is ________
1. 10 km/hr
2. 5 km/hr
3. 20 km/hr
4. 22 km/hr

12.

A man can row upstream at 25 km/hr and downstream at 35 km/hr then find the speed of the man in still water.
1. 30 km/hr
2. 10 km/hr
3. 60 km/hr
4. 5 km/hr

13.

A man can row upstream at 21 km/hr and downstream at 25 km/hr, then the speed of current is______?
1. 4 km/hr
2. 2 km/hr
3. 23 km/hr
4. 46 km/hr

14.

A man swims downstream 30 km and upstream 18 km taking 3 hours each time. What is the speed of the man in still
water?
1. 2 km/hr
2. 8 km/hr
3. 16 km/hr
4. 4 km/hr

15.

A man can row 6 km/hr in still water. When the river is running at 1.2 km/hr, it takes him 1 hour to row to a place
and back. How far is the place?
1. 3.12 km
2. 2.88 km
3. 3 km/hr
4. 2 km/hr

16.

A man can row 25 km/hr in still water and the stream is flowing at 15 km/hr and it takes him 15 hours to row to a
place and back. How far is the place?
1. 12 km
2. 60 km
3. 120 km
4. 240 km

17.

The current of a stream runs at the rate of 4 km/hr. A boat goes 6 km and back to the starting point in 2 hours, then
find the speed of boat in still water.
1. 10 km/hr
2. 21 km/hr
3. 8 km/hr
4. 6 km/hr

18.

The current of a stream flows at 1 km/hr. A boat goes 35 km upstream and back to the starting point in 12 hours. The
speed of the boat in still water is ___?
1. 2 km/hr
2. 10 km/hr
3. 8 km/hr
4. 12 km/hr

od
ay
.

am
sT

Ex

an
k

.B

w
w

A man whose speed is 4.5 km/hr in still water rows to a certain upstream point and back to the starting point in a
river which flows at 1.5 km/hr. Find his average speed for the entire journey.
1. 8 km/hr
2. 4 km/hr
3. 2 km/hr
4. 10 km/hr

19.

co
m

8.

20.

Speed of a boat in still water is 7 km/hr and speed of the stream is 1.5 km/hr. A distance of 7.7 km in upstream is
covered in how many minutes?
1. 2 km/hr
2. 1.5 km/hr
3. 3.5 km/hr
4. 4 km/hr

21.

A man can row downstream at 18 km/hr and upstream at 10 km/hr. Find the speed of the man in still water and the
speed of stream (in km/hr).
1. 13; 3
2. 12; 6
3. 15; 3
4. 14; 4

Get all banking study material on BankExamsToday.com

Page 178

www.BankExamsToday.com, best study material for bank exams


CRT
A man can swim at 6 km/hr downstream and 4 km/hr upstream. Find the speed of the man in still water and also the
speed of the stream.
1. 1; 5
2. 5; 1
3. 6; 2
4. 2; 6

23.

A man can swim downstream at 8 km/hr and upstream at 2 km/hr. Find mans rate in still water and the speed of
current.
1. 5; 3
2. 3; 5
3. 15; 3
4. 3; 15

24.

A boat can cover 2 km against the stream in 20 minutes and return in 15 minutes. Find the speed of boat in still water
and the speed of current.
1. 8; 3
2. 3; 6
3. 8; 1
4. None

25.

A man rows 10 km upstream and back again to the origin point in 55 minutes. If the speed of the stream is 2 km/hr,
find the speed of rowing in still water.
1. 11 km/hr
2. 22 km/hr
3. 33 km/hr
4. None

26.

A boat goes down stream at U m/sec and upstream at V m/sec. Then the speed of boat in still water (in m/sec) is
_______
1.

1
(U - V)
2

2. U V

3.

od
ay
.

co
m

22.

1
(U + V)
2

4. U + V

The speed of a boat in still water is 15 km/hr and the rate of stream is 5 km/hr. Find the distance traveled downstream
in 24 min.
1. 4 km
2. 8 km
3. 6 km
4. 16 km

28.

If a mans downstream rate is 10 km/hr and the rate of stream is 1.5 km/hr, then the mans upstream rate is ______
1. 13 km/hr
2. 10 km/hr
3. 3 km/hr
4. 7 km/hr

29.

If a man rows at 8 km/hr in still water and his upstream rate is 5 km/hr, then the mans rate along the current is
______
1. 21 km/hr
2. 8 km/hr
3. 16 km/hr
4. 11 km/hr

30.

The rowing speed of man in still water is 20 km/hr. Going downstream, he moves at the rate of 25 km/hr. Find the
rate of stream.
1. 45 km/hr
2. 2.5 km/hr
3. 12.5 km/hr
4. 5 km/hr

31.

If a man goes upstream at 6 km/hr and the rate of stream is 2 km/hr, then the mans speed in still water is ________
1. 4 km/hr
2. 8 km/hr
3. 2 km/hr
4. 12 km/hr

32.

A boat goes 12 km upstream in 48 minutes. The speed of stream is 2 km/hr. The speed of boat in still water is
_______
1. 13 km/hr
2. 17 km/hr
3. 15 km/hr
4. None

Ex

an
k

.B

w
w

A boat takes 4 hours for traveling downstream from point A to point B and coming back to point A upstream. If the
velocity of the stream is 2 km/hr and the speed of the boat in still water is 4 km/hr, what is the distance between A
and B?
1. 8 km
2. 9 km
3. 4 km
4. 6 km

33.

am
sT

27.

34.

A boat goes 20 km downstream in 1 hour and the same distance upstream in 2 hours. The speed of boat in still water
is ______.
1. 15 km/hr
2. 10 km/hr
3. 5 km/hr
4. 7.5 km/hr

35.

A boat takes 9 hours to travel upstream and 3 hours to travel the same distance downstream. If its speed in still water
is 4 km/hr, what is the velocity of the stream?
1. 4 km/hr
2. 3 km/hr
3. 6 km/hr
4. None

Get all banking study material on BankExamsToday.com

Page 179

www.BankExamsToday.com, best study material for bank exams


CRT
A person can swim at 7.5 km/hr in still water. In a river with 1.5 km/hr current, the person swims to a certain distance
and comes back within 50 minutes. What is the distance between the two points?
1. 3 km
2. 4 km
3. 2 km
4. 1 km

37.

A man rows a boat in 18 km in 4 hours downstream and returns upstream in 12 hours. The speed of the stream is
____ km/hr.
1. 1
2. 1.5
3. 1.75
4. 2

38.

A boat goes 8 km in 1 hour along the stream and 2 km in 1 hour against the stream. The speed of the stream in km/hr
is _____.
1. 2
2. 3
3. 4
4. 5

39.

In one hour a boat goes 11 km along the stream and 5 km against the stream. The speed of the boat in still water in
km/hr is _____.
1. 6
2. 8
3. 9
4. 5

40.

The speed of a boat is 15 km/hr in still water. If the speed of the stream is 3 km/hr, and it takes 15 hours for the boat
to reach to a place and return. What distant is the place from stating point?
1. 96 km
2. 54 km
3. 72 km
4. 108 km

an
k

11) 3
12) 1
13)2
14) 2
15) 2
16) 3
17) 3
18)1
19) 2
20) 2

21) 4
22) 2
23) 1
24) 4
25) 2
26) 3
27) 2
28) 4
29) 4
30) 4

w
w

.B

1) 1
2) 3
3) 2
4) 1
5) 2
6) 2
7) 4
8) 3
9) 3
10) 1

Ex

Answers

am
sT

od
ay
.

co
m

36.

Get all banking study material on BankExamsToday.com

Page 180

31) 2
32) 2
33) 4
34) 1
35) 4
36) 1
37) 2
38) 2
39) 2
40) 4

www.BankExamsToday.com, best study material for bank exams


CRT
PIPES AND CISTERNS
Pipes: Generally the pipes are connected to tank or cistern and are used to fill or empty the tank.
Inlet: A pipe connected with a tank or a cistern that fills it is known as inlet.

co
m

Outlet: A pipe connected with a tank or cistern emptying it is known as outlet.


Note:

Pipes and Cistern problems are similar to those on Time and Work.
The only difference here is, the work done is in terms of filling or emptying a cistern and the time taken
by a pipe or leak (Crack) to fill or empty a cistern respectively.
Generally, the time taken to fill a cistern is taken as positive and the time taken to empty a cistern is
taken as negative.
The amount of work done i.e., filling or emptying a cister n is generally taken as unity, unless otherwise
specified.

3.
4.

od
ay
.

1.
2.

am
sT

Formulae:

1
.
X

1.

If an inlet can completely fill the empty tank in X hours, the part of the tank filled in 1 hour =

2.

If an outlet can empty the full tank in Y hours, the part of the tank emptied in 1 hour =

3.

If both inlet and outlet are open, net part of the tank filled in 1 hour =

4.

Two pipes A and B can fill or empty a cistern in X and Y hours respectively, while working alone. If

Ex

1
X

1
.
Y

both the pipes are opened together, then the time taken to fill or empty the cistern =
three pipes are opened together, the time taken to fill the cistern =

XYZ
hours.
ZX XY

w
w
YZ

A tank takes X hours to be filled by a pipe. But due to a leak, it is filled in Y hours. The amount of time

in which the leak can empty the full tank =

8.

XYZ
hours.
YZ ZX

Note: This type of formulae can be generated by replacing negative sign wherever a pipe starts emptying
the cistern instead of the standard positive sign.
Two pipes A and B can fill a cistern in X hours and Y hours, respectively. There is also an outlet C. if
all the three pipes are opened together, the tank is full in Z hours. The time taken by C to empty the full
tank =

7.

XY

.B

6.

XY
hours.
X Y

Three pips A, B and C can fill a cistern in X, Y and Z hours respectively, while working a lone. If all the

an
k

5.

1
.
Y

XY
hours.
Y X

A cistern has a leak which can empty it in X hours. A pipe which allows Y litres of water per hour into
the cistern is turned on and now the cistern is emptied in Z hours. The capacity of the cistern is
XYZ
litres.
Z X

9.

One fill pipe A is k times faster than the other fill pipe B.
a) If B can fill a cistern in x hours, then the time in which the cistern will be full, if both the full
pipes are opened together, is

x
k

hours.

Get all banking study material on BankExamsToday.com

Page 181

www.BankExamsToday.com, best study material for bank exams


CRT
b) If A can fill a cistern in y hours, then the time in which the ciste rn will be full, if both the full
k

pipes are opened together, is

yhours.

10. If one fill pipe A is k times faster and takes x minutes less time than the other fill pipe B, then

b) B will fill the cistern in

x
k

minutes.

kx
k 1

minutes.

co
m

a) A will fill the cistern in

SOLVED EXAMPLES
1.

1
.
4

am
sT
1
20

Ex

A pipe can empty a cistern in 20 minutes. Find the time in which


2
part of the cistern is emptied in 20
5

2
5

2
part of the cistern will be emptied.
5

8 minutes.

an
k

Sol:

A pipe can empty a cistern in 20 hours. Find the part of the cistern emptied in 4 hours.
1
20

.B

Sol: The part of the cistern emptied in 1 hour =

So, the part of the cistern emptied in 4 hours = 4

1
20

1
.
5

A tap can fill a cistern in 8 hours and another can empty it in 12 hours. If both the taps are opened
simultaneously, find the time in hours to fill the cistern.

w
w

5.

minutes.

1
1
part of the tank is filled in 20
= 4 minutes.
5
5

So,

4.

1)2

1
part of the tank will be filled.
5

A pipe can fill a tank in 20 minutes. Find the time in which


Sol: The part of the tank filled in 1 minute =

3.

(k

A pipe can fill a tank in 4 hours. Find the part of tank filled in one hour.
Sol: The part of tank filled in 1 hour =

2.

kx

od
ay
.

c) The time taken to fill a cistern, if both the pipes are opened together is

Sol: Here, X = 8 and Y = 12

Part of the cistern filled in 1 hour =

1
X

1
Y

1
8

1
12

1
.
24

Total time taken to fill the cistern = 24 hours.

6.

Two pipes A and B can fill a cistern in 20 and 30 minutes, respectively. If both the pipes are
simultaneously then find how long will it take to fill the cistern?
Sol: Here, X = 20 and Y = 30
Part of the cistern filled by (A + B) in 1 minute =
1
20

1
30

1
20

Get all banking study material on BankExamsToday.com

1
30

5
60

Page 182

1
12

www.BankExamsToday.com, best study material for bank exams


CRT
Hence, both the pipes together will fill the cistern in 12 minutes.
7.

Two pipes A and B can separately fill a cistern in 4 hours and 6 hours respectively, while a third pipe C
can empty it in 3 hours. In what time will the cistern be full, if all the pipes are opened together?
Sol: Here, X = 4, Y = 6 and Z = - 3

8.

4 6
3
(4 6) (6
3) (4

3)

12 hours.

Two taps A and B can fill a cistern in 15 and 30 minutes respectively. There is a third exhaust tap C at
the bottom of tank. If all taps are opened at the same time, the cistern will be full in 25 minutes. In what
time can exhaust tap C empty the cistern when full?

C can empty the full tank in

od
ay
.

Sol: Here, X = 15, Y = 30 and Z = 25


XYZ
YZ ZX XY

(15 30)

15 30 25
minutes
(30 45) (15 45)
10 minutes.

am
sT

15 30 25
1125

9.

72
6

co
m

So, the cistern will be full in

A pipe can fill a tank in 6 hours. Due to leakage in the bottom, it is filled in 12 hours. If the tank is full,
how much time will the leak take to empty it?
Sol: Here, X = 6 and Y = 12

Ex

So, the time taken by the leak to empty the full tank =
6 12
12 6

12hours.

an
k

XY
hours
Y X

10. A leak in the bottom of a tank can empty the full tank in 3 hours. An inlet pipe fills water at the rate of 2
litres per minute. When the tank is full, the inlet is opened and due to leak, the tank is empty in 5 hours .
Find the capacity of the tank.

.B

Sol: Here, X = 3, Y = 2 x 60 = 120 and Z = 5


XYZ
litres
Z X

w
w

So, the capacity of the tank =

3 120 4
5 3

720 litres.

11. One fill pipe A is 2 times faster the second fill pipe B. If A can fill a cistern in 9 minutes, then find the
time when the cistern will be full if both fill pipes are opened together.
Sol: Here, k = 2 and y = 9

So, Cistern will be full in

k
k 1

2
9
2 1

6 minutes.

12. One fill pipe A is 7 times faster the second fill pipe B takes 72 minutes less than th e fill pipe B. When
will the cistern be full if both fill pipes are opened together?
Sol: Here, k = 7 and x = 72,

Cistern will be full in

Get all banking study material on BankExamsToday.com

kx
(k

Page 183

1)

72

(7

1)2

14 minutes.

www.BankExamsToday.com, best study material for bank exams


CRT
Exercise
A tap can fill a cistern in 16 hours and another can empty it in 32 hours. If both the taps are opened simultaneously,
find the time (in hours) to fill the tank.
1. 30
2. 32
3. 48
4. None

2.

A cistern normally takes 7 hours to be filled by a tap, but because of a leak it takes one hour more. In how many
hours will the leak empty a filled cistern?
1. 56
2. 52
3. 50
4. 72

3.

Pipe P can fill an empty tank in 5 hours while pipe Q can empty the full tank in 8 hours. If both are opened in that
empty tank, in how much time the tank will be filled?
2. 13 hrs

3. 13

1
hrs
3

4. None

od
ay
.

1. 12 hrs

co
m

1.

A tap can fill a cistern in 12 hours and another empties in 10 hours. If both are opened at the same time, how much
time will it take to fill the cistern?
1. 3 hrs
2. 20 hrs
3. 5 hrs
4. Never

5.

Two pipes P and Q can fill a cistern in 12 minutes and 18 minutes respectively, while a third pipe R can empty it in 9
minutes. In what time will the cistern will be full, if they all work simultaneously?
1. 30 min
2. 34 min
3. 36 min
4. 42 min

6.

A pipe can fill a cistern in 4 hours. What part of cistern is filled in 60 minutes?
1.

1
6

2.

am
sT

4.

1
4

3.

1
2

4.

1
3

Two pipes P and Q separately fill a cistern in 7 and 5 minutes respectively and drain pipe R can carry off 14 litres
per minute. If all the pipes are opened, when the cistern is full, it is emptied in one hour. How many litres does it
hold?
1. 40 litres
2. 55 litres
3. 45 litres
4. 60 litres

8.

Two pipes P and Q can fill a cistern in 60 minutes and 80 minutes respectively. Both the taps were opened
simultaneously but Q is turned off after 20 minutes. Find the total time required to fill the cistern.
1. 20 min
2. 35 min
3. 45 min
4. 1 hr

9.

Three taps can fill a cistern in 10, 15 and 18 minutes respectively. In an empty cistern all the three taps are kept open
and after three minutes the third tap is closed. In how many minutes more will the cistern be full?
1. 1 min
2. 3 min
3. 2 min
4. 4 min

an
k

.B

A tap can fill a cistern in 20 minutes. And another can fill it in 30 minutes. If both are opened simultaneously, find
the time when the cistern will be full?
1. 10 min
2. 15 min
3. 20 min
4. 12 min
Two pipes P and Q can fill a cistern in 10 minutes and 15 minutes respectively. But an empty pipe R can empty it in
5 minutes. The pipes P and Q are opened for 4 minutes and then the empty pipe R is also opened. In what time is the
cistern emptied?
1. 10 min
2. 20 min
3. 15 min
4. 12 min

11.

w
w

10.

Ex

7.

12.

A cistern is filled by two taps in A and B hours respectively. Then emptied by a tap in C hours. If all the three taps
are opened simultaneously, the cistern is filled in F hours. The relation between F, A, B and C are given by
_________.
1. F = (A + B) - C

2. F =

A x b
c

Get all banking study material on BankExamsToday.com

3.

1
1 1 1
= + +
F
A B C

Page 184

4. F =

1
1 1
+
A B

1
C

www.BankExamsToday.com, best study material for bank exams


CRT
Two pipes P and Q can separately fill a cistern in 12 minutes and 15 minutes respectively. While a third pipe R can
empty it in 6 minutes. The pipes P and Q are kept open for 5 minutes, then the pipe R is also opened. In what time the
cistern be emptied?
1. 30 min
2. 35 min
3. 40 min
4. 45 min

14.

Two pipes P and Q can separately fill a cistern in 12 minutes and 24 minutes respectively. And a waste can drain off
20 gallons per minute. If all the three pipes are opened, the tank is filled in 24 minutes. What is the capacity of the
tank?
1. 360 gallons
2. 380 gallons
3. 240 gallons
4. 320 gallons

15.

A tank can be filled separately by two pipes P and Q in 45 and 36 minutes respectively. A tap R at the bottom can
empty the full cistern in 30 minutes. If the tap R is opened 7 minutes after the two pipes P and Q are opened, find
when the tank becomes full.
1. 39 min
2. 45 min
3. 49 min
4. 59 min

od
ay
.

1
hours. It now takes
2
half-an-hour longer. If the cistern is full, how long it would take the leakage to empty the tank, if the water leaks out
at double the rate after half the cistern becomes empty.
1. 10 hrs 15 min
2. 11 hrs 15 min
3. 12 hrs 15 min
4. 13 min

There is a leak in the bottom of the cistern. When there was no leak, the cistern was filled in 2

am
sT

16.

co
m

13.

An empty cistern has three taps P, Q and R. P and Q can fill it in 3 and 4 hours respectively. R can empty it in one
hour. If P, Q and R are kept opened at 1 P.M., 2 P.M., 3 P.M., respectively, find when the cistern will be empty?
1. 3:20 PM
2. 5:20 PM
3. 5:12 PM
4. 6:20 PM

18.

An electric pump can fill a tank in 3 hours. Because of a leak, it took 3 hour to fill the tank. In how much time the
leak can drain the full tank?
1. 21 hrs
2. 20 hrs
3. 32 hrs
4. 25 hrs

19.

Two pipes P and Q can fill a cistern in 1 hour and 75 minutes respectively. There is also an outlet R. If all the three
pipes are opened together, the tank is full in 50 minutes. How much time will be taken by R to empty the full tank?
1. 50 min
2. 70 min
3. 100 min
4. 120 min

20.

Two pipes P and Q can fill a cistern in 24 minutes and 32 minutes respectively. If both are opened simultaneously,
after how much should Q be closed so that the tank is full in 18 minutes?
1. 8 min
2. 12 min
3. 15 min
4. 13 min

21.

Two pipes P and Q can fill a cistern in 15 and 20 hours respectively, while a third pipe R can empty the full tank in
25 hours. All the three pipes are opened. After 10 hours R is closed. The tank will be full in how many hours?
1. 10 hrs
2. 12 hrs
3. 15 hrs
4. 6 hrs

an
k

.B

w
w

A tank has a leak, which could empty it in 8 hours. A tap is turned on which admits 6 litres a minute into the cistern
and it is now emptied in 12 hours. How many litres does the cistern hold?
1. 8840 litres
2. 8540 litres
3. 8000 litres
4. 8640 litres

22.

Ex

17.

23.

Two taps can fill a tank in 12 and 18 minutes respectively. Both are kept opened for 2 minutes and the first is turned
off. In how many minutes more will the tank be filled?
1. 15 min
2. 20 min
3. 11 min
4. 13 min

24.

Two pipes P and Q can fill a cistern in 12 minutes and 18 minutes respectively. They are turned on at the same time.
If the tap P is turned off after 4 minutes, how long will tap Q take to fill the rest cistern?
1. 8 min
2. 9 min
3. 7 min
4. 10 min

Get all banking study material on BankExamsToday.com

Page 185

www.BankExamsToday.com, best study material for bank exams


CRT
25.

If two pipes functions simultaneously, the reservoir will be filled in 12 hours. One pipe fills the reservoir 10 hours
faster than the other. How many hours does it take the second pipe along to fill the reservoir?
1. 25 hrs
2. 20 hrs
3. 6 hrs
4. 30 hrs

26.

Two pipes can fill a cistern in 14 hours and 16 hours respectively. The pipes are opened simultaneously and it was
found that due to leakage in the bottom, it took 32 minutes more to fill the cistern. When the cistern is full, in what
time will the leak empty it?
2. 8 hrs

3.

112
hrs
15

4. 7 hrs 28 min.

co
m

1. 112 hrs

Two pipes P and Q can fill a cistern in 24 minutes and 32 minutes respectively. If both are opened simultaneously,
after how much time Q should be closed, so that the tank is full in 18 minutes?
1. 12 min
2. 10 min
3. 8 min
4. 6 min

28.

A cistern is 2/5th full. Pipe P can fill the total tank in 10 minutes and Pipe Q can empty it in 6 minutes. If both the
pipes are opened, how long will it take to empty or fill the tank completely?
1. 6 min to empty
2. 6 min to fill
3. 9 min to fill
4. None of these

29.

Two pipes P and Q can fill a cistern in 12 minutes and 15 minutes respectively, while a third pipe R can empty the
full cistern in 6 minutes. P and Q are kept open for 5 minutes in the beginning and then R is also opened. In what
time the cistern is emptied?
1. 30 min
2. 45 min
3. 32 min
4. 33 min

30.

Three pipes P, Q and R can fill a tank in 6 hours. After working at it together for 2 hours, R is closed and P and Q can
fill the remaining part in seven hours. The number of hours taken by R alone to fill the tank is ________.
1. 10
2. 12
3. 16
4. 14

31.

One pipe can fill a tank 3 times as fast as another pipe. If together the two pipes can fill the tank in 36 minutes, then
the slower pipe alone will be able to fill the tank in __________.
1. 81 min
2. 144 min
3. 108 min
4. 192 min

32.

Bucket P has thrice the capacity as Bucket Q. It takes 60 turns for Bucket P to fill the empty drum. How many turns
will it take for both the buckets P and Q having each turn together to fill the empty drum?
1. 30
2. 45
3. 40
4. 90

33.

Two pipes can fill a cistern in 20 minutes and 24 minutes respectively. And a drain pipe can empty 3 gallons per
minute. All the three pipes working together can fill the tank in 15 minutes. The capacity of the tank is ________.
1. 60 gallons
2. 100 gallons
3. 120 gallons
4. 180 gallons

34.

Three taps P, Q and R can fill a tank in 12, 15 and 20 hours respectively. If P is opened all the time and Q and R are
opened for 1 hour each alternatively, the tank will be full in __________.

w
w

.B

an
k

Ex

am
sT

od
ay
.

27.

2. 6

2
hours
3

3. 7

1
hours
2

4. 7 hours

1. 6 hours

35.

Two taps can fill an empty tank in 36 minutes and 60 minutes respectively. A third tap can empty the full tank in 90
minutes. At what time will an empty tank be filled if all the three are opened simultaneously at 8.50 A.M.?
1. 9:00 AM
2. 9:20 AM
3. 9:30 AM
4. 10:00 AM

36.

Two pipes can fill half of the tank in 20 minutes. If one of the pipe fills the whole tank in 60 minutes, find the time
taken by the other pipe to fill the tank individually?
1. 90 min
2. 135 min
3. 100 min
4. 120 min

37.

Two pipes P and Q can fill a tank in 40 and 60 minutes respectively. When the tank will be filled, if the pipes are
opened in alternative minutes starting with pipe P?
1. 24 min
2. 49 min
3. 47 min
4. 48 min

Get all banking study material on BankExamsToday.com

Page 186

www.BankExamsToday.com, best study material for bank exams


CRT
38.

Two pipes P and Q can fill a cistern in 20 minutes and 30 minutes respectively. Another leak pipe R can empty the
whole tank in 60 minutes. A man opened the two filling pipes and came to stop them when the tank should be filled
but he finds the leak pipe R is also opened and thus he stopped the leak pipe R. Find the time taken by the pipes to fill
the tank after the leak pipe was closed.
1. 2

2
min
5

2. 3 min

3. 4 min

4. 3

1
min
3

Three pipes P, Q and R are connected to a cistern. P and Q together can fill the cistern in 12 hrs, Q and R together in
20 hrs and R and P together in 15 hrs. In how much time will each pipe fill the cistern separately?
1. 10 hrs; 15 hrs; 30 hrs 2. 20 hrs; 15 hrs; 60 hrs
3. 20 hrs; 30 hrs; 60 hrs 4. 20 hrs; 30 hrs; 45 hrs

40.

A pipe can fill a tank in 20 minutes and another pipe in 30 minutes, but a third pipe can empty it in 10 minutes. The
first two pipes are kept open for 8 minutes in the beginning and then the third pipe is also opened. In what time is the
cistern emptied?
1. 35 min
2. 38 min
3. 40 min
4. 30 min

Answers
21) 2
22) 4
23) 4
24) 1
25) 4
26) 1
27) 3
28) 1
29) 2
30) 4

am
sT

11) 2
12) 4
13) 4
14) 3
15) 1
16) 2
17) 3
18) 1
19) 3
20) 1

w
w

.B

an
k

Ex

1) 2
2) 1
3) 3
4) 4
5) 3
6) 2
7) 1
8) 3
9) 3
10) 4

od
ay
.

co
m

39.

Get all banking study material on BankExamsToday.com

Page 187

31) 2
32) 2
33) 3
34) 4
35) 2
36) 4
37) 3
38) 1
39) 3
40) 3

Você também pode gostar